Ευκλειδης Β 61

Page 1

Ελληνική Μαθηματική Εταιρεία


I ' ' ! t

,. I

/

_)

/

ι

...

; i

�_)

I

j

/

i

_/,·

'ι I I

ΕΚΠΑΙΔΕΠΙΚΑ ΒΙΒΛΙΑ

yιa μαΒπτίς και εκπαιδευτικοιίς

MAPI.A fΥΣfλθΙΟΥ

Ει'ΙΕΥθΕΡ!ΟΣ ΠΡΟΤΟΠΜΑί

ΜΕθΟΔΟΛΟΓΙΑ

Γεωμετρία

Μεθοδολογία Άλγεβρας

Γεωμετρία

Α' Ενιαίου Λυκείου

Α' Ενιαίου Λυκείου

Μ. Ευσταθίου, Ε. Πρωτοπαπάς

Β' Ενιαίου Λυκείου Γενικής Παιδείας

Γ. Βιδάλης, Β. Γκιμίσης

Ε. Πρωτοπαπάς

ΜΕθΟΔΟΛΟΓΙΑ

μαθηματικά Γ' Ενιαίου Λυκείου

� Uli.Ottlt φ ιuι:rΛΚΗ

Γενικής Παιδείας

Γενικής Παιδείας

Θετικής και Τεχνολογικής κατεύθυνσης

Α' τόμος: Ε. Πρωτοπαπάς

Μ. Τσιλπιρίδης

Ε. Πρωτοπαπάς

ΕΚΔΟΕΕΙΕ

ΠΑΤΑΚΗ tπην

Γ' Ενιαίου Λυκείου

Γ' Ενιαίου Λυκείου

Β' Ενιαίου Λυκείου

πρωτοπορ-ία

Μαθηματικά

Μαθηματικά

Μεθοδολογία Άλγεβρας

εmαίδεvση

σ

ε:

ό

ι4a.

Β' τόμος: Ε. Πρωτοπαπάς, Σ. Γκούμας τ

α.

b

ι

b

ι4 ι

ο

n

w

ι4 ε:ίa.

ΒΙΒΛΙΟΠΩΛΕΙΟ ΠΑΤΑΚΗ: ΑΚΑΔΗΜΙΑΣ 65, I 06 78 ΑΘΗΝΑ. ΤΗΛ. 210.38.11.850 KENTPIKH ΔΙΑΘΕΣΗ: ΕΜΜ . ΜΠΕΝΑΚΗ 16, I 06 78 ΑΘΗΝΑ. ΤΗΛ. 210.38.31.078 ΥΠΟΚ/ΜΑ: Ν. ΜΟΝΑΠΗΡΙΟΥ 122. 563 34 ΘΕΣΣΑΛΟΝΙΚΗ, ΤΗΛ. 2310.70.63.54 Web site: http://www.patakis.gr - e-mail: info@patakis.gr. sales@patakis.gr


ΕΛΛΗΝΙΚΗ ΜΑΘΗΜΑΤΙΚΗ ETAIPEIA 60 ο

Τεύχος

Ιούλιος

Αύyουστος

ο

ο

2006ο

Σεπτέμβριος

ΜΑΘΗΜΑΤΙΚΟ

ΠΕΡΙΟΔΙΚΟ

Αθ'

Έτος

eomail: info@hms.gr www.hms.gr

ΓΙΑ

ΤΟ

3,50

Ευρώ:

ο

ΛΥΚΕΙΟ

ΠΕΡΙΕΧΟΜΕΝΑ

./ Γράμμα από το Δ.Σ. της Ε.Μ.Ε

./ Αθανάσιος Φωκάς

.

.

.

.

.

.

.

.

.

.

.

.

.

.

.

.

.

ο

.

.

.

. . .

.

.

.

.

ο

. . . .

.

.

.

.

.

.

. .

.

.

.

.

.

.

.

.

.

.

.

./ Ιστορικές Μαθηματικές Αναφορές - γωνία της Ιστορίας των Μαθηματικών ./ Το Βήμα του Ευκλείδη

./ Homo· Mathematicus

.

.

.

.

.

.

.

.

.

.

.

.

. . .

.

.

.

.

.

.

. . .

.

. .

.

.

.

.

ο

.

.

. . . . . .

.

.

Μαθη,μ,οτ·ικά Α'

./ Άλγεβρα

.

./ Γεωμετρία

ο

. . . . . . .

.

.

.

.

Τάξης •

ο

.

.

ο

ο

. . .

.

.

.

.

.

.

.

.

.

. .

.

.

.

.

.

.

.

.

.

.

.

.

2

3

5

11

22

26

................................................................... 35

./ Γεωμετρία

./ Άλγεβρα

.

.

. .

Τάξη;ς

Μα:θιημ:ατικά Β.' ο

.

.

.

.

.

.

.

.

.

.

.

.

.

.

.

.

.

.

.

.

.

.

.

.

.

.

.

.

.

.

.

.

.

ο

./ Κατεύθυνση

44 49

58

Γ" Τά.ξη:ς

Μαθ,ηματι,κά

./ Μαθηματικά Κατεύθυνσης

.

./ Μαθηματικά Γενικής Παιδεία

./ Ο Ευκλείδης προτείνει

•••

.

. .

.

. . . .

.

.

.

.

Ευκλείδη και

./ Τα Μαθηματικά μας Διασκεδάζουν

.

.

.

.

•••

ο

.

.

.

.

.

.

.

. . . .

.

.

. . . . . . . .

Διόφαντο

ο

.

.

.

.

.

.

.

.

.

.

.

.

.

.

.

.

.

.

.

.

.

.

.

.

.

.

.

.

.

.

.

.

.

. .

.

.

.

. •

.

.

.

.

.

.

.

.

.

.

.

.

.

.

.

.

.

. •

.

.

.

.

.

. . .

.

.

.

.

62

68

74

79

·······························································�··············

ΕΚΔΟΣΗ ΤΗΣ ΜΑΘΗΜΑτJΚΗΣ ΠΑΙΡΕΙΑΣ

ΠΑΝΕΠΙΠΗΜΙΟΥ 34

Τηλ.: 210 3617784 Fox:2103641025

ο

ο

106 79 ΑΘΗΝΑ

3616532

Εκδότης: Εξαρχάκος Θcόδωρος

Διcuθuντής: Τuρλής Ιω6ννης

Κωδικός ΕΛ.ΤΑ.: 2055 ISSN:

1105-7998

Επιμέλεια 'Εκδοσης:

Κυριακόπουλος Αντώνης Ευσταθίου Βαγγέλης

Εκτελεστική Γραμματεία

Πρόεδρος: Κυριακόπουλος Αντώνης

Αντιπρόεδροι:

Α·: Ευσταθίου Βαyyέλης

ι·: Ταααόnουλος Γιώργος

Γραμματέας: Χριστόπουλος Παναγιώτης

Μέλη:

Αργυράκης Δ. Δρούτσας Π. Λουρίδας Σ. Ταπcινός Ν.

Σuvταvτική επιτροπή

Αθανασόπουλος Γεώργιος Ανδρουλακάκης Νίκος Αντωνόπουλος Νίκος Αργυράκης Δημήτριος Βακαλόπουλος Κώστας Βλάχου Αγγελική Γράψας Κων/νος Δρούτσας Παναγιώτης Ευσταθίου Βαγγέλης Ζαχαρόπουλος Κων/νος Ζώτος Βαγγέλης Καλίκας Σταμάτης Καρακατσάνης Βασίλης Καρκάνης Βασίλης Κατσούλης Γιώργος Κερασαρίδης Γιάννης Κηπουρός Χρήστος Κόντζιας Νίκος Κοτσιφάκης Γιώργος Κυριακόπουλος Αντώνης Κυριακόπουλος Θανάσης Κυβερνήτου Χρυστ.

Λαζαρίδης Χρήστος Λουρίδας Σωτήρης Μαλαφέκας Θανάσης Μεταξάς Νικόλαος Μυλωνάς Δημήτρης Μώκος Χρήστος Ρέγκλης Δημήτρης Σσ:tτη Εύα Σταθόπουλος Γεώργιος Σταϊκος Κώστας Στάϊκος Παναγιώτης Στρατής Γιάννης Ταπεινός Νικόλαος Τασσόπουλος Γιώργος Τριάντος Γεώργιος Τσαγκάρης Ανδρέας Τσικαλουδάκης Γιώργος Τσιούμας Θανάσης Τυρλής Ιωάννης; Χαραλαμποπούλου Λίνα Χαραλάμπους Θάνος Χριστόπουλος Παναγιώτης

•••••••••••••••••••••••••••••••••••••••••••••••••••••••••••••••••••••••••••••• •

Τα διαφημιζόμενα βιβλία δε σημαίνει ότι προτείνονται από την Ε.Μ.Ε. Οι συνεργάτες, τα άρθρα, οι προτεινόμενες ασκήσεις, οι λύσεις ασκήσεων κτλ. πρέπει να στέλνονται έγκαιρα, στα γραφεία της Ε.Μ.Ε. με την ένδειξη "Για τον Ευκλείδη β"'. Τα χειρόγραφα δεν επιστρέφονται. τιμή Τεύχους ευρώ 3,50 Ετήσια σuνδQομή (10,00 + 4,00 Ταχυδρομικά εUQώ 14,00) Ετήσια συνδρομή για Σχολεία εUQώ 10,00 Το αντίτιμο για τα τεύχη που παραγγέλνονται στέλνεται με απλή επιταγή σε διαταγή Ε.Μ.Ε. Ταχ. Γραφείο Αθήνα 54 Τ.Θ. 30044 ή πληρώνεται στα γραφεία της Ε.Μ.Ε. =

Εκτύπωαη: /ΝτΕΡΓΙΡΕΣ Α.Ε. τηλ.: 210 8160330 Υneιίluνος τunoypaφeίou: 8. Σωτηριάδης


Γράμμα από

το

Δ.Σ. της Ε.Μ.Ε ..

Αγαπητοί συνάδελφοι, Μια νέα σχολική χρονιά αρχίζει και το Δ.Σ. της Ελληνικής Μαθηματικής Εταιρείας εύχεται σε όλους σας να είναι επιτυχημένη και δημιουργική. Με την έναρξη της χρονιάς αυτής θα ήθελε επίσης να εγκαινιάσει έναν νέο τρόπο επικοινωνίας και ενημέρωσης μέσα από τις περιοδικές εκδόσεις της Εταιρείας. Θα θέλαμε, έτσι, να υπενθυμίσουμε ότι πέρα από τα περιοδικά ΕΥΚΛΕΙΔΗΣ Α' και ΕΥΚΛΕΙΔΗΣ Β·. όπου δημοσιεύονται άρθρα, μέσα στα πλαίσια σύγχρονων παιδαγωγικών επιστημονικών και διδακτικών μεθόδων κατάλληλα για όλες τις ηλικίες των μαθητών του Γυμνασίου και του Λυκείου αντίστοιχα απαραίτητο βοήθημα στο μαθηματικό ερευνητή και στο μαθηματικό εκπαιδευτικό είναι οι εκδόσεις Ευκλείδης Γ και Μαθηματική Επιθεώρηση. Ο Ευκλείδης Γ δημοσιεύει ερευνητικά αποτελέσματα του πεδίου της διδακτικής των Μαθηματικών και της Μαθηματικής Εκπαίδευσης, καθώς και άρθρα που συμβάλλουν στην ευρύτερη και βαθύτερη κατάρτιση όσων διδάσκουν μαθηι.;.ατικά Το περιοδικό Μαθηματική Επιθεώρηση (Μ.Ε.) έχει στόχο να ενημερώνει τους συναδέλφους μαθηματικούς της χώρας μας σε θέματα Μαθηματικών, Ιστορίας και Φιλοσοφίας των Μαθηματικών, Εφαρμογών

των

Μαθηματικών,

Εκπαιδευτικής Έρευνας,

Ειδικών

θεμάτων,

Παιδαγωγικών

θεμάτων κ.λπ.

Ο Μικρός Ευκλείδης, πρωτοποριακή έκδοση στο χώρο της Α/θμιας Εκπαίδευσης, μπαίνει στην σκέψη των μαθητών του Δημοτικού Σχολείου. Είναι ένα εργαλείο για την προσέγγιση των Μαθηματικών με νέες ιδέες και ευρηματικότητα ώστε να αγαπήσουν οι μικροί μαθητές τα Μαθηματικά. Πρέπει ακόμη να σας ενημερώσουμε για την επανέκδοση του Αστρολάβου με θέματα πάνω στις Νέες Τεχνολογίες της Πληροφορίας και Επικοινωνίας. Η Ελληνική Μαθηματική Εταιρεία ζητά τη συμμετοχή σας στην έκδοση των περιοδικών αυτών αλλά και τη συνδρομή σας ώστε να γίνουν γνωστά και απαραίτητα σε μαθητές και εκπαιδευτικούς. Ένα ακόμη σημαντικό θέμα είναι η προσπάθεια Ύια την τροποποίηση του Καταστατικού Όπως γνωρίζετε στη συνεδρίαση του Δ.Σ. της Ε.Μ.Ε. της 1ης Φεβρουαρίου 2006 και σύμφωνα με την ημερησία διάταξη, συζητήθηκε η ομόφωνη πρόταση που κατέθεσε η 8μελής Επιτροπή που είχε οριστεί με ομόφωνη απόφαση της Γενικής Συνέλευσης της 27ης Φεβρουαρίου 2005. Το Δ.Σ. αποφάσισε ομόφωνα να αποδεχθεί την πρόταση, όπως αυτή έλαβε τελική μορφή μετά από νομοτεχνική επεξεργασία και την έθεσε υπόψη Καταστατικής Γενικής Συνέλευσης του 2006. Την πρόταση σε πλήρη μορφή μπορείτε να τη βρείτε στην ιστοσελίδα της Ε.Μ.Ε. www.hms.gr, ή να την προμηθευτείτε σε έντυπη μορφή από τα γραφεία της εταιρείας. Το πρωτοποριακό εκλογικό σύστημα που προτείνεται είναι μια σύζευξη του ενιαίου ψηφοδελτίου και της απλής αναλογικής όπως προκύπτει από την πρόταση της επιτροπής . Επειδή στην Καταστατική Συνέλευσης που πραγματοποιήθηκε το Φεβρουάριο του 2006 δεν υπήρξε απαρτία το Δ.Σ. της Ε.Μ.Ε. πιστεύει ότι πρέπει να γίνει μια νέα προσπάθεια για να πραγματοποιηθεί νέα Καταστατική Συνέλευση στην αρχή του 2007, θεωρώντας δεδομένη την βοήθεια σας για την ενεργοποίηση όλων των μελών, προκειμένου να υπάρξει ένα θετικό αποτέλεσμα Τέλος, θα θέλαμε να σας υπενθυμίσουμε την κορυφαία ετήσια εκδήλωση στο χώρο της Μαθηματικής Παιδειας, το 23° Πανελλήνιο Συνέδριο Μαθηματικής Παιδείας με θέμα ((Τα μαθηματικά

ως πολιτισμός στο σύγχρονο κόσμο, προl:κτάσεις στην ιφιτιι.:ιί σιt·έψη, στην

που θα γίνει στις 24, 25 και 26 Νοεμβρίου στην Πάτρα. Ο Γενικός Γραμματέας Ο Πρόεδρος Καθηγητής Θεόδωρος Εξαρχάκος Ιωάννης Τυρλής

επιχειρηματολογία και στην αισθητικιίη

ΕΥΚΛΕΙΔΗΣ Β' τ. Ι /2


Αθανάσιος Φωκάς

Ένας σύγχρονος σκαπανέας της Επιστήμης αντάξιος των μεγάλων διανοητών του 19ου αιώνα. του Γιάννη Δ . Στρατή

Α. Φωκάς γεννήθηκε στην Κεφαλονιά το 1 952 και πήρε εκεί τη βασική εγκύκλια μόρφωση μέχρι την προτελευταία τάξη του Γυμνασίου, την εβδόμη (έτσι λέγανε τότε η σημερινή Γ ' Λυκείου) την έβγαλε στην Αθήνα όπου έκανε φροντιστήριο προκειμένου να δώσει εισαγωγικές εξετάσεις για το Πολυτεχνείο. Δεν τα κατάφερε ! .:::.επέρασε γρήγορα το σοκ της aπογοήτευσης (ήταν άριστος μαθητής) κι έφυγε στο εξωτερικό, στην Αγγλία, αν και αγνοούσε πλήρως την αγγλική γλώσσα. Έπεσε με τα μούτρα στην εκμάθηση των αγγλικών (έφτασε να μαθαίνει 300 λέξεις τη μέρα) και χωρίς καμιά καθυστέρηση πήρε το πτυχίο Αεροναυπηγικής απ ' το Ιμπιριαλ Κολλετζ του Λονδίνου, πέρνωντας και βραβείο ως καλύτερος φοιτητής. Στη συνέχεια ξεκίνησε ένα διδακτορικό στην αεροναυπηγική, σύντομα ωστόσο διαπίστωσε ότι χρειαζόταν μαθηματικά υψηλού επιπέδου. Του συνέστησαν το Ινστιτούτο Τεχνολογίας της Καλιφόρνια (ΚΑΛΤΕΚ) των Η.Π.Α. όπου είχε ξεκινήσει ένα νέο τμήμα Εφαρμοσμένων Μαθηματικών με υψηλές προδιαγραφές. Εκεί ερωτεύθηκε τα Μαθηματικά και αποφάσισε να τους αφοσιωθεί ολοκληρωτικά. Τελικά πήρε διδακτορικό δίπλωμα στα Μαθηματικά με άριστες επιδόσεις. Το 2000 η Μαθηματική Εταιρεία του Λονδίνου του απένειμε το βραβείο Νέιλορ για την ανακάλυψη μιας τεχνικής επίλυσης διαφορικών εξισώσεων. Το βραβείο αυτό είναι απ ' τα πιο σημαντικά για τη Μαθηματική Επιστήμη και την Μαθηματική Φυσική κι έχουν τιμηθεί με το βραβείο αυτό εξέχουσες επιστημονικές προσωπικότητες, όπως ο Ρ. Πενρόουζ και ο Σ. Χώκινγκ. Σήμερα είναι καθηγητής στην έδρα Μη Γραμμικών Επιστημών του Πανεπιστημίου Κέιμπριτζ. Η έδρα αυτή ανήκει στο τμήμα Εφαρμοσμένων Μαθηματικών και Θεωρητικής Φυσικής, όπου έχουν διδάξει μεγάλες φυσιογνωμίες της Επιστήμης όπως ο Ισαάκ Νιούτον και ο Στίβεν Χώκινγκ. Μία από τις ερευνητικές περιοχές του Α. Φωκά είναι η λύση μαθηματικών προβλημάτων σχετικών με την μελέτη του ανθρώπινου εγκεφάλου καθώς βρίσκεται σε λειτουργία. Παρατηρούμε ότι το 1986 πήρε δίπλωμα Ιατρικής απ ' το Πανεπιστήμιο του Μαϊάμι Η.Π.Α. . Σε συνεργασία με τον Ίσραελ Γκέλφαντ (Ρώσος μαθηματικός και βιολόγος παγκόσμιας ΕΥΚΛΕΙΔΗΣ Β' τ. Ι /3


Αθανάσιος Φωκάς

αναγνώρισης) με τη συνδρομή και άλλων επιστημόνων αναλόγου ειδικού βάρους, συνέβαλε στη λύση του κύριου μαθηματικού προβλήματος στο οποίο στηρίζεται η λεγόμενη τεχνική SPECT η οποία μας δίνει την δυνατότητα να παρατηρούμε, με όλο και μεγαλύτερη ακρίβεια τον ανθρώπινο εγκέφαλο καθώς λειτουργεί. Η επίλυση του προβλήματος αυτού, που πραγματοποιήθηκε πολύ πρόσφατα, ήταν κατά κοινή παραδοχή , εξαιρετικά δυσκολότερη από την επίλυση του μαθηματικού προβλήματος, στο οποίο στηρίχθηκε ο Α. Κόρμακ ( φυσικός και μαθηματικός, που τιμήθηκε με το βραβείο Νόμπελ της Σουηδικής Ακαδημίας Επιστημών το 1979) για την κατασκευή του αξονικού τομογράφου. Ο αξονικός τομογράφος και ο μαγνητικός τομογράφος (για την ανακάλυψη του οποίου τιμήθηκε με το βραβείο Νόμπελ το 2003 ο Πήτερ Μάνσφηλντ) έφεραν πραγματική επανάσταση στη Νευρολογία. Και όχι μόνο, ολόκληρος ο κλάδος της Ιατρικής Επιστήμης επηρεάστηκε θετικά. Παρ' όλα αυτά οι σημαντικότατες αυτές τεχνικές δίνουν στατικές εικόνες (στιγμιότυπα). Αν και είναι λεπτομερέστατες δεν μας επιτρέπουν να παρακολουθούμε , καθώς δουλεύει, ένα όργανο ζώντος οργανισμού, πολύ περισσότερο τον ανθρώπινο εγκέφαλο. Το Γενάρη του 2005 ο Πρόεδρος της Δημοκρατίας Κωνσταντίνος Στεφανόπουλος απένειμε στον Α. Φωκά το παράσημο του Τάγματος του Φοίνικα. Το παράσημο αυτό απονέμεται σε ανθρώπους που διακρίθηκαν στη Δημόσια Διοίκηση, τις Επιστήμες, τις Τέχνες και τα Γράμματα, στο Εμπόριο, τη Βιομηχανία και τη Ναυτιλία. Την Άνοιξη του 2004 τιμήθηκε με το Αριστείο της Ακαδημίας Αθηνών και το Δεκέμβρη του ίδιου χρόνου εκλέχτηκε μέλος της. Είναι το νεότερο μέλος της Ακαδημίας Αθηνών ( από αρκετούς επιστήμονες και διανοούμενους στο εξωτερικό η σύγχρονη Ακαδημία μας θεωρείται συνέχεια της Ακαδημίας του Πλάτωνα). Επίσης έχει ανακηρυχθεί επίτιμος διδάκτορας του Εθνικού Μετσοβείου Πολυτεχνείου και του Πολυτεχνείου Κρήτης, καθώς επίσης και των Πανεπιστημίων Πάτρας και Αθήνας. Το Μάρτη του 2005 , ανταποκρινόμενος στις παραινέσεις του δάσκαλου I. Γκέλφαντ, ανέλαβε πρόεδρος του Εφορευτικού Συμβουλίου της Εθνικής (Βαλλιάνειου) Βιβλιοθήκης με μόνο κίνητρο όπως δηλώνει «να συμβάλλει στο να κατακτήσει η Βιβλιοθήκη τη θέση που της αρμόζει, στον Ευρωπαϊκό χώρο». Τώρα ζει στο Cambridge με τη σύζυγό του Ρεγγίνα (γλωσσολόγος) και τα τρία παιδιά τους Αλέξανδρο, Αναστασία, Ιωάννα. Όπως λέει ο Α. Φωκάς «όλες αυτές οι εξαιρετικές τιμές με γεμίζουν βέβαια συγκίνηση, αλλά κυρίως μεγαλώνουν ακόμα περισσότερο το βαθύτατο αίσθημα ευθύνης που νιώθω. Γιατί όλες αυτές τις μεγάλες τιμές δεν τις είδα ποτέ σαν αναγνώριση του έργου μου, αλλά σαν κατάθεση προσδοκιών για συνέχιση της πορείας μου». ΕΥΚΛΕΙΔΗΣ Β ' τ. l/4


Ε πιμέλεια: Χ ρή στος Κηπουρ ό ς

Τα Προ- Ευκλείδεια Μαθηματικά ια τα αρχαία ελληνικά μαθηματικά, ο Ευ­ Γ' κλείδης κατέχει ένα ρόλο παρόμοιο με αυ­

. . . . τόν του Σωκράτη στην ελληνική φtλοσοφία. Είναι δηλαδή ο επιστήμονας που με το έρyο του χω­ ρίζει την ιστορία των αρχαίων ελληνικών μαθηματι­ κών σε δύο διακριτές ιστορικές περιόδους: την προ­ Ευκλείδεια και την μετα-Ευκλείδεια. Η πρώτη από τις δύο αυτές περιόδους, η προ-Ευκλείδεια, παρου­ σιάζει ιδιαίτερο ιστορικό ενδιαφέρον για δύο κυρίως λόγους: ο ένας είναι ότι ιστορικά έχει μελετηθεί πολύ λιγότερο από την μετα-Ευκλείδεια, και ο δεύτερος είναι ότι αρκετά από τα παγιωμένα συμπεράσματα της μαθηματικής ιστοριογραφίας ανατρέπονται από τα στοιχεία που φέρνει στο φως η σύγχρονη ιστορική έρευνα. Για την ιστορική εξέταση της προ-Ευκλείδειας περιόδου υπάρχει μια πολύ μεγάλη αντικειμενική δυσκολία: η παντελής έλλειψη πρωτευουσών ιστορι­ κών πηγών, δηλαδή μαρτυριών από πρώτο χέρι. Ως πρωτεύουσες ιστορικές πηγές θεωρούνται τα πραγ­ ματικά αποσπάσματα του έρyου ενός συγγραφέα, ή έστω η κατά λέξη αξιόmστη μεταφορά τους από κά­ ποιον άλλο . Σπανίζουν όμως και οι δευτερεύουσες ιστορικές πηγές, δηλαδή οι μαρτυρίες πρώιμων αυθεντιών, ό­ πως είναι ο Πλάτων, ο Αριστοτέλης και οι άμεσοι μαθητές τους. Η παλαιότερη δευτερεύουσα ιστορική πηγή που έχουμε για την προ-Ευκλείδεια περίοδο είναι δύο αποσπάσματα από το μη διασωθέν έρyο "Γεωμετρι­ καί Ιστορία" του μαθητή του Αριστοτέλη Εύ δη μου του Ρόδιου (β ' μισό 4ου αι. π.Χ.). Το πρώτο από αυ-

Στέλιος Λα μνής

τά αναφέρεται στον τετραγωνισμό των μηνίσκων από τον Ιπποκράτη τον Χίο και το δεύτερο στον τε­ τραγωνισμό του κύβου από τον Αρχύτα τον Ταρα­ ντίνο. Εννοείται πως οι δευτερεύουσες πηγές ενέχουν ήδη ένα στοιχείο αβεβαιότητας, εφόσον οι πρωτότυ­ πες έννοιες μπορεί να έχουν διαστρεβλωθεί, παρου­ σιαζόμενες έτσι ώστε να συμμορφώνονται προς έναν πλατωνικό ή aριστοτελικό τρόπο σκέψης.

Με την ευκαιρία να πούμε ότι ιστορίες των ελ­ ληνικών μαθηματικών στην προ-Ευκλείδεια περίοδο, εκτός από τον Εύδημο, έγραψαν και οι Ξενοκράτης ο Χαλκηδό νιος (4ος αι. π.Χ.) και Θε όφραστος ο Λέ­ σ βιος (372-287 π.Χ.). Το δυστύχημα είναι ότι κανένα από τα δύο αυτά έρyα δεν διασώθηκε. Ιστορία των μαθηματικών στην μετα-Ευκλείδεια περίοδο έγραψε και ο Γεμίνος ο Ρό διος (περί το 70 π.Χ.), η οποία λέγεται ότι στηρι­ ζόταν στην ιστορία του Εύδημου, αλλά δυστυχώς κι αυτή δεν διασώθηκε.

ΕΥΚΛΕΙΔΗΣ Β' τ. l/5


-------

Ευκλείδης

Ιστορικές Μαθηματικές Αναφορέ

Ό,τι γνωρίζουμε για τα προ-Ευκλείδεια μαθημα­ τικά ουσιαστικά προέρχεται από μεταγενέστερους συγγραφείς, οι περισσότεροι από τους οποίους έζη­ σαν μέχρι και Ι 000 χρόνια αργότερα από την περίο­ δο που εξετάζουμε. Είναι άγνωστο αν αυτοί οι συγ­ γραφείς (δοξογράφοι, υπομνηματιστές, σχολιογρά­ φοι, χρονικογράφοι κ.λ.π.) είχαν πρόσβαση σε πηγές ή σε πρωτογενές υλικό. Το πιθανότερο είναι ότι βα­ σίζονταν σε επιτομές έργων που έγιναν από την aρι­ στοτελική σχολή (π.χ. η ιστορία του Εύδημου), οπότε είναι πιθανό οι μαρτυρίες τους να υπέστησαν δια­ στρεβλώσεις κατά τη διαδικασία της συνεχούς μεταβίβασης. Αυτές θα μπορούσαν να ονομαστούν τρι­ τεύουσες πηγές, αν και το μεγαλύτερο μέρος τους δεν είναι καν από τρίτο χέρι. Στις τριτεύουσες πηγές ανήκει ο Πρόκλος ο Λύ ­ κειος ή Βυζάντιος (410-485 μ.Χ), που θεωρείται σήμερα η πιο αξιόλογη πηγή πληροφοριών για την ιστορία της προ-Ευκλείδειας Γεωμετρίας. Ο Πρό­ κλος δεν ήταν ιστορικός ούτε έγραψε ποτέ ιστορία των μαθηματικών. Έγραψε σχόλια στο πρώτο Βιβλίο των "Στοιχείων" του Ευκλείδη, το "Υπόμνημα εις το πρώτον των Ευκλείδου Στοι χείων", όπου περιέχονται βιογραφικά και βιβλιογραφικά στοιχεία για αρκετούς αρχαίους Έλληνες γεωμέτρες και τα έργα τους. Στο έργο του Πρόκλου υπάρχει μια ολιγοσέλιδη (σελίδες 65-70 του βιβλίου) παρουσίαση της εξέλιξης της ελ­ ληνικής Γεωμετρίας από τους αρχαιοτάτους χρόνους, η οποία ονομάζεται σήμερα "Σύνοψις ". Στις τριτεύουσες πηγές ανήκουν και οι σχολια­ στές των έργων άλλων αρχαίων μαθηματικών. Τέ­ τοιοι σχολιαστές ήταν ο Θ έων ο Σμυρναίος (2°ς αι. μ.Χ.), ο Πάππος ο Α λεξανδρινός (3ος αι. μ.Χ.), ο Θ έων ο Αλε ξ αν δ ρινός (320-395 μ.Χ.) και η κόρη

του Υπατία (375-415 μ.Χ.), ο Ισίδωρος ο Μιλή σιος και ο μαθητής του Ευτόκιος (6ος αι. μ.Χ.), ο Σιμπλί­ κιος (6ος αι. μ. Χ.) και άλλοι. Με βάση τα παραπάνω στοιχεία είναι φανερό ότι οι γνώσεις μας για την προ-Ευκλείδεια περίοδο αντι­ κειμενικά είναι ελάχιστες και αποσπασματικές. Προ­ κύπτουν έτσι μερικά εύλογα ερωτήματα σχετικά με την φερεγγυότητα των πληροφοριών που έχουμε σήμερα για την ιστορία των προ-Ευκλείδειων μαθη­ ματικών. Το πρώτο ερώτημα αφορά στον τρόπο με τον οποίο παρουσιάστηκαν οι μαθηματικοί της προ­ Ευκλείδειας περιόδου από τους αρχαίους ιστορικούς. Θα διαmστώσουμε ότι υπάρχουν περιπτώσεις μαθη­ ματικών της προ-Ευκλείδειας περιόδου που αντιμε­ τωπίστηκαν με φανερή ιδεολογική προκατάληψη. Η μομφή αφορά κυρίως τον Πρόκλο, που όπως είπαμε θεωρείται η mo αξιόπιστη που διαθέτουμε για τα προ-Ευκλείδεια μαθηματικά. Ο Πρόκλος στη Σύνο­ ψή του (σελ. 65.21-66.18), παραθέτει ένα κατάλογο των Ελλήνων γεωμετρών που άκμασαν στην περίοδο μεταξύ Πυθαγόρα και Πλάτωνα. Σ' αυτόν τον κατά­ λογο περιλαμβάνονται κατά χρονολογική σειρά εμ­ φάνισης τα ακόλουθα ονόματα: Ι) Αναξαγόρας ο Κλαζομένιος (500-428 π.Χ.), 2) Οινοπίδης ο Χίος (περί το 440 π.Χ.), 3) Ιπποκράτης ο Χίος (470-400 π.Χ.), 4) Θεόδωρος ο Κυρ η ναίος (περί το 390 π.Χ.), 5) Λεωδάμας ο Θάσιος (περί το 400 π.Χ.), 6) Αρχύ τας ο Ταραντίνος (428-365 π.Χ.) και 7) Θεαίτητος ο Αθηναίος (417-369 π.Χ.). Η πρώτη παρατήρησή μας είναι ότι από τα επτά ονόματα που αναφέρονται, τα τέσσερα τελευταία σχετίζονται άμεσα με τον Πλάτωνα και τη σχολή του. Ο Θεόδωρος ο Κυρηναίος αναφέρεται από τον Ιάμβλιχο ως Πυθαγόρειος, ενώ σύμφωνα με μια άλ­ λη πληροφορία υπήρξε δάσκαλος του Πλάτωνα στα μαθηματικά. Ο Λεωδάμας ο Θάσιος υπήρξε μαθητής του Πλάτωνα και σύμφωνα με τον Πρόκλο διδάχτη­ κε από τον Πλάτωνα την αναλυτική μέθοδο, με την οποία ο ίδιος ανακάλυψε τις γεωμετρικές προτάσεις που του αποδίδονται. Ο Αρχύτας ο Ταραντίνος ήταν γνωστός Πυθαγόρειος και επιστήθιος φίλος του Πλά­ τωνα, τον οποίο διέσωσε από βέβαιο θάνατο όταν ο

ΕΥΚΛΕΙΔΗΣ Β ' τ. t /6


Ιστορικές Μαθηματικές Αναφορές-------

τελευταίος βρέθηκε στην Κάτω Ιταλία. Τέλος, ο Θεαίτητος ήταν μαθητής και φίλος του Πλάτωνα, αλλά αναφέρεται επίσης και ως μαθητής του Θεόδω­ ρου του

Ιπποκράτης

όνομα του Αβδηρίτη φιλοσόφου δεν αναφέρεται ού­ τε μία φορά. Υπάρχει μάλιστα η μαρτυρία του Αρι­ στόξενου, ενός μαθητή του Αριστοτέλη, ότι ο Πλά­ των απεχθανόταν τόσο πολύ τον Δημόκριτο, ώστε σχεδίαζε να κάψει όσα συγγράμματά του είχε κατορ­ θώσει να συγκεντρώσει. Τον συγκράτησαν όμως οι Πυθαγόρειοι Αμύκλας και Κλεινίας, οι οποίοι του εξήγησαν ότι κάτι τέτοιο ήταν άσκοπο γιατί τα βι­ βλία του Δημόκριτου ήταν πολύ διαδεδομένα και τα είχαν πλέον στην κατοχή τους και πολλοί άλλοι (Δι­ ογένης Λαέρτιος, ΙΧ. 40). Στο σημείο αυτό να λά­ βουμε υπόψιν μας ότι από τα πολυάριθμα συγγράμ­ ματα του Δημόκριτου δεν διασώθηκε ούτε ένα, σε αντίθεση με τα έργα του Πλάτωνα που σώθηκαν όλα. Πασίγνωστη επίσης είναι η απέχθεια του Πλάτω­ να και προς τους Σοφιστές. Στη διαμάχη του όμως προς τους Σοφιστές, ο Πλάτων επέλεξε την τακτική της ευθείας αντιπαράθεσης, γι' αυτό και οι απόψεις των Σοφιστών αποτέλεσαν το θέμα αρκετών πλατωνι­ κών διαλόγων. Ποια ήταν όμως η βαθύτερη αιτία αυ­ τής της διαμάχης; Την απάντηση μας δίνει ο καθηγη­ τής Χαράλαμπος Θεοδωρίδης, ο οποίος στο βιβλίο του ''Επίκουρος. Η αληθινή όψη του αρχαίου κόσμου" (σελ. 63), εmσημαίνει: «Δε χρειάζεται μεγάλος κόπος

δεύτερη παρατήρησή μας είναι ότι από τον κατάλογο του Πρόκλου λείπουν τα ονόματα ορισμέ­ νων πραγματικά αξιόλογων μαθηματικών αυτής της περιόδου, όπως του Δημόκριτου του Αβδηρίτη (460-370 π.Χ.) και επίσης και των Σοφιστών Ιππία του Ηλείου (β ' μισό του 5ου αι. π.Χ), Αντιφώντα του Αθηναίου (περί 430 π.Χ.) και Βρύσωνα του Ηρακλείωτη (περί 430 π.Χ.). Οι τέσσερις αυτοί δια­ νοητές, παρά την εmκρατούσα αντίληψη, ασχολήθη­ καν με τα μαθηματικά και μάλιστα είχαν διατυπώσει γι(Χ να εννοήσει κανείς τη λύσσα της αριστοκρατίας για πρωτοποριακές μαθηματικές θεωρίες που προηγήθη­ τους "νεωτεροποιούς", που έβαζαν άλλη μια φορά σε καν κατά πολύ της εποχής τους. Επειδή είναι αδύνα­ κίνδυνο τα ιερά και τα όσια. Πολέμησαν με πείσμα τη το η μαθηματική συνεισφορά του Δημόκριτου και σοφιστική. Αντίθετα στην καινούργια αγωγή σκόρπισαν των Σοφιστών να μην ήταν γνωστή στην αρχαιότητα, το σύνθημα να ξαναγυρίσουν στην "πάτριον αγωγήν", το πιθανότερο είναι ότι δεν έχουμε να κάνουμε με όπως δημοκοπούσαν οι υπερπατριώτες με το πίσω στην μια αθώα ή τυχαία "αβλεψία" του Πρόκλου, αλλά με "πάτριον πολιτείαν", εννοώντας μ' αυτό την πολιτεία μια συνειδητή επιλογή του. Γιατί όμως ο Πρόκλος που οι ίδιοι ποθούσαν. Ο Πλάτων μια ζωή πολέμησε τους σοφιστές μ' επιχειρήματα λίγο πειστικά, που άμα παρέλειψε αυτά τα ονόματα; Μια πρώτη εξήγηση είναι ότι η "αβλεψία" του τα εξετάσεις προσεχτικά περισσεύει η εκτίμησή σου στο Πρόκλου οφείλεται στη γνωστή σε όλους ιδεολογική πρόσωπό τους. Στην προσπάθειά του είχε πολυάριθ­ αντιπαλότητα του Πλάτωνα προς τον Δημόκριτο και μους βοηθούς, που δυνατότερός τους στάθηκε η νοητι­ τους Σοφιστές. Ο Πλάτων δεν είχε μαζί τους απλώς κή και συναισθηματική κατάσταση των χρόνων ύστερα επιστημονικές ή φιλοσοφικές διαφορές, είχε πλήρη από τον [ενν. Πελοποννησιακό] πόλεμο. Ώσπου κατά­ φεραν να παραμορφώσουν και να κάνουν αγνώριστες ιδεολογική και πολιτική διάσταση. Ενδεικτική της αντιπάθειας· του Πλάτωνα προς τις μεγάλες εκείνες φυσιογνωμίες». Ο Πρόκλος τώρα ήταν γνωστός νέο-πλατωνικός τον Δημόκριτο είναι η πλήρης απάλειψη του ονόμα­ τος του Δημόκριτου από το σύνολο των πλατωνικών φιλόσοφος και ένας από τους τελευταίους διευθυντές διαλόγων. Σε όλους τους πλατωνικούς διαλόγους το της Ακαδημίας του Πλάτωνα. Ήταν ο άνθρωπος που Η

ΕΥΚΛΕΙΔΗΣ Β' τ. Ι /7


-------

Ιστορικές Μαθηματικές Αναφορέ

συστηματοποίησε τη νεοπλατωνική φιλοσοφία και της έδωσε την τελειωτική της μορφή, αυτήν με την οποία έγινε γνωστή στους μεταγενέστερους χρόνους. Αξίζει να εmσημάνουμε ότι όλα τα σωζόμενα φιλο­ σοφικά συγγράμματα του Πρόκλου σχετίζονται με τον πλατωνισμό (υπομνήματα στα πλατωνικά έργα

γνωρίζουμε για τα μαθηματικά του Δημόκριτου προέρχεται από δύο σύντομες αναφορές άλλων συγ­ γραφέων, μια του Αρχιμήδη και μια του Πλούταρ­ χου.

τίμαιος, Πολιτεία, Παρμενίδης, Αλκιβιάδης και Κρα­

Με βάση τα παραπάνω στοιχεία, η "αβλεψί­ α" του Πρόκλου φαίνεται ότι ήταν εσκεμμένη και οφείλετο στην φιλοσοφική τοποθέτησή του υπέρ του Πλατωνισμού. Είναι ικανοποιητική όμως η εξήγηση ότι ο Πρό­ κλος παρέλειψε τα ονόματα του Δημόκριτου και των Σοφιστών απλώς και μόνο επειδή θέλησε να συμ­ μορφωθεί με τις επιταγές του Πλάτωνα; Νομίζω πως όχι. Κάτι βαθύτερο πρέπει να κρύβεται πίσω από αυτήν την παράλειψη. Έχω την εντύπωση ότι ο Πρόκλος, όπως άλλω­ στε όλοι οι Πλατωνικοί, διαφωνούσε πρωτίστως με το είδος των μαθηματικών ενασχολήσεων των ιδεο­ λογικών αντιπάλων του. Ο Δημόκριτος και οι Σοφι­ στές ήταν συνεχιστές της μαθηματικής παράδοσης που είχαν εγκαινιάσει ο Θαλής και οι άλλοι Ίωνες φιλόσοφοι, δηλαδή της παράδοσης που έδινε προτε­ ραιότητα στα μαθηματικά των μετρήσεων και των υπολογισμών. Είναι όμως γνωστό πως αυτού του είδους τα μαθηματικά ήταν αδιανόητα για τους Πλα­ τωνικούς. Γι' αυτούς, τα μαθηματικά και ειδικά η γεωμετρία έπρεπε να αναφέρεται αποκλειστικά και μόνο στον κόσμο των άϋλων όντων, δηλαδή στον ιδεατό πλατωνικό κόσμο των Ιδεών. Η σύγκρουση Δημοκρίτου και Πλάτωνα λοιπόν δεν ήταν τίποτα άλλο παρά η διαμάχη υλισμού και ιδεαλισμού στον τομέα των μαθηματικών. Με τι είδους μαθηματικά όμως ασχολήθηκε ο Δημόκριτος; Από τους τίτλους των μαθηματικών συγγραμμάτων του [(1) "Περί διαφορής γνώμης ή τύλος).

Θαλής ο Μιλήσιος

Ο Αρχιμήδης, στην εισαγωγή της περίφημης "Μεθόδου" του, που ως γνωστόν ανακαλύφθηκε τυχαία το 1906, αναφέρει ότι ο Δημόκριτος ήταν ο πρώτος που διατύπωσε προτάσεις σχετικά με τη σχέ­ ση των όγκων του κώνου και του κυλίνδρου. Ο Δη­ μόκριτος βρήκε ότι ο όγκος ενός κώνου ισούται με το ένα τρίτο του όγκου ενός κυλίνδρου, που έχει την ίδια βάση και ίσο ύψος. Επίσης βρήκε ότι ο όγκος μιας πυραμίδας ισούται με το ένα τρίτο του όγκου ενός πρίσματος, που έχει την ίδια βάση και ίσο ύψος (Heath, σ. 223). Είναι άγνωστο αν ο Δημόκριτος είχε καταφέρει να αποδείξει τις παραπάνω προτάσεις, τις οποίες πάντως απέδειξε περίπου πενήντα χρόνια αρ­ γότερα ο Εύδοξος ο Κνίδιος.

περί ψαύσιος κύκλου και σφαίρας", (2) "Περί Γεωμε­

τρίης", (3) "Γεωμετρικών", (4) "Αριθμοί " (5) "Πε­ ,

ρί αλόγων γραμμών και ναστών", (6) 'Έκπετάσμα­

Αρχιμήδης ο Συρακούσιος

Ο Πλούταρχος στο βιβλίο του "Περί των κοινών δεν αναμένουμε καμιά βοήθεια, απλούστατα γιατί δεν διασώθηκε καμιά ασφαλής πληροφορία εννοιών προς τους Στωικούς", περιλαμβάνει μια δή­ σχετικά με το περιεχόμενό τους. Έτσι λοιπόν ό,τι λωση του Δημόκριτου σχετικά με τον όγκο του κώ­ νου. Σ' αυτήν ο Δημόκριτος φέρεται να δηλώνει ότι τα "]

ΕΥΚΛΕΙΔΗΣ Β ' τ. Ι /8


-------

Ιστορικές Μαθηματικές Αναφορέr------

αν ένας κώνος τμηθεί από ένα επίπεδο παράλληλο προς τη βάση του, τότε υπάρχει ένα ερώτημα σχετικά με το είδος της τομής. Είναι αυτές ίσες ή άνισες; Αν οι τομές είναι άνισες, τότε θα έχουμε ως αποτέλεσμα έναν ακανόνιστο κώνο, ο οποίος θα εμφανίζει πολλές εγκοπές, θα έχει δηλαδή κλιμακωτή δομή και ανω­ μαλίες. Αν οι τομές είναι ίσες, τότε ο κώνος θα απο­ τελείται από ίσους κύκλους, δηλαδή ουσιαστικά θα είναι κύλινδρος. Όπως παρατηρεί ο Heath, η φράση «ο κώνος θα αποτελείται από ίσους κύκλους» μας επιτρέπει να υ­ ποθέσουμε ότι ο Δημόκριτος είχε συλλάβει ήδη την ιδέα ότι κάθε στερεό σχήμα είναι το άθροισμα ενός απείρου πλήθους απείρως λεπτών παράλληλων εm­ πέδων, τα οποία βρίσκονται μεταξύ τους σε απεφο­ στή απόσταση(Ηeath, σ. 227). Είναι αδύνατο να μη συσχετίσει κανείς αυτή την ιδέα με την ατομική θε­ ωρία του Δημόκριτου. Συνεπώς, με τον Δημόκριτο έχουμε για πρώτη φορά στην ιστορία των μαθηματικών τη διατύπωση των βασικών αρχών της γεωμετρίας των απειροστών. Ο Δημόκριτος, δύο αιώνες δηλαδή πριν από τον Αρ­ χιμήδη, διατύπωσε τον πυρήνα μιας ριζοσπαστικής μαθηματικής ιδέας, την οποία όπως γνωρίζουμε υιο­ θέτησε και τελειοποίησε αργότερα ο μεγάλος Συρα­ κούσιος σοφός. Ας εξετάσουμε όμως τώρα και τα μαθηματικά των Σοφιστών. Από τους τρεις προαναφερθέντες Σοφιστές, πιο γνωστός για τη μαθηματική του συνει­ σφορά είναι ο Ιππίας ο Ηλείος. Αυτός επινόησε μια ειδική καμπύλη, γνωστή ως τετραγωνίζουσα του Ιπ­ πία, για την επίλυση του προβλήματος της τριχοτό­ μησης μιας γωνίας. Πολύ αξιόλογη όμως ήταν η μαθηματική συνει­ σφορά του Αντιφώντα και του Βρύσωνα. Ο Αντιφών ήταν ο πρώτος Έλληνας που είχε την ιδέα να υπολο­ γίσει θεωρητικά το εμβαδόν του κύκλου. Η ιδέα του, όπως διασώθηκε από τους Θεμίστιο και Σιμπλίκιο (H.Diels-W.Κranz, "Dίe Fragmente der Vorsok­ ratίker", σ. 340-341 ), ήταν να εγγ ράψει στον κύκλο ένα κανονικό πολύγωνο (ισόπλευρο τρίγωνο ή τε­ τράγωνο) και μετά να διχοτομεί συνεχώς τα σχηματι­ ζόμενα (ίσα) τόξα έτσι, ώστε να προκύπτουν συνε-

χώς κανονικά πολύγωνα που να καλύπτουν ολοένα και περισσότερο τον κυκλικό δίσκο. Ο Αντιφών πί­ στευε ότι έτσι θα κατέληγε σε ένα κανονικό πολύγω­ νο, του οποίου οι πλευρές, λόγω του μικρού μεγέ­ θους τους, θα συνέπιπταν με την περιφέρεια του κύ­ κλου. Περιττό βέβαια να πούμε ότι η ριζοσπαστική αυ­ τή ιδέα του Αντιφώντα προκάλεσε την οργή του πλα­ τωνικού μαθηματικού κατεστημένου. Η ιδέα του Α­ ντιφώντα θα παρέμενε για πάντα άγνωστη αν δεν είχε ερεθίσει τα συντηρητικά ανακλαστικά του Αριστοτέ­ λη, ο οποίος άθελά του την διέσωσε όταν θέλησε να της ασκήσει κριτική ("Φυσικά", 185a). Η γνώμη του Αριστοτέλη ήταν ότι δεν υπήρχε ανάγκη να αντικρού­ σει κανείς την ιδέα του Αντιφώντα, μιας και δεν βασί­ ζεται σε αναγνωρισμένες αρχές της Γεωμετρίας. Ο Βρύσων όχι μόνο αποδέχτηκε τη μέθοδο του Αντιφώντα αλλά την βελτίωσε ακόμη περισσότερο. Σύμφωνα με τον σχολιαστή του Αριστοτέλη Αλέ­ ξανδρο τον Αφροδισιέα, ο Βρύσων θεώρησε αρχικά τρία τετράγωνα · το ένα εγγεγραμμένο στον κύκλο, το δεύτερο περιγεγραμμένο στον ίδιο κύκλο και το τρίτο ανάμεσά τους. Η ιδέα του ήταν να διπλασιάζει συνεχώς τις πλευρές τόσο των εγγεγραμμένων όσο και των περιγεγραμμένων τετραγώνων, όπως ακρι­ βώς έκανε και ο Αντιφών με τα εγγεγραμμένα κανο­ νικά πολύγωνα.

ΕΥΚΛΕΙΔΗΣ Β ' τ. Ι /9

Συνάδenqιοι


-------

Ιστορικές Μαθηματικές Αναφορέ·ι-----

Πίστευε ότι, αν αυτή η διαδικασία συνεχιστεί, τότε θα έχουμε ένα εγγεγραμμένο και ένα περιγε­ γραμμένο πολύγωνο που θα διαφέρουν τόσο λίγο ως προς το εμβαδόν τους, ώστε αν μπορεί να οριστεί ένα πολύγωνο με εμβαδόν ίσο με το ημιάθροισμα των δύο ω.λων, τότε ο κύκλος, που έχει κι αυτός εμβαδόν ανάμεσα στο εγγεγραμμένο και στο περιγεγραμμένο πολύγωνο, θα είναι ισοδύναμος μ' αυτό το ενδιάμεσο πολύγωνο. Η ιδέα του Βρύσωνα βεβαίως είναι λανθασμένη, ωστόσο, αποτελεί μια σαφή βελτίωση της μεθόδου του Αντιφώντα. Το σημαντικότερο όμως είναι ότι αυτή η βελτιωμένη μέθοδος των Αντιφώντα - Βρύ­ σωνα αποτελεί την πρώτη, την αρχική μορφή της "μεθόδου της εξάντλησης", η πατρότητα της οποίας ως γνωστόν αποδίδεται στον Εύδοξο τον Κνίδιο και στον Αρχιμήδη. Η τρίτη παρατήρησή μας είναι ότι περιέργως ελά­ χιστες είναι οι γνώσεις μας ακόμη και για τους γεωμέ­ τρες που περιλαμβάνει ο Πρόκλος στη Σύνοψή του. Για τον Αναξαγόρα τον Κλαζομένιο γνωρίζουμε απλώς ότι ασχολήθηκε με το πρόβλημα του τετρα­ γωνισμού του κύκλου, όταν βρέθηκε κάποτε στη φυλακή στην Αθήνα γιατί δίδασκε ότι ο ήλιος και οι aστέρες ήταν νεκροί πύρινοι λίθοι. Στον Οινοπίδη το Χίο, ο Πρόκλος αποδίδει δύο γεωμετρικές προτάσεις, οι οποίες περιέχονται στα "Στοι χεία" του Ευκλείδη. Πρόκειται για την πρόταση 1 .12, η οποία αναφέρεται στη χάραξη μιας ευθείας κάθετης προς δοσμένη ευθεία από σημείο εκτός αυ­ τής, και στην πρόταση 1 .23, η οποία αναφέρεται στην κατασκευή γωνίας ίσης με δοσμένη γωνία. Για τον Ιπποκράτη το Χίο, τον σημαντικότερο από τους επτά γεωμέτρες της Σύνοψης, ο Πρόκλος αναφέρει ότι ήταν ο πρώτος που συνέγραψε βιβλίο Γεωμετρίας, το οποίο όμως δεν σώθηκε. Ο Ιπποκρά­ της όμως είναι περισσότερο γνωστός για δύο πάρα πολύ σημαντικά μαθηματικά επιτεύγματά του. Το πρώτο είναι ο τετραγωνισμός ορισμένων μηνίσκων που σήμερα φέρουν το όνομά του, και το άλλο είναι η αναγωγή του προβλήματος του διπλασιασμού του κύβου στην εύρεση δύο μέσων αναλόγων. Οι λιγοστές πληροφορίες μας για τον Θεόδωρο τον Κυρηναίο προέρχονται κυρίως από τον πλατωνικό

διάλαyο Θεαίτητος. Εκεί αναφέρεται ότι ο Θεόδωρος απέδειξε πρώτος την ασυμμετρία των J3 , ..J5, . . . .Jl7 . Ακόμη πιο λιγοστές είναι οι πληροφορίες μας για τον Λεδάμαντα τον Θάσιο. Γνωρίζουμε μόνο ότι ήταν μαθητής του Πλάτωνα και ότι διδάχτηκε από τον δάσκαλό του την αναλυτική μέθοδο στη Γεωμετρία (Διογένη Λαέρτιος 111. 24). Ο Heath όμως πιστεύει ότι η πληροφορία του Διογένη είναι λάθος και οφείλεται σε παρανόηση. Κατά τη γνώμη του Πλάτων δεν επι­ νόησε την αναλυτική μέθοδο. Ο Αρχύτας ο Ταραντίνος υπήρξε μια πραγματι­ κά πολυδιάστατη προσωπικότητα. Αναφέρεται ως πολιτικός, φιλόσοφος, στρατιωτικός, αλλά επίσης και ως μηχανικός, εφευρέτης, aστρονόμος, μουσικός και γεωμέτρης. Το σίγουρο πάντως είναι ότι αν και προ­ σωπικός φίλος του Πλάτωνα δεν συμμεριζόταν κα­ θόλου τις ιδεολογικές προκαταλήψεις του φίλου του. Αυτό αποδεικνύεται από τις μαθηματικές και άλλες ενασχολήσεις του Αρχύτα. Πρώτα απ' όλα ο Αρχύτας πέτυχε να λύσει το Δήλιο πρόβλημα με μια ευφυέστατη στερεομετρική κατασκευή. Επινόησε αρκετά θεωρήματα και προτά­ σεις πάνω στην πυθαγόρεια θεωρία της μουσικής και στη θεωρία των λόγων. Το κυριότερο όμως είναι ότι ο Αρχύτας, σύμφωνα με τον Διογένη Λαέρτιο (VIII 79-83), ήταν ο πρώτος που εισήγαγε τη χρήση των μαθηματικών στη μελέτη της μηχανικής. Αναφέρεται ότι σχεδίασε και ο ίδιος μερικές θαυμαστές μηχανές, όπως για παράδειγμα ένα ξύλινο περιστέρι που είχε τη δυνατότητα να πετά. τέλος, στον Αρχύτα αποδί­ δεται και η ευφυής προσεγγιστική μέθοδος για τον υπολογισμό των τετραγωνικών ριζών. Ο Θεαίτη τος ο Αθη ναίος γνωρίζουμε ότι ασχο­ λήθηκε με δύο κυρίως θέματα: με τη θεωρία των αρρήτων και με τα πέντε κανονικά στερεά. Από τον πλατωνικό διάλογο που φέρει το όνομά του, μαθαί­ νουμε ότι ο Θεαίτητος απέδειξε ότι κάθε αριθμός της μορφής � είναι aσύμμετρος, όταν ο κ δεν είναι τετράγωνο ακεραίου αριθμού. Το όνομα του Θεαίτη­ του συνδέεται επίσης και με την γεωμετρική κατα­ σκευή των κανονικών στερεών, ειδικά του οκταέ­ δρου και του εικοσαέδρου. Ο Heath είναι βέβαιος ότι όλη η σχετική θεωρία των "Στοιχείων" οφείλεται στον Θεαίτητο.

ΕΥΚΛΕΙΔΗΣ Β' τ.Ι/10


π

I

Η στήλη αυτή έχει ως στό χο την ανάπτυξη μα θηματικού διαλόγου. Φιλο δοξούμε να συμμετάσχουν όλοι

όσοι έχουν ένα γενικότερο ενδιαφέρον για τα Μαθηματικά.

Επιμέλεια: Γιάννης Στρατή ς - Βαγγέλης Ευσταθίου

Στο τεύχος 53 του περιοδικού μας (Ιούλι­ ος - Αύγουστος - Σεπτέμβριος 2004) στο κομμάτι που αναφέρεται στη Α! τάξη του Λυκείου είχαμε δημοσιεύσει ένα άρθρο με τίτλο: «Για να κατα­ νοήσουμε καλύτερα τους αριθμούς» (Γιάννης Στρατής). Ήταν μια σειρά θέματα πάνω στις ταυ­ τότητες, τις εξισώσεις α! βαθμού, τις μόνιμες ανι­ σότητες (και με περιορισμό), τις ανισώσεις α! βαθμού. Το άρθρο συμπληρωνόταν με μερικά προβλήματα (συνολικά 5 · δύο στις εξισώσεις, τρία στις ανισότητες) . Στη σελίδα 19 είχαμε δημοσιεύσει το παρα­ κάτω ζήτημα: Αφού αποδειχθεί η ταυτότητα: ( α2 + β2 + γ2 )( χ2 + / + z2 ) - ( αχ + βy + γz )2 =

Ο συνάδελφος Παναγιώτης Οικονομάκος μας απέστειλε με γράμμα του (Γενάρης 2005) την πα­ ρακάτω παρατήρηση: «Το πρόβλημα 3 της σελίδας 1 9 (περιόδου 1ουλ. - Αυγ. - Σεπτ. 2004) έχει λανθασμένη υπόθε­ ση, διότι αν Ρ μεταβλητό σημείο στο εσωτερικό τυχαίου τριγώνου ΑΒΓ και χ, y, z είναι οι αποστά­ σεις του από τις πλευρές του, τότε το άθροισμα χ + y + z δ εν είναι σταθερό (Υπογράμμιση δική μας). Παρακάτω δείχνουμε με δύο τρόπους ότι η πα­ ρατήρηση του συναδέλφου δεν ισχύει. Ακριβέστε­ ρα υπάρχουν άπειρα σημεία Ρ στο εσωτερικό ενός σκαληνού τριγώνου, που το άθροισμα: χ + y + z των αποστάσεών του απ' τις αντίστοιχες πλευρές του τριγώνου είναι ένας σταθερός αριθμός c, ο οποίος διατρέχει ένα συγκεκριμένο διάστημα.

= ( βχ - αy )2 + ( γy - βz )2 + ( γχ - αz )2 , Ένας Δεύτερος Τρόπος να βρεθεί στο εσωτερικό τριγώνου ΑΒΓ σημείο Ρ έτσι ώστε: «αν χ, y, z είναι οι αποστάσεις του Ρ 2"ς τρόπος (Γιάννης Δ. Στρατής) απ' τις πλευρές του τριγώνου και χ + y + z = c Χωρίς βλάβη της γενικότητας δεχόμαστε ότι: όπου c κάποιος θετικός αριθμός σταθερός ( κατάλ­ α :::; β :::; γ . Αν ρ η ακτίνα του εγγεγραμμένου κύληλος) το άθροισμα χ2 + y2 + z2 να είναι το μι- κλου, τότε ισχύουν οι συνθήκες: 2Ε < α + 2β ρ < ---2α + β ρ < -2Ε κρότερο δυνατό» (το 'χε προτείνει ο συνάδελφος (l) -α α γ β Θανάσης Κυριακόπουλος, μέλος της Συντακτικής Επιτροπής). Πραγματικά: Πρέπει να παραδεχθούμε κατ' αρχάς ότι η λέξη 2Ε < α + 2β ρ � α + β + γ ρ < α + 2β ρ � -«κατάλληλος» δεν υπήρχε στο κείμενο που είχαμε γ γ β β δημοσιεύσει. Απ' την όλη όμως διαπραγμάτευση του � αβ + β2 + βγ < αγ + 2βγ ζητήματος γινόταν φανερό ότι ο c είναι μεν σταθε­ � αβ + β2 < αγ + βγ � ( α + β ) β < ( α + β ) γ , ρός, αλλά εννοείται ότι είναι προσδιοριστέος. που είναι αληθής. ----

ΕΥΚΛΕΙΔΗΣ Β ' τ. Ι/11

----

----


Το Βή μα του Ευκλείδη

2α + β � α2 + 2αβ < 2αβ + β2 � α + 2β < --ρ Αν ισχύουν οι συνθήκες (3) ο c δύο δυνατότη­ --ρ α β τες έχει: 2Ε Οποτε: α + 2β < c < -. 2 , κ = 2Ε - βc . 2 � α < β , που είναι αληθής. (ι):. --ρ β γ-β β 2Ε 2α + β < α + β + γ ρ � 2α + β < -�--ρ --ρ , α α α α Κι επομενως: κ < 2Ε - αc (7). � 2α + β < α + β + γ � α < γ , που είναι αληγ-α 2Ε - βc < 2Ε - αc � θής. Αφού: 2Ε ως προς τις γ-β γ-α , τωρα , τη θ'εση του β Αναζητουμε �2Ε( γ -α) - β( γ-α)c<2Ε{ γ -β) -α( γ -β)c � ανισοτικές σχέσεις ( Ι ) �2Ε[( γ -α) -( γ -β)] <c[-{l( γ -β) +β( γ -α)J � 2Ε α + 2β � 2Ε < γc , που είναι αληθής. Κατ' αρχάς έχουμε: --ρ < - . β β Αφού: 2Ε ::ςc < --ρ 2α + β . 0ποτε: .. ) , κ = 0 . Κι επομε-, ( ιι: α--ρ + 2β < α + β + γ ρ � α + 2β < α + β + γ � α β β β νως: ισχύει η (7). � β < γ , που είναι αληθής. 2Ε Αφού: 2Ε αc > Ο � c > - , που είναι αλη­ Έχουμε την ισοδυναμία: α α 2Ε 2α 2α + + β + + β β γ θής. ρ < --ρ - < --ρ � α α β β 2Ε . < c τότε: (4) συνθήκες Αν οι υν ο ισχύ 2 2 α + αβ β . � α2 + αβ + αγ < 2αβ + β2 � γ < β α ο , , , ισχυει η (7). ποτε: κ = 2Ε - βc . Κι επομενως: Συνεπώς διακρίνουμε τις περιπτώσεις: γ-β β2 -._ α2 + αβ Δηλαδή σε κάθε περίπτωση ισχύει: (Ι): γ < .:... α , ορι'ζεται το ανοικτο, κ < 2Ε - αc και συνεπως 2 - α2 + αβ β γ-α (Π): γ 2 . α διάστημα: κ, 2Ε - αc . Στην πρώτη περίπτωση ισχύει: γ-α 2Ε < α + 2β ρ < 2Ε < 2α + β ρ < 2Ε (3) Στη συνέχεια αποδεικνύουμε ότι: «για κάθε β β α α γ z ε κ, 2Ε -- ααc το σύστημα: Στη δεύτερη περίπτωση ισχύει: γ 2Ε 2Ε 2Ε < α + 2β ρ < 2α + β ρ::ς < x+y =c- z (4) έχει μοναδική θετική λύση» β α β α γ αχ +βy =2Ε-γz Εκλέγουμε τώρα ένα σταθερό c απ' το ανοικτό Α πό δε ι ξη διάστημα: Η ορίζουσα D του συστήματος είναι: 2β ρ,2α -ρ + β κι ας ειναι α-+, D = β - α > Ο . Και συνεπώς το σύστημα είναι α β Cramer 2 2 . Βρίσκουμε τώρα τις ορίζουσες Οχ και DY του συστήματος. αν : c 2 2Ε Έχουμε: Dx = βc - 2Ε + (γ - β)z , β (5) κ- 2Ε 2Ε βc , DY = 2Ε - αc - (γ - α )z αν : c < β γ-β D χ = βc - 2Ε + (γ - β)z , και συνεπώς: χ =_ D β-α Οπότε: κ 2 Ο και κ 2 2Ε - βc . γ-β •

..:....

__

)

[

{

[

!ο,

)

) }

[

χ

-

ΕΥΚΛΕΙΔΗΣ Β ' τ. Ι/12


Το Βή μα του Ευκλείδη

= 2Ε - αc - ( γ - α ) z (8) D β-α Έχουμε τώρα, λόγω της (6), 2Ε____. - βc_.:. < z < 2Ε - αc . γ-β γ-α Οπότε: Ο < βc - 2Ε + ( γ - β ) z , 2Ε - αc - ( γ - α ) z > Ο . Δηλαδή: χ > Ο , y > O . 2Ε Προφανώς ισχύει: Ο < z < ­ (9) γ 2Ε διότι: Αφου:, z > κ � Ο και z < 2Ε - αc < γ-α γ ( 2Ε - αc ) γ < ( γ - α ) 2Ε � -αcγ < -α2Ε ή 2Ε , που ειναι . αληθής. c>γ Εύκολα διαπιστώνεται ακόμη ότι: χ < hα, y < hβ (10) y=

Dy

_

Συνεπώς το σύνολο των σημείων Ρ, των οποίων οι αποστάσεις χ, y, z απ' τις πλευρές ΒΓ, ΓΑ, ΑΒ είναι αντίστοιχα χ, y, z κι έχουν την ιδιότητα χ + y + z = c · όπου c μια σταθερή του ανοικτού διαστήματος: α +_ 2β _ 2α _ + β είναι ισοδύναμο με το ανοικτό _ β ' α διάστημα: κ, 2Εγ -- ααc δηλαδή έχει τη δύναμη του συνεχούς.

(

(

)

)

Σ' ένα δεύτερο επίπεδο αποδεικνύουμε ότι: «το Ρ

γράφει το εσωτερικό r.νός συγκεκριμένου ευ­

θυγράμμου τμήματος»

Πραγματικά: Αν απαλείψουμε το z απ' τις εξι­ σώσεις (8) φτάνουμε στην εξίσωση: ( γ - α ) χ + ( γ - β ) y = cγ - 2Ε (ΙΙ) γ

;

2 βημΓ =

Α(βσυνΓ, βημΓ)

______

θ=90'-Γ Χ = γc-2Ε (γ- α)εφΓ = γc-2Ε γ γ α Σ( γc- 2Ε ' Ο) (γ- α)ημΓ 1

I

Ας είναι τώρα ( χ, y ) μια κάποια λύση του συ­ στήματος. Φέρουμε την ευθεία ε1 παράλληλη προς τη ΒΓ στο ημιεπίπεδο που βρίσκεται το Α και την ε2 παράλληλη προς την ΑΓ στο ημιεπίπεδο που βρί­ σκεται το Β. Προφανώς τέμνονται σ' ένα σημείο Ρ στο εσωτερικό του τριγώνου. Αποδεικνύουμε ότι: ( ΡΜ ) = z . Πραγματικά: απ' τη δεύτερη εξίσωση του συ, . Ι στηματος εχουμε: -αχ + Ι-βy = Ε - Ι-γz ή ισοδύ2 2 2 ναμα.. Ε ( BPr) + E(ri>A) = Ε( ArB) -Ι2 γz · Και συνεπώς: 1 η,Ι-γ ΡΜ )Ι = -γz . Ε( ΑΡΒ) = -γz 2( 2 2 Κι επομένως: z =( ΡΜ ) . "

χ·� ����� Γ

-

����

--

γ·

Ας πάρουμε την περίπτωση Γ < 90° [όμοια κα­ τά βάση εργαζόμαστε στην περίπτωση: Γ > 90° . Η περίπτωση Γ = 90° είναι πιο απλή]. Φέρνουμε την κάθετη της ΓΒ στο Γ και παίρ­ νουμε τη διεύθυνση ΓΒ ως τη θετική διεύθυνση Χ 'Χ, ενώ η κάθετη της ΓΒ στο Γ προς το μέρος του Α λαμβάνεται ως θετική διεύθυνση του άξονα ΥΥ. Αναζητούμε τις σχέσεις που συνδέουν τα χ, y με τις συντεταγμένες Χ = ( ΓΚ ) , Υ = ( ΓΗ ) του σημείου Ρ. Επειδή τα μέτρα των γωνιών <rHPΛ και <rΗΓΛ είναι ίσα [αφού έχουν τις πλευρές τους κάθετες] συμπεραίνουμε ότι: χ = ( ΓΗ ) = Υ και y = ( ΘΡ ) συνθ = ( ΘΡ ) ημΓ .

ΕΥΚΛ ΕΙΔΗΣ Β ' τ.l/13


Τ ο Βή μα του Ευκλείδη

θεία που διέρχεται απ' το Ν της ΓΑ κι έχει συ­ Αλλά: ντελεστή διεύθυνσης λε με ( ΘΡ ) = ( ΗΡ ) - ( ΗΘ ) = Χ - ( ΓΗ ) εφθ = Χ - ΥσφΓ . Άρα: χ = Υ και y = ΧημΓ - ΥσυνΓ (12) λε = - ( γ - β)ημΓ ( 1 8) γ συνΓ α γ β) ) ( ( Έτσι η ( 1 1) γίνεται: που είναι αρνητικός αριθμός [αφού: [ ( γ - β) ημΓJ Χ + [( γ - α ) - ( γ - β)συνΓ] Υ = ( γ - β)ημΓ > Ο και γ - α > γ -β 2:: ( γ - β)συνΓ ] . = cγ - 2Ε , (1 3), Ο συντελεστής διεύθυνσης της ΑΒ είναι: που είναι εξίσωση κάποιας ευθείας ε. Ο - βημΓ ( 19), που είναι αρνητικός α­ λ ΑΒ α - βσυνΓ Αναζητούμε τώρα το σημείο Ν όπου τέμνονται οι ε και ΓΑ (εφ' όσον φυσικά υπάρχει). Κι επειδή ριθμός [αφού: η εξίσωση της ΓΑ είναι Υ = ΧεφΓ (14), η (13), λόγω της ( 14 ) γίνεται: Χ = cγ - 2Ε και συνε( γ - α ) εφΓ Ισχυριζόμαστε ότι: πώς Υ = cγ - 2Ε . Δηλαδή η ε τέμνει τη ΓΑ στο λε > λΑΒ (20) γ-α Πραγματικά: Λόγω των ( 1 8) και ( 19) έχουμε: σημείο ( γ - β)ημΓ βημΓ Ν cγ - 2Ε ' cγ - 2E ( 1 5) λε - λΑΒ = α - βσυνΓ ( γ - α ) - ( γ - β)συνΓ = ( γ - α ) εφΓ γ - α γ ( β - α ) ημΓ Για να δείξουμε ότι το Ν είναι εσωτερικό ση­ = >Ο ( α - βσυνΓ) [( γ - α ) - ( γ - β)συνΓJ μείο του ευθύγραμμου τμήματος ΓΑ δείχνουμε τις ανισότητες: Συνεπώς οι ευθείες ε και ΑΒ τέμνονται · και 0 < cγ - 2Ε < βσυνΓ (16) μάλιστα το σημείο τομής τους - ας πούμε Τ - δε ( γ - α ) εφΓ μπορεί να είναι στην προέκταση της ΑΒ προς την πλευρά του Α. Άρα το Τ ή θα ανήκει στο ευθύ­ και γραμμο τμήμα ΑΒ, συμπεριλαμβανομένου του Β ή ο cγ - 2Ε 2Ε < (17) θα ανήκει στο εσωτερικό του ΒΓ, οπότε το ονομά­ <γ-α α ζουμε Σ. Τελικά ο γεωμετρικός τόπος του Ρ είναι Για τη (16): το πρώτο σκέλος είναι προφανές το εσωτερικό ενός ευθύγραμμου τμήματος ΝΤ ή [αφού έχουμε δεχθεί: Γ < 90° ] το δεύτερο σκέ­ ενός ευθύγραμμου τμήματος ΝΣ. λος είναι ισοδύναμο με τη σχέση: Οπωσδήποτε υπάρχει μια ασάφεια στον προσ­ cγ - 2Ε < β ( γ - α ) ημΓ . διορισμό του γεωμετρικού τόπου. Πότε το Τ είναι Η τελευταία είναι ισοδύναμη με τη συνθήκη: στο ΑΒ και πότε στο ΒΓ. Η ασάφεια αυτή αίρεται cγ - ;Ε < βγημΓ - � με την παρακάτω διαδικασία. Για Υ = Ο , η (13) γίνεται: αβημΓ = 2Ε , που <=> cγ < βγημΓ <=> c < βημΓ = α α γ - β)ημΓ] Χ = cγ - 2Ε ή Χ = cγ - 2Ε . [( είναι αληθής. ( γ - β ) ημΓ Για τη ( 1 7): το πρώτο σκέλος είναι προφανές. Δηλαδή η ε τέμνει την ΑΒ στο σημείο Το δεύτερο είναι ισοδύναμο με τη συνθήκη: αγc - 2αΕ < 2γΕ - 2αΕ <:::::> αγc < 2γΕ 2Ε , cγ - 2Ε o . ' <:::::> c < - , που ειναι αλη θης. Q α ( γ - β)ημΓ ' Η ε λοιπόν μπορεί να περιγραφεί ότι είναι η ευ__

,

(

J

(

ΕΥΚΛΕΙΔΗΣ Β' τ.Ι/14

J


Το Βή μα του Ευκλείδη

Έχουμε τώρα: cγ - 2Ε � α <=> cγ - 2Ε � αγη μΓ - αβημΓ ( γ - β ) ημΓ <=> cγ 2: αγημΓ [αφού 2Ε = αβημΓ ] 2Ε . <=> c � αημΓ = β Έτσι διαμορφώνουμε το παρακάτω κριτήριο. 2Ε , τοτε ' το Q ειναι ' το Β η' δεξια' του Β . Αν c � β Οπότε η ευθεία ε συναντάει το ευθύγραμμο τμήμα ΑΒ σ' ένα σημείο τ. 2Ε , τοτε , το Q ειναι , , καποιο Αν c < σημειο, Σ β του ΓΒ. Αντίστροφα : Αν ένα σημείο Ρ ανήκει στο εσω­ τερικό ενός ευθυγράμμου τμήματος ΝΤ ή ΝΣ και είναι χ, y, z οι αποστάσεις του απ' τις πλευ­ ρές του τριγώνου, θα αποδείξουμε ότι x+y+z=c. Π ραγματικά :

Ας είναι Χ, Υ οι συντεταγμένες

του Ρ. Τότε: [( γ - β)ημΓ] Χ + [(γ - α ) - ( γ - β)συνΓ] Υ = = cγ - 2Ε <=> (1 ) ( γ - α )Υ + ( γ - β) ( ΧημΓ - ΥσυνΓ ] = cγ - 2Ε <=>2 <=> (γ - α ) χ + (γ - β) y = cγ - 2Ε <=> γ( χ + y ) - ( αχ + βy ) = cγ - 2Ε (21) Κι επειδή: 2Ε = αχ + βy + γz

[ Ε( . ) = Ε( . ) + Ε( . ) + Ε( . ) ] ΑΒΓ

ΑΡΒ

ΒΡΓ

ΓΡΑ

συμπεραίνουμε απ' την (21) ότι: γ( χ + y) = cγ - γz ή x + y = c - z δηλαδή: x + y + z = c . Σχετικά με το ακρότατο της παράστασης : χ2

+ y2 + z2 Εύκολα αποδεικνύονται οι συνθήκες:

α + 2. -ρ β < 3ρ < --ρ 2α + β και -α β 2Ε - αc (11): κ < ρ < --γ-α Η (I) είναι ισοδύναμη με τη συνθήκη: α < β [που ισχύει από υπόθεση] Για το πρώτο σκέλος της (11) διακρίνουμε τις περιπτώσεις 2Ε , τοτε , κ=Ο<ρ. ..,. Αν c � β 2Ε , τοτε , κ = 2Ε - βc < ρ <=> ..,. Αν c < β γ-β <=> ( α + β + γ) ρ - βc < (γ - β) ρ <=> c > α + 2β ρ [που είναι αληθής λόγω της (2)] β Για το δεύτερο σκέλος της (11) διαπιστώνουμε , η συνθηκη: , ρ < 2Ε - αc ειναι , ισοδυναμη , οτι με γ-α 2α +. -ρ β [που ειναι , αλη θης, , c < -τη συνθηκη: α λόγω της (2)] . Έτσι οι (8) για c = 3ρ γίνονται: + ��(γ - β) z ( 2β - α - γ) ρχ = �--� (22) β-α - (γ - α )z(β + γ - 2α�)ρ�� y = �-β-α Θεωρούμε τώρα τη συνάρτηση (23) f ( z) = χ 2 + y2 + z2 Οπότε: dx dy f ' ( z ) = 2x · - + 2y · - + 2 z = dz dz (1):

{ [ _ γ - α [ (β + γ - 2α)ρ β-α β-α

]

= 2 γ - β ( 2β - α - γ) ρ + (γ - β) z β-α β-α β-α

{

_

] }

(γ - α ) z + z = β-α

=2 � [( γ -β)(2β -α �γ) -(γ -α)( β+γ-2α)] + ( β-α) ΕΥΚΛΕΙΔΗΣ Β' τ. Ι/15


Τ ο Βή μα του Ευκλείδη

' το σημειο' νεται όταν: -χ = -Υ = -z , δηλαδη' οταν α β γ Ρ συμπίπτει με το σημείο από το οποίο περνούν οι τρεις συμμετροδιάμεσοι του τριγώνου» 1

Σχόλιο τ η ς Σ . Ε .

Το ελάχιστο που βρίσκει ο συνάδελφος είναι ολικό. Το πρόβλημα 3 που είχαμε θέσει ζητούσε το ελάχιστο της παράστασης χ2 + / + z2 με τον περιορισμό χ + y + z = c όπου c μια κατάλληλη σταθερή. Το ζήτημα μοιάζει με το παρακάτω παράδειγ­ μα:

Κι επομένως:

( ( z) = Ο <=:> z = ρ

και r ' (z) > O <=:> z > ρ . παρουσιάζει (ολικό) ελάχιστο για

Άρα η f z=ρ. Αντικαθιστώντας την τιμή του z = ρ στις (22) παίρνουμε: χ = y = ρ . Ο συνάδελφος 11 . Οικονο μάκος στο γράμμα του, που προαναφέραμε, γράφει: «Στο τυχαίο τρίγωνο ΑΒΓ ακυρώνουμε τη λανθασμένη και περιττή υπόθεση:χ + y + z = c, αφού μας αρκεί ότι ισχύει πάντα αχ + βy + γz = 2Ε (Ι) όπου Ε το εμβαδό του τριγώνου ΑΒΓ. Καθώς γνωρίζουμε ισχύει η ταυτότητα Lagrange: (χ2 + y2 + z2 )(α2 + β2 + γ2 ) - (αχ + βy + γz)2 = = (αy - βχ)2 + (αz - γχ)2 + (βz - γy)2 (2) Οπότε, λόγω της (Ι), έχουμε: (χ2 + y2 + z2 )(α2 + β2 + γ2 ) - 4Ε2 = = (αy - βχ)2 + (αz - γχ)2 + (βz - γy)2 ::::: 0 Και συνεπώς

f(ξ1) fmax - - - - - - - - -- Ρ =

Α ι

I I I I

ι

Β

: f(x.,) - � - - Ρ0 :L Ρ2 � f(ξ2) fmin ---+-α��Χ�ο---ξ�--ξ� 2 ---βL_ =

_

I J___ι_____ I I I

_ _ _

I

- - - - -

I

____

Η συνάρτηση του σχήματος έχει την ιδιότητα: f { ξ2 ) � f ( χ ) � f ( ξ 1 ) , για κάθε χ Ε [ α, β ] . Δηλαδή: f{ ξ2 ) = min { f { χ ) Ι χ Ε [ α, β ] } Ενώ f { x0 ) = mi n { f { χ ) ! χ Ε [ α,ξ1 ] } Με άλλα λόγια η τιμή f(x0) είναι τοπικό ελάχι­ στο δηλαδή είναι η ελάχιστη τιμή όταν η ανε­ ξάρτητη μεταβλητή διατρέχει το διάστημα [α,ξ, ] .

1

Το σημείο αυτό αναφέρεται στη βιβλιογραφία ως σημείο Lemoine. Για την έννοια της συμμετροδιαμέσου και τα

σχετικά θεωρήματα μπορεί ο αναγνώστης να ανατρέξει στα •

παρακάτω συγγράμματα: Νικ. Α. Κισκίρα «Θεωρήματα και Προβλήματα Γεωμετρί­ ας» Επιπεδομετρία βιβλίον τέταρτον -βιβλίον πέμπτο. Α­

Άρα ελαχίστη τιμή για το άθροισμα χ2 + y2 + z2 είναι: 2 4Ε22 2 και επιτυγχάα +β +γ

θήναι I 973 . Σελ. 93 και πέρα. Αριστείδου Φ. Πάλλα «Μεγάλη Γεωμετρία» Τόμος Α ' Επιπεδομετρία τεύχος Α ' πρώτον και δεύτερον βιβλίον.

Έκδοσις Τρίτη Αθήνα \ 962 σελ. 227 και πέρα. Γιάννη Ντάνη «Γεωμετρία», Αθήνα 1 97 1 σελ. 254 Νείλου Σακελαρίου «Θεωρητική Γεωμετρία» Τόμος Β ' σελ. \ 85

ΕΥΚΛΕΙΔΗΣ Β' τ.l/1 6


Τ ο Βή μα του Ευκλείδη

& Το Σωστό με τους

σχέση Π ρ οτασιακούς τύπους και το

Λάθος

σε

Τ ου Γιώργου Τ ασσόπουλου

Τα τελευταία χρόνια στις εξετάσεις έχει καθιε­ ρωθεί να δίνονται και θέματα στα οποία οι μαθη­ τές καλούνται να απαντήσουν αν μια πρόταση εί­ ναι Σωστή (Αληθής) ή Λάθος (Ψευδής). Όμως τις περισσότερες φορές οι εξεταστές α­ κόμη και στις Πανελλαδικές Εξετάσεις αντί να αναφερθούν σε προτάσεις, αναφέρονται σε προτα­ σιακούς τύπους μιας ή περισσοτέρων μεταβλητών με αποτέλεσμα, όπως θα δούμε ,παρακάτω να μην έχει καν νόημα η ερώτηση. Ως γνωστόν λέγοντας μεταβλητή εννοούμε ένα σύμβολο π.χ. ένα γράμμα χ που χρησιμοποιούμε για να συμβολίσουμε ένα οποιοδήποτε στοιχείο ενός συνόλου Ω το οποίο έχει τουλάχιστον δυο * στοιχεία . Λέμε τότε ότι το χ διατρέχει το σύνολο Ω. Σε αντιδιαστολή κάθε στοιχείο του Ω λέμε ότι είναι μια σταθερά. Προτασιακό ς τύπο ς μιας μεταβλητής χ λέγε­ ται κάθε έκφραση που περιέχει τη μεταβλητή αυτή και γίνεται πρόταση όταν η μεταβλητή αντικατα­ σταθεί με ένα οποιοδήποτε στοιχείο του συνόλου Ω;t:0 το οποίο διατρέχει η μεταβλητή χ. Συμβολί­ ζεται δε με ρ( χ). Το σύνολο των στοιχείων ξ του Ω για τα οποία η p(ξ) είναι αληθής πρόταση λέγεται σύνολο αλήθειας του προτασιακού τύπου p(x) και συμβολίζεται με Α= {χεΩ/p(χ)}, λέμε δε ότι για κάθε χεΑ, ο προτασιακός τύπος p(x) ισχύει, επα­ ληθεύεται. (Σχετικά με προτάσεις και Προτασιακούς τύ­ πους βλέπε τα άρθρα περί Μαθηματικής Λογικής του συναδέλφου Αντώνη Κυριακόπουλου στα τεύ­ χη 57 έως 60). Για παράδειγμα αν η μεταβλητή χ διατρέχει το σύνολο Ω= { 1 ,2,3} τότε οι παρακάτω εκφράσεις Ρ1(χ): Το χ είναι άρτιος αριθμός Ρ2(χ): χ+ 1>2 Ρ3 (χ): χ+ 12::2 Ρ4(χ): χ+2<3 είναι προτασιακοί τύποι με σύνολα αλήθειας Α ι ={2} , Α2={2,3}, Α3= { 1 ,2,3 }=Ω, Α4=0 Κάθε προτασιακός τύπος λοιπόν μπορεί να γίνει αληθής πρόταση στις εξής περιπτώσεις: 1 . Για κάποιες τιμές της μεταβλητής χεΩ 2. Για όλες τις τιμές της μεταβλητής χεΩ Αν το Ω είναι μονοσύνολο τότε συμβ ατικά το ρ θεω είται μεταβλητή εκφυλισμένη σε σταθερά •

χ

3.

Για καμιά τιμή της μεταβλητής χε Ω

Για τις τρεις αυτές περιπτώσεις χρησιμοποιού­ με τις ακόλουθες τρεις εκφράσεις αντιστοίχως, που χαρακτηρίζονται ως αληθείς προτάσεις. 1 . Υπάρχει χ ε Ω ώστε να ισχύει p(x), Συμβολικά ΞJ χεΩ, p(x) 2. Για κάθε χ ε Ω ισχύει p(x), Συμβολικά V χεΩ, p(x) 3. Δεν υπάρχει χ ε Ω ώστε να ισχύει ρ( χ), Συμβολικά 3 χεΩ, p(x), ή 3χ ε Ω, p (x) Δεν είναι λοιπόν ορθό να ζητήσουμε από τους μαθητές να μας χαρακτηρίσουν κάποια από τις εκφράσεις Ρ1(χ), Ρ2(χ), Ρ3 (χ), Ρ4(χ) ως αληθή ή ψευδή, διότι αυτές δεν είναι προτάσεις αλλά προ­ τασιακοί τύποι. Μπορούν να γίνουν προτάσεις μό­ νον όταν το χ αντικατασταθεί με κάποιο στοιχείο του Ω= { 1 ,2,3 }, ή αν προτάξουμε μια από τις εκ­ φράσεις υπάρχει, για κάθε, δεν υπάρχει, όπως φαίνεται παρακάτω. Η πρόταση: Για κάθε χεΩ ισχύει χ+ Ι>2 είναι ψευδής, αφού για χ= 1 δεν ισχύει. Η πρόταση: Για κάθε χε Ω, ισχύει χ+ 12::2 είναι αληθής Η πρόταση: Υπάρχει χεΩ ώστε να ισχύει χ+2<3 είναι ψευδής Η πρόταση: Δεν υπάρχει χ ε Ω, ώστε να ισχύει χ+ Ι>3 είναι ψευδής -

Π αρ α δ ε ίγ μ ατα Αν χ JR , τότε

ε να χαρακτηρίσετε ως αληθή ή ψευ δή την ισότητα ημ2χ=l+συν2 χ

Η έκφραση αυτή δεν είναι πρόταση αλλά προ­ τασιακός τύπος αφού εξαρτάται από τη μεταβλητή χ, άρα δεν μπορεί να χαρακτηριστεί ως αληθής ή ψευδής. Αν όμως γράφαμε: Για κάθε χ ε JR , ισχύει ημ2χ= Ι +συν2χ, τότε θα ήταν μια ψευδής πρόταση αφού για χ=Ο δεν ισχύει, διότι 0\t: Ι + Ι 2 Αν γράφαμε: Υπάρχει χ ε JR , ώστε να ισχύει ημ2χ= Ι +συν2χ, τότε θα ήταν μια αλη θής πρόταση αφού για χ = π ισχύει, διότι Ι 2= Ι +02 . Μάλιστα 2

' για κα' θε χ = κπ + -π ενω' για ισχυει 2 δεν ισχύει, όπου κ ε Ζ .

ΕΥΚΛΕΙΔΗΣ Β ' τ.l/17

χ

π :1:- κπ + -

2


Τ ο Βή μα του Ε υκλείδη

Αν ΑΒΓ ορ θ ογώνιο τρίγωνο με υποτείνου σ α ΒΓ και ύ ψ ος ΑΔ, τότε να χα ρ ακτηρίσετε ως αλη θή ή ψ ευ δή την ισότητα ΑΒ 2=ΒΓ.ΓΔ (θ έμα Π ανελλαδικών)

Η έκφραση αυτή δεν είναι πρόταση, αφού εξαρτάται από τη μεταβλητή χ = Α Β Γ που διαγράφει το σύνολο των ορθογωνίων τριγώνων. Αν γράφαμε: Για κάθε ορθογώνιο τρίγωνο ισχύει ΑΒ2=ΒΓΤΔ, τότε θα ήταν ψευδής πρόταση Αν γράφαμε: Υπάρχει ορθογώνιο τρίγωνο ώστε να ισχύει ΑΒ 2=ΒΓ·ΓΔ τότε θα ήταν αλη­ θής πρόταση, διότι ισχύει για κάθε ισοσκελές ορθογώνιο τρίγωνο. Ανάλογα οφείλουμε να εκφραζόμαστε και σε περίπτωση προτασιακών τύπων με περισ­ σότερες από δυο μεταβλητές. Για παράδειγμα: Δ

Αν α, β e [O,+oo) , τότε να χα ρ ακτηρίσετε ω ς ή ισότητ α την αλη θή ψευ δή = ..Γα β α+ +

.J Jβ Η έκφραση αυτή δεν είναι πρόταση αλλά προ­ τασιακός τύπος με δύο μεταβλητές. Αν γράφαμε: Για κάθε α,βε [Ο,+οο), ισχύει .Jα + β = -Γα + Jβ θα ήταν ψευδής πρόταση, αφού για α= ι ' β= ι δεν ισχύει, διότι .J2 2 . Αν γράφαμε: Υπάρχουν α,βε [Ο,+οο), ώστε να ισχύει .Jα + β = -Γα + Jβ τότε θα ήταν α­ ληθής πρόταση, αφού για α= ι , β=Ο ισχύει. *

Αν α,β ε � τότε να χαρ ακτηρίσετε ως αλη­ θή ή ψευ δή την ισοδυναμία: α2+β 2=0<::>α=Ο ή β=Ο,

Επίσης η έκφραση αυτή δεν είναι πρόταση. Αν όμως γράφαμε: Για κάθε α,β Ε JR. ισχύει ισοδυναμία: α 2 + β 2 = Ο <=> α = Ο ή β Ο , τότε θα ήταν ψευδής πρόταση αφού για α= ι , β=Ο η πρόταση ι 2+02=0 είναι ψευδής ενώ η πρόταση (Ι =Ο ή 0=0) είναι αληθής. Όπως ξέρουμε η ισοδυνα­ μία p�q, δύο προτάσεων p, q είναι αληθής, μόνον όταν οι p, q είναι συγχρόνως αληθείς ή ψευδείς. Αν γράφαμε: Υπάρχουν α,β Ε JR. ώστε να ισχύει η ισοδυναμία: α2+β2=0<=:>α=Ο ή β=Ο, τό­ τε θα ήταν αληθής πρόταση, αφού για α=Ο, β=Ο είναι αληθείς και δύο προτάσεις: 02+02=0 και (0=0 ή 0=0). Αλλά και για α=ι , β=2 είναι και δύο προτάσεις ι 2+22=0 ( ι=Ο ή 2=0) ψευ­ δείς. Άρα η ισοδυναμία ισχύει και στις δυο περιπτώσεις. =

Τέλος είναι γνωστό ότι μια έκφραση είναι πρόταση όταν έχει (πλήρες και αυτοτελές) νόημα και μπορεί κατά ένα και μόνο τρόπο να χαρακτηρισθεί ως αληθής ή ψευδής. Για παράδειγμα δεν είναι ορθό να ρωτήσουμε αν είναι αληθής ή ψευδής η πρόταση: Για κάθε χ Ε JR. , ισχύει χ ( χ + ι ) = χ + ι , αφού χ η έκφραση αυτή για χ=Ο δεν έχει νόημα, διότι τότε το πρώτο μέλος της δεν ορίζεται. Κανονικά πρέπει να γράψουμε: Για κάθε χ Ε JR. ισχύ­ ει χ ( χχ+ ι ) = χ + I (αληθής) και ανάλογα: Υπάρχει χ Ε JR. ώστε να ισχύει χ ( χ + ι ) = χ + ι χ (αληθής). Στη δεύτερη περίπτωση όμως συνηθίζεται να μην καθορίζουμε εμείς το σύνολο των τι­ μών του χ για τα οποία έχει νόημα αυτή η έκ­ φραση (πεδίο ορισμού εξίσωσης ή ανίσωσης), αλλά αφήνουμε να το βρει ο ερωτώμενος. Δη­ λαδή ρωτάμε αν: Υπάρχει χ Ε JR. ώστε να ι. . χ(χ + ι) = χ + ι και εννοουμε: σχυει χ Αν από τα χ Ε JR. για τα οποία αρχικά έχει νόημα αυτή η έκφραση κάποια επαληθεύουν τον προτασιακό τύπο που προκύπτει. Με αυτή τη λογική θεωρείται ορθό να ρωτήσουμε άν: Υπάρχει χ ε :ΙR ώστε ι.(;1 = if;J (αντί του κανονικού: Υπάρχει χ Ε [ Ο, +οο ) ώστε να ισχύει ι� = if;J ). Δεν είναι ορθό όμως να ρωτήσουμε αν: Για κάθε χ Ε JR. ισχύει ι� = if;J , αφού για χ<Ο δεν έχει νόημα αυ­ τή η έκφραση, διότι δεν ορίζεται το δεύτερο μέλος της. Το ορθό είναι να ρωτήσουμε αν: Για κάθε χε[Ο,+οο) ισχύει: ι.(;1 {;! Κλείνουμε με ένα ακόμη μη ορθώς διατυ­ πωμένο ερώτημα Πανελλαδικών εξετάσεων: Να χαρακτηρίσετε ως αληθή ή ψευδή την ισό. τητα εφ 2 α = 2εφα Δεν προκειται για προ-. ι + εφ α ταση αλλά για προτασιακό τύπο ο οποίος για άπειρα α=κπ ισχύει και για άπειρα κπ + -π , κπ , οπου . α =Ι- κπ + -π2 , κ ε Ζ δεν ι2 4 σχύει. Μερικά ακόμη ενδιαφέροντα παραδείγ­ ματα υπάρχουν στο άρθρο Απόλυτες τιμές Ρίζες πραγματικών αριθμών, για την Α· Λυκείου της συναδέλφου Κατερίνας Κλάδη.

ΕΥΚΛΕΙΔΗΣ Β ' τ.Ι/18

=

2


Τ ο Βή μα του Ευκλείδη

r Γύρω από

Θαλή: Δυο Αξιοσημείωτες Παρατηρήσεις το

Θεώρη μα

του

Δημήτρης Ι. �πουνάκης 1.

Από το Πυθαγό ρειο θεώρ η μα σ το Θεώ­ ρη μα του Θαλή .

Είναι γνωστό ότι ένας τρόπος για να αποδει­ χθεί το Πυθαγόρειο θεώ ρημα είναι να χρησιμοποι­ ηθεί η ομοιότητα των τριγώνων, η οποία στη ρίζε­ ται στο θεώρημα του Θαλή. Θα δείξουμε και το αντίστροφο, δηλαδή ότι με τη βοήθεια του Πυθα­ γορείου θεωρήματος μπορεί να αποδειχθεί το Θεώρημα του Θαλή. Επίσης θα δώσουμε μια από­ δειξη του θεωρήματος του Θαλή με τη βοήθεια της έννοιας του εμβαδού ευθύγραμμου σχήματος. Εί­ ναι φανερό ότι για να αποδειχθεί το γενικό θεώ ρη­ μα του Θαλή, αρκεί να αποδειχθεί το θεώρημα του Θαλή σε τρίγωνο. I. Ειδική περίπτωση .

Έ στω κατ ' αρχήν ορθογώνιο στο Α τρίγωνο Β Δ ΒΕ = - (ισο­ ΑΒΓ και Δ Ε//ΒΓ. Τ ότε ισχύει ΒΑ Β Γ ΒΔ ΒΕ δύναμα - = - ) ΔΑ ΕΓ

Φέρνουμε ΕΖ κάθετη στην ΑΓ. Από το ο ρθο­ γώνιο τρίγωνο ΕΖΓ, λόγω και του ο ρθογωνίου ΑΔΕΖ , έχουμε ΕΓ2=ΑΔ2+ΖΓ2 , ή (α-μ)2=(γ-κ)2+(β-λ)2 , ή λόγω των ( 1 ) αμ=γκ+ βλ ή αμ-γκ= βλ, ή α2 μ2+γ2r-2αμγκ=β2(μ2-κ2), ή μ2(α2-β2) +κ2(γ2+β2)-2αμγκ=Ο, ή μ2γ2+κ2α2-2αμγκ=Ο, ή ( μγ-κα)2=0 ' η, μγ=κα η, μα yκ η, ΒΕ ΒΔ =

'

ΒΓ

=

ΒΑ

1 1 . Γενική Περίπτω ση

Έ στω για τυχόν τ ρίγωνο ΑΒΓ και Δ Ε//ΒΓ. Τ ότε ισχύει ΑΔ ΑΕ ΑΔ ΑΕ , - = - (ισοδυναμα - = - ) ΑΒ ΑΓ ΔΒ Ε Γ

Α

-

Απόδειξη

Έστω κ=ΒΔ, λ=ΔΕ, μ=ΒΕ Β

Β

Γ

Απόδειξη

Αν η γωνία Β είναι οξεία (βλ. σχήμα) φέρνου­ με ΑΗ κάθετη στη ΒΓ, οπότε είναι κάθετη και στη ΔΕ. Από το ο ρ θογώνιο τρίγωνο ΑΒ Η , όπου ΔΖ//Β Η, καθώς και από το ορθογώνιο τρίγωνο ΑΓΗ , όπου ΖΕ//ΗΓ λόγω της περίπτωσης (Ι) ΑΔ ΑΖ ΑΖ ΑΕ οποτε , ΑΔ ΑΕ ΑΒ ΑΗ ' ' ΑΒ ΑΓ ΑΗ ΑΓ Όμοια εργαζόμαστε αν η γωνία Β είναι αμ­ βλεία, ενώ αν είναι ορθή έχουμε την περίπτωση (1). Επίσης όμοια είναι η απόδειξη αν η ΔΕ τέμνει τις προεκτάσεις των ΑΒ, ΑΓ. - = -

Ζ Α β Έχουμε μ2=κ2+λ2 , α2= β2+γ2 ( 1 )

Γ

ΕΥΚΛΕΙΔΗΣ Β' τ. l/19

- = -

- = - .


Τ ο Βήμα του Ευκλε ίδη 2.

Το Ο r.ώ ρ η μ α του Θ αλή με τη β ο ή θ ε ι α

της i:ννο ιας τ ο υ r. μ β α δ οίJ .

Με τη βοήθεια του τύπου για την εύρεση του εμβαδού τριγώνου θα αποδείξουμε το θεώρημα του Θαλή. Π ρ όταση

Έστω τρίγωνο ΑΒΓ και ΔΕ//ΒΓ . Τότε ισχύει ΑΔ ΑΕ (ισοδυναμα ΑΔ ΑΕ ) ΑΒ ΑΓ ΒΔ ΕΓ ,

- = -

- = -

Λ π ό δ r. ιξη

Επειδή τα τρίγωνα ΑΔΕ, ΒΔΕ έχουν το ίδιο ύψος από το Ε, έχουμε ( ΑΔΕ ) ΑΔ ( ΔΕΒ ) ΒΔ Α

Γ

Β

Ως γνωστό στα σχολικά βιβλία η απόδειξη του θεωρήματος του Θαλή συνήθως παραλείπεται, κυ­ ρίως λόγω δυσκολίας που υπάρχει στην περίπτωση άρρητου λόγου. Το θεώρημα όμως αυτό μπορεί να αποδειχθεί με τη βοήθεια της έννοιας του εμβαδού όπως είδαμε, όπου βέβαια η δυσκολία μεταβιβάζε­ ται στην απόδειξη του τύπου του εμβαδού τετρα­ γώνου με πλευρά άρρητο. Με την βοήθεια του τύ­ που αυτού αποδεικνύεται, ως γνωστόν, ο τύπος για το εμβαδόν τριγώνου που χρησιμοποιήσαμε παρα­ πάνω στο (2). Όμως το εμβαδόν έχει μια άμεση εποπτεία και φυσικότητα που το καθιστά σχεδόν πρωταρχική έννοια για τα σχολικά μαθηματικά. Έτσι αν προηγηθεί στην ύλη της Γεωμετρίας της Β ' Λυκείου η έννοια του εμβαδού, θα δώσει τη δυνατότητα να αποδειχθεί, κατ ' αρχήν το Πυθαγό­ ρειο Θεώρημα (με τη θαυμάσια απόδειξη του Ευ­ κλείδη) αλλά και το Θεώρημα του Θαλή, με μια από τις παραπάνω αποδείξεις. Υπόψη ότι με τη βοήθεια του εμβαδού μπορούν να αποδειχθούν ευκολότερα και κομψότερα και πολλές άλλες γεω­ μετρικές προτάσεις και μπορεί να αποτελέσει μελ­ λοντικά βάση για ένα διαφορετικό τρόπο διδασκα­ λίας της Γεωμετρίας στη Β 'Λυκείου. Σχόλιο

του

τ ασσόπουλου

Γ

Όμοια από τα τρίγωνα ΑΔΕ, ΔΕΓ έχουμε ( ΑΔΕ ) ΑΕ ( ΔΕΓ ) ΕΓ Αλλά τα τρίγωνα ΔΕΒ, ΔΕΓ είναι ισοδύναμα, αφού έχουν την ίδια βάση ΔΕ και ίσα ύψη, λόγω ΔΕ//ΒΓ. Έτσι από τις παραπάνω αναλογίες προκύπτει

συν αδ έλφου

κ.

Γ ιώργ ου

Αγαπητέ συνάδελφε, η εργασία σας κρίθηκε αρκετά ενδιαφέρουσα και δημοσιεύθηκε με μεγά­ λη μας ευχαρίστηση. Με ικανοποίησε δε ιδιαίτερα το γεγονός ότι έχετε πλήρη συναίσθηση του ότι δε βασίζεται μόνο στο Πυθαγόρειο Θεώρημα με τη μορφή εμβαδών (ΑΒΔΕ)+ (ΑΓΖΗ)=(ΒΓΚΛ), όπως υπάρχει στα στοιχεία του Ευκλείδη αλλά στην ι­ σοδύναμη μορφή β2+γ2=α2 •

=

z

ΑΔ ΑΕ ΒΔ ΕΓ

-

=

-

Δ ι δ α κτ ι κ ό Σχόλιο

Λ

ΕΥΚΛΕΙΔΗΣ Β' τ. Ι /20

κ


Τ ο Βή μα του Ευκλείδη

β2+y2=α2 προκύπτει με δυο απλές ομοιότητες τρι­ γώνων, ενώ αν προηγηθεί το Πυθαγόρειο με τη μορφή β2+γ2=α2 τότε όπως βλέπουμε η απόδειξη είναι αρκετά πιο δύσκολη.

γ

Επ' ευκαιρία παραθέτω και μια πιο σύντομη απόδειξη του Θ. Θαλή με βάση το Πυθαγόρειο Θεώρημα (από τις παραδόσεις του καθηγητή κ. Στυλιανού Νεγρεπόντη στο Μεταπτυχιακό τμήμα Διδακτικής και Μεθοδολογίας των Μαθηματικών).

β

Αν λοιπόν συμπληρωθεί το σχήμα σας ώστε να Δηλαδή όπως ορθώς επισημάνατε θα πρέπει να προηγηθεί, το ότι το εμβαδόν του τετραγώνου Τ με δημιουργηθεί το παραλληλόγραμμο ΒΑΓΡ τότε πλευρά χ, είναι (τ) = χ2 , του οποίου άμεση εφαρ­ προφανώς (ΔΑΖΕ) = (ΕΗΡΘ) (1) μογή είναι ο τύπος του εμβαδού ορθογωνίου, και Οπότε (1) τριγώνου ::::> λ · ( γ - κ ) = κ ( β - λ ) ::::> λγ = κβ ::::> -κ = ­λ γ β Όμως και η απόδειξη αυτού του τύπου και η z z z z μ => -κ = -λ = -μ κ =λ = κ + ').! = απόδειξη του θεωρήματος Θαλή γίνονται ουσια­ ::::> z z z 2 γ β γ + β αz γ β α στικά με βάση την πρόταση ότι: Αν δυο πραγματι­ κοί αριθμοί είναι άνισοι τότε υπάρχει μεταξύ τους ρητός αριθμός ή με βάση τον ισοδύναμο αριθμό Με φιλικούς χαιρετισμούς Γ, Τα σ σόπουλος ισότητας λόγων κατά τον Εύδοξο. Η διαφορά είναι ότι αν προηγηθεί το Θ. Θαλή τότε η σχέση

Μουσική

κα ι

Η.λupοφοpικιί

Υπεύθυνος: Νίκος Ανδρουλακάκης - Μουσικομαθηματικός. Ο Χάρης Στυλιανάκης (Β ' Λ υκείου Κορυδαλλού)

με επιστολή του μας ζητά να δημοσιεύσουμε «α­ ναλυτικό άρθρο σχετικά με Μουσική και Μαθημα­

Το θέμα είναι τεράστιο, οπότε θα το καλύ­ ψουμε σε συνέχειες, και μάλιστα επεκτεινόμενοι στην «Μουσική και Πληροφορική». Λεπτομερέστερα, στο μέλλον θα εξετάσουμε πώς ο Πυθαγόρας συνέδεσε την αρμονία της Μου­ σικής με την αρμονία του σύμπαντος και των α­ ριθμών. Πώς με απλούς αριθμούς, δημιούργησε τις νότες, τις κλίμακες, τα διαστήματα. Πώς «μετρά­ με» τους ήχους με τη συχνότητα και τα Ντεσι­ μπέλ-Λογαρίθμους. Πώς, όσες/οι ενδιαφέρονται, θα παίξουν στα πλήκτρα, πρακτικά, αρχικά με το ένα δάκτυλο, αγαπημένες τους μελωδίες και μετά, τικά».

αν θέλουν, να τις συνοδεύσουν με το αριστερό χέ­ ρι, με απλές συγχορδίες και ρυθμούς. Ακόμα, πώς οπτικοποιούμε τους ήχους με γραφικές παραστά­ σεις. Επίσης, πώς θα χρησιμοποιήσουμε τον ηλ. υπολογιστή μας, ακόμα κι αν δεν είμαστε μουσι­ κοί, ώστε, διαλέγοντας τραγούδια, ν' αναλύσουμε την ενορχήστρωσή τους και να τα διασκευάσουμε, αλλάζοντας ρυθμούς και όργανα, απ' όποιο είδος μουσικής προτιμάμε. Τέλος, θα είναι χαρά μας, όποιες/οι αναγνώ­ στριες/ες μας θέλουν να ενημερωθούν για κάτι σχετικό και ιδιαίτερο, να μας το ζητήσουν στην ηλεκτρονική διεύθυνση i n t'o (ιι\1 n1 s . � Γ

ΕΥΚΛΕΙΔΗΣ Β ' τ. t /2 1


'1.

�� " """

"r

-n"

, Jι �' ���� ,

.

ο ο

::;;., �

.

Η'Ν' NιiTHeNAT/�Vf

Η Homo Mathematicus είναι μια στήλη στο περιοδικό μας, με σκοπό την ανταλλαγή απόψεων και την ανάπτυξη προβληματι­

σμού πάνω στα εξής θέματα: I ) Τι είναι τα Μαθηματικά, 2) Πρέπει ή όχι να διδάσκονται, 3) Ποιοι είναι οι κλάδοι των Μαθημα­ τικών και ποιο το αντικείμενο του καθενός, 4) Ποιες είναι οι εφαρμογές τους, 5) Ποιες επιστήμες ή κλάδοι επιστημών απαιτούν καλή γνώση των Μαθηματικών για να μπορέσει κάποιος να τους σπουδάσει. Για τους συνεργάτες της στήλης: παράκληση ! τα κείμενα της στήλης αυτής, ως προς το περιεχόμενό τους και ως προς το επί­ πεδό τους, θα πρέπει να είναι συμβιβαστά με τα ενδιαφέροντα και το επίπεδο κατανόησης από μέρους των παιδιών.

I.

Ε π ι μέλεια : Γ. Κερασαρίδης, Β . Ταπεινός, Β . Καρκάvης, Β . Ζώτος

"οι συ νεργάτες της στιί).ης γράφουν-ερωτούν "

Προλεγόμενα .

Ι " θέμα: Οι μέλισσες σπου δ άζουν Μαθ η ματικά και Α ρχιτεκτονική

Συνεχίζουμε με το δεύτερο και τελευταίο μέρος της ενδιαφέρουσας αυτής εργασίας. «Το κυνήγι της βέλτιστης λύση ς», του Παναγιώτη Τρ ιπικέλη ( Κό ρ ινθος) Το πρόβλη μα στις τρεις δ ιαστάσεις : (2ο Μ έρος)

Ωστόσο είναι φανερό ότι οι μέλισσες αντιμετωπίζουν το πρόβλημα στις τρεις διαστάσεις ,αφού τα κελιά τους εκτός από πόρτες έχουν και εσωτερικό. Εδώ τα πράγματα δυσκολεύουν και η μελέτη του αντιστοίχου προβλήματος ξεπερνά τις γνώσεις του αναγνώστη. Αυτό που αξίζει να αναφερθεί είναι ότι αν και θα περίμενε κανείς τα κελιά της κυψέλης να είναι κανονικά εξαγωνικά πρίσματα, αυτό τελικά δεν ισχύει. Η κάθετη τομή ενός κελιού είναι κανονικό εξάγωνο ,αλλά το πίσω τοίχωμα του αποτελείται από τρεις ίσους ρόμβους. Η αντοχή τους ,δε, είναι εξωπραγματική. Αν και το τοίχωμα του κελιού είναι μόνο 0,05 χιλιοστά παχύ , το κελί αντέχει καταπόνηση 25 φορές μεγαλύτερη από το βάρος του.

Έχοντας στο μυαλό του το πρόβλημα χωρητικότητας των κελιών στις τρεις διαστάσεις ο Ούγ­ γρος μαθηματικός Fejes Toth διατύπωσε το «ισο­ περιμετρικό πρόβλημα κυψέλης» όπου αναζητείται στις τρεις διαστάσεις το σχήμα του κελιού με τη μεγαλύτερη χωρητικότητα. Αν και μέχρι το 1 998 το πρόβλημα δεν είχε λυθεί έχει αποδειχθεί ότι η λύση που έχουν δώσει οι μέλισσες πλησιάζει αλλά δεν είναι η ΒΕΛ τΙΣΤΗ! ! Η αίσθηση του συντάκτη του άρθρου είναι ότι οι μέλισσες έχουν το θεωρητικό υπόβαθρο για να δώσουν λύση ,αλλά δυσκολεύονται στη κατα­ σκευή! ! [e- βιβλιογραφία :www .microscopy­ uk.org.uk/artsep98/hexagon.html ]

2° θέμα : Εφαρμο γές από τη «θεωρία του μαθη ματικο ύ μπιλιάρ δ ου»

Συνεχίζουμε με το τρίτο και τελευταίο μέρος της ενδιαφέρουσας αυτής εργασίας. «Τα Μ αθ η ματικά και το " αεικίνητο " » (3" μέρος), από το Θ άνο Λύ πα Ακολουθήσατε λαθεμένο δρόμο αν προσπαθήσα- προκαλεί καθόλου διαμαρτυρίες. Σε πολλές περι­ τε να βρείτε κάποιο σφάλμα στη Φυσική - στο γε- πτώσεις αυτή η αδιαφορία για τις διαστάσεις των γονός, για παράδειγμα, ότι αγνοήσαμε τη διάδοση σωμάτων είναι δικαιολογημένη. Για παράδειγμα, η της θερμότητας δια μεταφοράς. Γιατί; Επειδή μπο- κίνηση ενός σώματος υπό την επίδραση δεδομένων ρούμε να δημιουργήσουμε κενό μέσα στο δοχείο. δυνάμεων που ασκούνται στο κέντρο της μάζας του Όμως, τι άλλο υπάρχει μέσα στη μικρή μας «μηχα- δεν επηρεάζεται από το μέγεθός του και το σχήμα νή» εκτός από Μαθηματικά και Φυσική ; Το σφάλ- του. Ή, όπως απέδειξε ο Νεύτων, ένα σώμα που μα κρύβεται στο σύνορο ανάμεσα σ' αυτές τις δύο αποτελείται από πλήθος ομόκεντρων ομογενών επιστήμες - στη μετάβαση από τη φυσική διαδικα- σφαιρικών στρωμάτων έλκει τα άλλα σώματα όπως τα έλκει η ίδια μάζα συγκεντρωμένη στο κέντρο σία στο μαθηματικό της μοντέλο. Θυμηθείτε ότι ασχοληθήκαμε με δύο σώματα του. Βλέπουμε, λοιπόν, ότι η αντικατάσταση ενός τοποθετημένα στις εστίες. Επομένως αγνοήσαμε τις σώματος με ένα σημείο είναι μια συνηθισμένη διαστάσεις των σωμάτων. Αυτός είναι ένας συνη- πρακτική στη Φυσική. Στην περίπτωσή μας, όμως, θισμένος τρόπος σκέψης στη Φυσική, όπου η φρά- αυτό οδηγεί σε σφάλμα. ση «τοποθετούμε ένα σώμα στο σημείο Μ» δεν Ας δούμε κατά πόσο εξακολουθούν να ισχύουν οι συλλογισμοί μας όταν ως σώματα θεωρήσουμε σφαί­ ρες με μη μηδενικές ακτίνες και με κέντρο τους τις εστίες της έλλειψης. Προλεγό μενα .

ΕΥΚΛΕΙΔΗΣ Β ' τ.Ι/22


ιΥ,ιΙΙ, ιΙΙιiΤιΥ&ιΙΙΑ//CΙ/f

------

σχήμα 2 σχήμα 3 σχήμα Ι Θεωρήστε τρεις ακτίνες που ανακλώνται στο ίδιο Εάν μια ακτίνα εκπέμπεται από την εστία μιας σημείο Μ της έλλειψης, το οποίο βρίσκεται στο α- έλλειψης, μετά την πρώτη ανάκλασή της θα περά­ ριστερό της τμήμα (σχήμα Ι): μια ακτίνα που σει από την άλλη εστία. Αν δεν υπάρχει εκεί κά­ «προέρχεται» από την εστία F 2 , και δύο ακτίνες ποιο σώμα να την εκτρέψει, ανακλάται και πάλι και ΑΜ και ΒΜ που προέρχονται από τα σημεία Α και επιστρέφει στην πρώτη εστία, έπειτα επιστρέφει Β, τα οποία απέχουν απόσταση r από την εστία F2 • ξανά στη δεύτερη, κοκ. Αυτό είναι αρκετά προφα­ Μετά την ανάκλαση , η πρώτη ακτίνα θα περάσει νές. Είναι λιγότερο προφανές, όμως, και αρκετά από την εστία F και οι άλλες δύο θα περάσουν από παράξενο, το ότι έπειτα από κάθε ανάκλαση η τ ρο­ σημεία που απέχουν ίση απόσταση R από την F χιά προσεγγίζει την ευθεία που διέρχεται από τις [διότι γων. AMF2 =γων. F2ΜΒ=γων.Β Ι Μ δύο εστίες, και ότι οριακά ταυτίζεται με το τμήμα F Ι =γων. F I MA I ] . Από την ομοιότητα των ορ- αυτής της ευθείας το οποίο εμπεριέχεται στην έλ­ θογωνίων τριγώνων AMF2 και F I MA I , έχου- λειψη. Όταν το πρώτο τμήμα της τροχιάς τέμνει με r/R= AF2/A I F I = F2M/ F I M. Το Μ βρίσκε- αυτή την ευθεία σ' ένα σημείο διαφορετικό από τις ται στο αριστερό μέρος της έλλειψης, και επο- εστίες, η τροχιά δεν περνά ποτέ από κάποια εστία. Επιπλέον αν το σημείο τομής βρίσκεται έξω από το μένως F2M<F I M, και άρα R>r. Αυτό σημαίνει ευθύγραμμο τμήμα που συνδέει τις δύο εστίες, το ότι κάποιες από τις ακτίνες που θα ξεκινούν ίδιο θα ισχύει για όλα τα σημεία τομής (σχήμα 2). από μια σφαίρα ακτίνας r και με κέντρο στο F2 Το πιο εντυπωσιακό είναι ότι υπάρχει μια μικρότε­ δεν θα επιστρέψουν ποτέ σ ' αυτήν έπειτα από ρη έλλειψη με τις ίδιες εστίες με την πρώτη, στην κάποιες aντανακλάσεις και ούτε θα χτυπήσουν οποία η τροχιά εφάπτεται έπειτα από κάθε ανάκλα­ στην ιδίων διαστάσεων σφαίρα στο F l (αυτές ση. Ομοίως, μια τροχιά που τέμνει το ευθύγραμμο οι ακτίνες διασπείρονται). Και το γεγονός αυτό μεταξύ των δύο εστιών (σχή μα 3 ) θα κάνει τμήμα καταστρέφει όλους τους συλλογισμούς και τις το ίδιο έπειτά από κάθε ανάκλαση, και θα εφάπτεκατασκευές μας. Όσοι αγαπούν τη Φυσική μπορούν τώρα να ανα- ται μιας υπερ βολής με τις ίδιες εστίες. (Υπερβολή πνεύσουν ανακουφισμένοι: 0 Δεύτερος Νόμος της είναι η καμπύλη η οποία σχηματίζεται από όλα ε­ Θερμοδυναμικής παραμένει απαραβίαστος. Και για κείνα τα σημεία που η διαφορά των αποστάσεών όσους αγαπούν τα Μαθηματικά έχουμε μερικά επι- τους από τις εστίες είναι σταθερή) πλέον περίεργα φαινόμενα που σχετίζονται με τη συμπεριφορά των ακτίνων που ανακλώνται σε ένα ελλειπτικό κάτοπτρο. ι

ι

.

3 ° Θέμα: Για την τριχοτόμη ση της γωνίας

Από το φίλο της στήλης μας , Νίκο Βαδιβούλη, λάβαμε μια εργασία σχετικά με το θέμα που ανέπτυξε ο Ιπποκράτης Δάκογλου ("Homo mathematicus", ΕΥΚΛΕΙΔΗΣ Β ' , τεύχος 58,). Εμείς το θεωρούμε σαν συμβολή στην ενημέρωση πάνω σ' αυτό το θέμα, γι' αυτό και το δημοσιεύουμε. Προλεγό μενα:

« Μ ια τριχοτό μος καμπύ λη», του Νίκου Βαδιβο ύλη (Άρτα)

Το πρόβλημα της τριχοτόμησης μιας τυχαίας πρόβλημα της τριχοτόμησης στηριζόμενοι στη θεω­ γωνίας είναι αλγεβρικό τριτοβάθμιο και αποδεί- ρία των κωνικών καμπύλων. Σημειώνω ότι οι κωνικές δεν είναι οι μοναδικές χθηκε άλυτο με τον ευκλείδειο τρόπο μαθηματικής αλγεβρικές καμπύλες που επιλύουν το πρόβλημα. συμπεριφοράς. Ο ευκλείδειος τρόπος μαθηματικής συμπεριφοΜια τέτοια, προσιτή στους μαθητές, θα παράς δέχεται την ύπαρξη ως κατασκευαστική δυνατό- ρουσιάσω εκκινώντας από τις μετρικές σχέσεις της ευκλείδειας γεωμετρίας. τητα με τη βοήθεια του κανόνα και του διαβήτη. Οι αρχαίοι Έλληνες με υπέρβαση του ευκλείδει­ ου τρόπου κατασκευής, πέτυχαν να επιλύσουν το Ας θεωρήσουμε, λοιπόν, την όχι ευθεία και μη μηδενική κυρτή γωνία ΧΑΨ και τα σημεία Β,Γ στις πλευρές της ΑΧ, ΑΨ αντιστοίχως, ώστε (ΑΒ )=(ΑΓ)= Ι . ΕΥΚΛΕΙΔΗΣ Β ' τ.l/23


Α (-Ι,α') ,... e

χ'

χ

σχ1Ί μα 4

ψ

I

I

ι

ψ

� ,

I/

i(-2, α'-4)

' , , (0, ο

,

α2-2)

A( J ,O / '

(��;_4)

I

I

I

I

I

f (2 ,α')

ιΔ(ξ,Ο)

σχ1Ίψ'μα 5

(ε)

χ

χ

I

Ι

Ι

,'

ι'

Ι

I

ψ \\

1 I I

\

ο

I

\

A( l ,O) \

' ',

/

( ; , -; )

σχ1Ίψ 'μα 6

ΙI

(ξ, 2δυνΑ) �

,' Δ(ξ,Ο)

στο τρίγωνο ΑΒΓ, του σχήματος που προηγήθηκε, τομείται με τριχοτόμο μήκος w= J3 -1>0. λαμβάνουμε:2 α2=1 2+ 1 2 -2· 1 · 1 · συνΑ� Αν θέσουμε (γων.ΧΑΨ)=Α, τότε με την εφαρ­ +α2-2=0 κα- μογή του θεωρήματος του συνημιτόνου γωνίας για -2συνΑ= α -2, οπότε η εξίσωση: χ3 -3χ θίσταται ισοδύναμη της εξίσωσης: χ3 -3χ-2συνΑ=Ο. κάθε ένα των διαστημάτων [-2,0 ], [0,1 ], [1 ,2] ισχύΑν3 τώρα2 θεωρήσουμε τη συνάρτηση f με ουν οι προϋποθέσεις του θεωρήματος Bo1zano. f(x)=x -3χ+α -2, διαπιστώνουμε ότι γι' αυτή και Άρα η εξίσωση f(x)=O, έχει ανά μία ρίζα σε καθένα Ακολούθως θεωρούμε την ύπαρξη των τριχοτόμων των διαστημάτων (-2,0), (0,1), (1 ,2) με δεκτή τη ρί­ ΑΤ, ΑΣ που τέμνουν την ευθεία ΒΓ στα σημεία ζα ξε(1 ,2), αφού πρέπει w=ξ-1>0. Με την προϋπό­ Δ,Ε αντιστοίχως, ώστε το σημείο Δ να ευρίσκεται θεση: α2>2<=:>(γων.ΧΑΨ)>90° και ύστερα από τη μεταξύ των Β,Ε ( σχ11 μα 4 ) . μελέτη της συνάρτησης f, παρουσιάζουμε το στικτό Εύκολα διαπιστώνεται ότι τα τρίγωνα ΑΒΔ, της διάγραμμα στο 9ix 9i επίπεδο ( σχ11 μ α 5 ) ΑΓΕ είναι tσα, οπότε μπορούμε να θέσουμε : Η δεκτή ρίζα ξ παρουσιάζεται στο σχ11 μα 5 ίση (ΒΔ)=(ΓΕ)=y, (ΑΔ)=(ΑΕ)=w και (ΒΓ)=α με το οπότε για το τριχοτόμο μήκος w Επειδή για το διχοτόμο μήκος δα στο τρίγωνο ισχύει:μήκος (ΟΔ), w=(ΑΔ)=(ΟΔ)-(ΟΑ)=ξ-1>0. Επειδή οι3 εξισώσεις: ΑΒΓ ισχύει: δ � =βγ[1-( -α- )2], κατ' αντιστοιχία χ -3χ+α2-2=0 και χ3 -3χ-2συνΑ=Ο β+γ είναι ισοδύναμες, από τη δεύτερη προκύπτει η ισο­ και στο τρίγωνο ΑΒΕ θα ισχύει: w= 1 -( α -+Υ )2 , δύναμη προς τις δύο προηγούμενες εξίσωση: χ3 w 1 3χ=2συνΑ, η οποία έχει τρεις ρίζες πραγματικές ή αφού w>O. το ίδιο ακριβώς η ευθεία (ε): y=2συνΑ τέμνει την Από την εφαρμογή του θεωρήματος της εσωτε­ καμπύλη (C): y= χ3 -3χ σε τρία σημεία ( συ1 μα 6 ) ρικής διχοτόμου στο τρίγωνο ΑΔΓ προκύπτει: και το ζητούμενο είναι εκείνο με συντεταγμένες (ξ,2συνΑ) με ξ> 1 . Επιπροσθέτως από τη μελέτη _Υ_ = _!_ � . . . . �w3+3w2+α2-4=0. της συνάρτησης h με h(x)= χ3 -3χ, προκύπτει ότι για α - 2y w Η τελευταία εξίσωση, ύστερα από την αντικα­ τα τοπικά της ακρότατα ισχύουν hmax=h(- 1 )=2, hmin=h(l)=-2. τάσταση w=x-1 γίνεται3 2 Επίσης ότι η τεταγμένη τυχαίου σημείου της χ -3χ+α -2=0. 2 ευθείας (ε) ανήκει στο διάστημα (-2,2), αφού Είναι προφανές ότι: α =2<=:>(γων.ΧΑΨ)=90° -2<2συνΑ<2, επειδή η γωνία ΧΑΨ, μέτρου Α, είναι α2>2<=>(γων.ΧΑΨ)>90° και κυρτή μη μηδενική και όχι ευθεία. 2 α <2<=:>(γων.ΧΑΨ)<90° Με την προϋπόθεση ότι Α<90° , η απεικόνιση Έτσι στην περίπτωση κατά την οποία της καμπύλης και της ευθείας (ε), ύστερα από (γων.ΧΑΨ)=90°, η εξίσωσή μας λαμβάνει τη μορ- τη μελέτη τους,(c)παρουσιάζεται στο 9ix 9i επίπεδο φή χ3 -3χ=Ο<=:> χ(χ+ J3 )(χ- J3 )=0, από την οποία με το σχ1Ί μα 6 . προκύπτει η δεκτή λύση χ= J3 και εκείθεν η λύση w= J3 - 1 , που αποδεικνύει ότι η ορθή γωνία τριχοΑπό την προηγούμενη απεικόνιση συμπεραίνουμε ότι η ζητούμενη ρίζα είναι: w=(ΑΔ)=(ΟΔ)­ (ΟΑ)=ξ-1 , που εκφράζει το τριχοτόμο μήκος. Κατανοώντας τα προηγούμενα ο μαθητής, παραλαμβάνει από τον υπολογιστή του το τριχοτόμο μήκος και με τη βοήθεια του διαβήτη το μεταφέρει για χρήση στο χαρτί σχεδίασης. Θέλω να σημειώσω ότι η τριχοτόμηση μιας μη κυρτής γωνίας ανάγεται στην τριχοτόμηση της αντι­ στοίχου κυρτής και ότι η τριχοτόμηση της ευθείας γωνίας είναι σχεδόν απλή. Επίσης θέλω να σημειώσω ότι ο P.L. Wantzel ανακοίνωσε το έτος 1 837 το άλυτο του προβλήματος με τον ευκλείδειο τρόπο μαθηματικής συμπεριφοράς, αφού απέδειξε ότι υπάρχουν άπειρες γωνίες που ΕΥΚΛΕΙΔΗΣ Β ' τ. l/24


τριχοτομούνται στοιχειωδώς και επισημαίνω ότι η διαπίστωση αυτή δεν αναιρεί το άλυτο του προβλήμα­ τος στη γενική του μορφή. Από τα προηγούμενα μπορεί ο μαθητής να δια- τρεις ρίζες πραγματικές με προσδιορισμό του προ­ πιστώσει ότι η ευκλείδεια γεωμετρία τροφοδοτεί σήμου αυτών. την ανάλυση με σπουδαίες ασκήσεις. Από αυτές ί\ σ κη ση 2 '1 : Αν Ο<α<2 και αε91, να μελετηθεί που εξάγονται από το προηγούμενο πόνημα θα α­ η συνάρτηση h με h(x)= χ3 -3χ+α2 -2, να γίνει η ναφέρω μόνο δύο και κατά την άποψή μου τις α­ γραφική της παράσταση στο 91χ 91 επίπεδο και να ντιπροσωπευτικότερες. αποδειχθεί ότι η εξίσωση h(x)=O έχει τρεις ρίζες Λσκηση l η : Να μελετηθεί η συνάρτηση f με πραγματικές με προσδιορισμό του προσήμου αυf(x)= χ3 -3χ-2συνΑ,όπου Α το μέτρο γωνίας τριγώ­ τών. νου, να γίνει η γραφική της παράσταση στο 91χ 91 επίπεδο και να αποδειχθεί ότι η εξίσωση f(x)=O έχει 4"

Θi;μα: Μ ισ: νi:α αντίλη ψη γ ω

το

μi:τρ ο γωνίας

Από το φίλο της στήλης Χρήστο Δ. Αγγελόπουλο, λάβαμε μια εργασία, με θέμα «Ένας έννοιας ''μέτρο γωνίας "». Εμείς σας παραθέτουμε τις σκέψεις του και περιμένουμε τις έ της ν ος ορισμός παρατηρήσεις σας. Π ρ ολr.γόμε,· α :

Υ

Β'

ο ρ ισμός ηnJ μf.τρου γωνiης " , k

του

Χρι1στου Αγγελόπουλου (AOl'i \'ff.)

Ο ρ ισμός

του

μi:ηHnJ γων ί α ς :

Θεωρούμε (σχ. 7) σύστημα αναφοράς Oxy. Προσαρτούμε σ' αυτό ένα τετράγωνο ΑΒΓ Δ, τυχαίας πλευράς, με τις διαγωνίους του πάνω στους άξονες. Θεωρούμε μια γωνία kox, μέτρου θ, με την αρχική πλευ ρά οχ πάνω στον ομώνυμο άξονα και την τελική πλευρά της ok να τέμνει την πλευρά (π.χ. ΑΒ) του τετραγώνου, στο Ζ . Συμβολίζουμε με s την αλγεβρική τιμή του ΑΖ . Ορίζουμε σαν μέτρο της γωνίας κοχ και το συμβολίζουμε με θ, το μ έ­ γεθος

θ = !__

Δ'

Λ

σχήμα 7

l:υμβι'lσεις:

Λ

R

Το μέτρο μ( θ ) της γωνίας θ (όπως φαίνεται από το σχήμα 7), δεν επηρεάζεται από την εκλογή του τετραγώνου, αφού για ένα άλλο τε­ τράγωνο Α ' Β 'Γ Δ ' , τα τρίγωνα ΑΟΖ , Α Ό Ζ' είναι όμοια, άρα θ= � = � R R

Λ

Λ

α) Στο εξής θα συμβολίζουμε με θ την ίδια τη γωνία και με μ( θ ) ή θ το μέτρο της γωνίας

kox. β) Η κίνηση, πάνω στην περίμετρο του τετραγώνου ΑΒΓΔ, που έχει φορά αντίθετη προς την κίνηση των δεικτών του ρολογιού, θα θεωρείται θετική. γ) Στην περίπτωση αυτή οι μονάδες μέτρησης των γωνιών δεν ονομάζονται πλέον μοίρες ή ακτίνια, ο­ νομάστε τες όπως θέλετε (π. χ. "αγγελίνια"). Το μέτρο της πλήρους γωνίας θα το συμβολίζουμε με 2π*. Π ρ<φλη μ α : Με βάση τα παραπάνω, μπορείτε να αποδείξετε τους παρακάτω τύπους ; π * , εφ θ = -Υ = θ με Os θ < π* 2π*=4 12 - R => π*=2 ν2r: , Os θ s----τ:χ R 2 2 ν2 - θ -::* J2 - θ θ Π συνθ= -------.---ι , ημθ= , ι , με Οs θ s 2 J2 {ι + θ2 - J2θ )2 J2 {ι + θ2 - J2θ )2 Λ

Λ

Λ

--

Λ

i\il ια παρατi1J>η ση : Από τη μορφή του παραπάνω τυπολογίου προκύπτει ότι, σε αντίθεση με την κλασική Τριγωνομετρία, εδώ μπορούμε να υπολογίσουμε τους τριγωνομετρικούς αριθμούς οποιασδήποτε γωνίας, χωρίς να καταφύγουμε στους τριγωνομετρικούς πίνακες. ΕΥΚΛΕΙΔΗΣ Β ' τ.Ι/25


h'ΩaΙιιιι•••• [}JO(]J fJ[j]iJ/ liJ1 fJ(/j{J[j] fJ(f)fJ) c6Jr!JωC3fJ(f)r!J Άλγ.ε. β ρα Απόλυτες Τιμές - Ρίζες Πραγμα.τικών Αριθμών

της Κατερ ίνας Κλάδη

Τελικά I χ I = -χ� χ::Ξ:Ο Η απόλυτη τιμή ενός πραγματικού αριθμού α, συμβολίζεται με I α I και ορίζεται ως εξής: η αν α� Ο ' ) Ε φαρμογή 2 ι αι = -α, αν α<Ο (Ι) (ορισμος, σχολικου, βιβλιου Ν α λυθο ύ ν οι ανισώσεις : i) Ι χ Ι :::: χ, ii) Ι χ Ι :::: -χ, iii) ΙχΙ>χ, iv) ΙχΙ> -χ π.χ 1 3 1 = 3, 1 0 1 = ο, I -5 1 = --(-5)=5 Ισοδύναμα ισχύει: Λ ύση α, αν α > Ο (11), ΙαΙ= Ο, αν α Ο (ΙΙΙ) , Αφού Ι α Ι = -α, I χ I =::χ και I χ Ι =::-χ, για κάθε χ ε IR , θα = αν α ::; Ο έχουμε: -α, ανα< Ο i) I χ I :::: χ <=> I χ I =χ <=> χ:::ο Ι α Ι = α,-α,αναναα�::;ΟΟ (IV) ii) I χ I ::::-χ � I χ I = -χ� χ::::ο iii) I χ I > χ � χ <Ο Οι ορισμοί (I) και (11) φαίνεται να μεροληπτούν ίν) l x l > -χ� χ>Ο υπέρ των θετικών ή των αρνητικών αριθμών αντι­ στοίχως, εVώ οι (ΠΙ) και (IV) είναι πράγματι αντι­ Ε φαρμογή 3 η κειμενικοί. Θα δούμε παρακάτω ότι για την απο­ Ν α λυθ ούν οι παρ ακάτω β ασικές εξισώσεις φυγή παρερμηνειών, συμφέρει να χρησιμοποιή­ ανισώσεις: σουμε τον τελευταίο ορισμό ii) I χ-1 1 = l 2x-5 l i) l -2x-l l =3 iii) l -3x+4 1 =-6 iv) Ι χ-4 1 =3χ+2 η l Ε φ αρ μ ογή vi) Ι χ-2 1 >4 ν) l -2x+l i ::Ξ:3 θ Ν α λυ ούν οι εξισώσεις: i) I χ I =χ, ii) I χ I = -χ viii) l 4x-l l ::;ο vii) l 3x+ l l 2:-2

{α, { {

Λύση

{α, ανα>Ο

i) I χ I =χ � χ=::Ο, αφού για χ<Ο είναι αδύνατη. ii) Για την εξίσωση I χ I = -χ, πιθανόν να παρερ­ μηνευθεί ο ορισμός του σχολικού βιβλίου και οι μαθητές να απαντήσουν I χ I = -χ � χ<Ο, ενώ ο ορισμός λέει ότι χ<Ο => I χ I = -χ και όχι αντιστρόφως. Εξετά­ ζουμε αν ισχύει και για χ=::Ο . Τότε Ι χ Ι = - χ <:::::> χ = -χ � 2χ = Ο � χ = Ο.

Λύση :

Για τη λύση των βασικών εξισώσεων, χρησι­ μοποιούμε τις ισοδυναμίες: 1 . Ι χ Ι =α � χ=α ή χ= -α με α=::Ο 2. l x l = l α l � χ=α ή χ= -α Για τη λύση βασικών ανισώσεων, χρησιμοποι­ ούμε τις ισοδυναμίες: 3. I χ I ::Ξ:θ � -θ::Ξ: χ::Ξ:θ , για θ>Ο. 4 . I χ I :::θ � χ ::Ξ: -θ ή χ�θ για θ>Ο.

ΕΥΚΛΕΙΔΗΣ Β ' τ.Ι/26


Μαθηματικά για την Α ' Λυκείου

} } ) (

} }

l -2x-1 1 =3 <::> -2χ-1=3 ή -2χ-1=-3 Ι α5 - 7α + 6 1 = 0 α5 - 7α + 6 = 0 <=> 2 <=> <=:>-2χ=4 ή -2χ= -2<::>χ=-2 ή χ= 1 α -α=Ο l αz - α! = Ο ii) Ι χ-1 1 = 1 2χ-5 Ι <=>χ-1=2χ-5 ή χ-1= =-2χ+5<=:>-χ= --4 ή 3χ=6 <::>χ=4 ή χ=2 " <=> α5 - 7α + 6 = 0 <=> α5 - 7α' + 6 = 0 <=> α=Οηα=l α ( α - 1) = 0 ίίί) l -3x+4 1 =-6 αδύνατη, διόtι l-3x+4 1 :::ο για κάθε χ Ε IR α5 - 7α+ 6 = 0 ή 15 � 7 · 1 + 6 = 0 <=> α = l <=> iν) Αφού δεν γνωρίζουμε το πρόσημο του 3χ+2. α-Ι α=Ο Διακρίνουμε τις περιπτώσεις: α) Αν 3χ+2<0 δηλαδή χ< -2/3 τότε είναι αδύνατη. Άσκη ση 1 '1 β) Αν 3χ+2:Ξ:Ο δηλαδή χ::: -2/3 τότε Ν α χα ρ ακτηρίσετε με (Σ), Σωστό ή (Λ), Λάθ ος, 1 χ--4 1 =3χ+2<=> χ--4=3χ+2 ή χ--4= -3χ-2 κά θ ε μια από τις παρακάτω προτάσεις και ν α <=:>-2χ=6 ή 4χ=2<::> χ= -3 ή χ=1/2 αιτιο λογήσετε την απάντησή σας. -3 � + άρα η τιμή χ= -3 απορρίπτεται, i) Για κάθε χ,ψ Ε IR , ισχύει: I χ-ψ I + χ-ψ:Ξ:Ο ii) Για κά θ ε α, β Ε IR , ισχύε ι: I α-β I < I α I + I β I 1 . 2 1 , , ενώ 2 Ε - 3 , + οο άρα η τιμη χ = -2 ειναι δεκτη. iii) Για κάθ ε χ,ψ Ε IR , ισχύει, I χ I + I Ψ I =Ο <=> χ=Ο ή ψ=Ο Μια άλλη λύση δίνεται με απαλοιφή του απολύτου iv) Για κάθε α, β Ε IR , ισχύει, I α I =β<:::>α=β ή διακρίνοντας τις περιπτωσεις χ - 4 � Ο και α= -β χ-4�0. Για κάθ ε χ Ε IR , αν ισχύει ν) i)

(

)

[-%, οο)

-

[ ) -

Έ να λάθος που μπορε ί να γ ί νει ε ί ναι να γράψου ­

Ι χ--4 1 =3χ+2<::> χ--4=3χ+2 ή χ--4= = -3χ-2 <::>χ= -3 ή x=l/2 l -2x+1 1 ::; 3 <::> -3 :=:: -2χ + 1::;3 <=> ν) <=> - 4 :::: -2χ :::: 2 <=> 2 ::: χ :Ξ:-1<=:>--- 1 ::; χ:Ξ:2 νί) Ι χ-2 1 >4<::>χ-2<--4 ή χ-2>4<::>χ<-2 ή χ>6 νiί) l 3x+ 1 1 :Ξ:-2 <=> χ Ε IR νiίί) l 4x-1 I ::;Ο <=> l 4x-I I =0<::> <::>4χ-1 =Ο<::>χ= Ι /4

Ι-3 χ+121=3χ-12 τότε θ α ισχύει χ:Ξ:4.

με

Ε φαρμ ογ ή 4'1

iii)

ii)

ίίί)

ν)

Λύση

ii)

i)

iv)

Ν α λύσετε τις ανισώσεις: i) d(x,2) :S 5 και ii) d(2x,l) :S Ο iii) d(α5-7α,-6) :S -d(α2 ,α)

i)

Λύση

d(x,2) ::; 5<::> I χ-2 1 :Ξ: 5<=:>-5:Ξ:χ-2:Ξ:5 <=:>-3:Ξ:χ:Ξ:7 d(2x, Ι) ::; 0<::> l 2x-1 1 :::: 0<::>2 χ-Ι = Ο <::>χ=Ι/2 d(α5-7α,-6):Ξ:-d(α2 ,α) <=>j ιi - Ία+ � � -1 ιi -� <=> <::> Ι α5 - 7α + 61 + Ι α 2 - α! � Ο

Σωστό, διότι Ι α I :::-α για κάθε α Ε IR . Άρα για α= χ-ψ έχουμε: I χ-ψ I :Ξ:-( χ-ψ)::::> 1 χ-ψ 1 + χ-ψ:Ξ:Ο για κάθε χ,ψ Ε IR Λάθος, διότι π. χ. αν α = 5 και β = -3 τότε 1 5-(-3) 1 = 1 5 1 + 1 -3 1 Γενικά για κάθε α,β Ε ΙR ισχύει: I α - β I � I α I + I β I Λάθος, διότι αν χ=Ο, ψ=5 τότε η I ο I + I 5 I =Ο είναι ψευδής και η (0=0 ή 5=0) είναι αληθής Λάθος, διότι για α = 5 και β = -5 η ( 1 5 1 = -5) είναι ψευδής) ενώ η (5 = -5 ή 5 = -(-5)) είναι αληθής. Σωστό διότι για χ=α, συγκεκριμένο, με ι-3α + Ι21 = 3α - Ι2 , οι συνεπαγωγές Ι -3α + Ι 2 1 =3α - 12::::::> -3α - Ι2:Ξ:Ο ::::::> α:Ξ:4 είναι σωστές και η αρχική υπόθεση σωστή. Άρα και το συμπέρασμα σωστό.

Κατά την απόδειξη την ανισότητας Ι α + β ι � Ια Ι + ιβι για κάθε α, β Ε JR γίνεται φανερό ότι το ίσον ισχύει μόνο όταν αβ � Ο καθώς και

Ση μείωση :

ΕΥΚΛΕΙΔΗΣ Β' τ.Ι/27


Μαθηματικά για την Α ' Λυκείου

στην ! α - β ! � ! α! + l β l το ίσον ισχύει μόνο ό­ Ι β Ι 3 τότε: < ταν α ( -β ) � Ο δηλαδή, αβ � Ο . Ά σ κηση 2 η

Λύση

1 l 3 α + 2β αβ + 6

}

<

1

i αi < 2 => Ι α 1 · 1 β Ι <6=> I αβ I <6 <3

Για κάθε α,β,γ,δ Ε IR . Να δείξετε ότι i) Αν α<β και l γ+δ l < l γ l + l δ l τότε αγδ>βγδ. ii) Ι α+ β i + Ια-β Ι = 2 Ι α Ι +2 Ιβl � α·β = ο

,β,

1 I

=>-6<αβ<6=>αβ+6>0 άρα αβ+6*0 , 3 α + 2β < 1 αρκει, να δει'ξουμε Για να δείξουμε οτι Λ ύ ση αβ + 6 i) Σύμφωνα με την προηγούμενη σημείωση: ότι: l 3α+2β I < I αβ+6 1 , ή l 3α+2β 1 2< I αβ+6 1 2 , I γ+δ I < I γ I + l δ l =>γδ<Ο. Άρα α<β=>αγδ>βγδ ή (3α+2β)2<(αβ+6)2 , ii) Γνωρίζουμε ότι για κάθε α, β Ε IR ισχύει: ή 9α2+12αβ+4β2< α2 β2+ 12αβ+36, i α + β l � i αi + l βl ή α2(9-β2)-4(9-β2)<0, ή (9-β2)( α2-4)<0, Ι α - β i � Ι αi + Ι βl η οποία ισχύει διότι: Άρα i α + β i + Ι α - β i � 2 i α i + 2 l β l I α I <2 => I α 1 2<4=>α2-4< Ο Η ισότητα Ι α+ β I + I α - β I = 2 1 α I +2 1 β I ισχύει και Ι β Ι <3=> I β l 2<9=>9-β2>0. μόνον όταν: Λ σ κη ση 5η ( Α πλοπο ί η ση π α ρ αστιΊσr.ων μr. από­ i α + � = l � + l � (Σ) Αλλά (Σ) � αβ � Ο �αβ =Ο λυτ α ) αβ � Ο ! α- � = 1 � + 1 �

}

}

}

)\. σ κ η ση 3 '�

Για κάθε α, β ε IR , να αποδείξετε ότι: l α+β l + l α-β l = l α l + l β l � l α l = l β l

Να γράψετε τις παρακάτω παραστάσεις με μορφή χωρίς το σύμβολο της απόλυτης τιμής i) Α=2χ-Ι-3χ+1 Ι ι + χι ιι χ - (άσκηση 2, σελίδα 4 3 της ii) Β = ι χ Άλγεβρας Α ' Λυκείου ) και να βρείτε το ευρύτερο υποσύνολο του IR * στο οποίο η Β είναι σταθερή.

Ι

ι\ (Jση

Ένας τρόπος για την απόδειξη σχέσεων με απόλυ­ τες τιμές, είναι να υψώνουμε και τα δύο μέλη στο τετράγωνο, εφόσον βέβαια είναι θετικά ή μηδέν ' ' την ιδιοτητα και να χρησιμοποιουμε I χ 1 2= χ 2 . Έ τσι έχουμε: l α+ β l + l α-β l = l α l + l β l � < Ι α+β Ι + Ι α-β Ι )2=( I α Ι + I β I )2 � (α+β)2+2 1 α2-β2 1 +(α-β)2 =α2+2 1 αβ I +β2�2+β2-2 1 αβ I +2 1 α2-β2 1 =Ο� l α l - l β i =O < Ι α i - I β Ι )2+2 1 α2-β2 1 =ο� α 2 - β 2 = 0 -

l

( l α l -l β l 2 � Ο � Ι α l = l β l , αφού 2 2) Ια - β ι � ο Λ σ κη ση

}

1

}

Λ ύ ση :

Για να aπλοποιήσουμε μια παρά­ σταση με απόλυτες τιμές, θα πρέπει να γνωρίζουμε το πρόσημο της παράστασης που βρίσκεται μέσα στο απόλυτο και να εφαρμόσουμε τον ορισμό της απόλυτης τιμής. π.χ. Ι χ2+2ψ2 Ι = χ2+2ψ2 διότι χ2+2ψ2Ξ':Ο για κάθε χ,ψ Ε IR ενώ 1 -α2+2αβ-β2 1 = 1 -(α-β)2 1 = (α-β)2 , διότι -(α-β)2 Ο για κάθε α, β Ε IR . Στην περίπτωση που η παράσταση η οποία βρίσκε­ ται μέσα στο απόλυτο, έχει μεταβλητό πρόσημο, εξετάζουμε το πρόσημό της για τις διάφορες τιμές της μεταβλητής και μετά εφαρμόζουμε τον ορισμό της απόλυτης τιμής, κατά προτίμηση τον (IV) . Π α ρ ατή ρη ση :

s

4'�

Για κάθε α, β Ε IR να δείξετε ότι: Αν I α I <2 και

ΕΥΚΛΕΙΔΗΣ Β' τ. Ι /28


Μαθηματικά για τη ν Α ' Λυκείου

-3χ + Ι 2': 0 � -3χ 2': -Ι � χ � -Ι οπότε: 3 {2χ - (-3χ + Ι) = 5χ - Ι, αν χ � .!_3 Α= 2χ - (3χ - Ι) = -χ + Ι, αν χ 2': -Ι 3 Παρατηρείστε ότι για χ = -Ι οι παραστάσεις 3 5 χ - Ι και - χ + Ι παίρνουν και οι δύο την τιμη, -2 3 ii) Στην περίπτωση αυτή, έχουμε δύο διαφορετι­ κές παραστάσεις μέσα στα απόλυτα, γι' αυτό θα εξετάσουμε το πρόσημό τους και θα χρησι­ μοποιήσουμε συγκεντρωτικό πίνακα. Στο IR. * λοιπόν έχουμε: Ι + χ ;::: Ο � χ Ε [-Ι, Ο) υ (Ο,+ οο) και Ι - χ 2': Ο � χ Ε (-οο,Ο) υ (Ο,Ι] χ - οο -Ι ο 1 + οο + ' + + Ι +χ + Ι-χ + α) Αν χ Ε (-οο,-Ι] τότε: Β = -Ι - χ - (1 - χ) = -2 χ χ β) Αν χ Ε [-Ι,Ο) υ (Ο,Ι] τότε: Β = Ι + χ - (1 - χ) = 2χ = 2 χ χ γ) Αν χ Ε [Ι,+οο) τότε: Β = Ι + χ - (-Ι + χ) = � χ χ χ αν χ Ε ( -οο, -Ι] 2' αν χ Ε [-Ι, Ο) υ (Ο, Ι] Άρα Β = 2, 2 αν χ Ε [Ι,+οο) χ Η παράσταση λοιπόν Β είναι σταθερή στο [-Ι, Ο) υ ( Ο, Ι] και όχι μόνο στο (-Ι, Ο) υ (Ο,Ι) . (Λάθος στο οποίο θα οδηγούνταν οι μαθητές από παρανόηση του ορισμού (II)). Ομοίως η παράσταση Β ισούται με 3_ στο [ l, +οο) και όχι μόνο στο χ (Ι,+οο) . (Λάθος στο οποίο οδηγεί η παρανόηση του ορισμού(!)). Σύγχυση μεταξύ των δύο αυτών ορισμών συμβαίνει όχι μόνο στους μαθητές αλλά -

+

-

και σε πολλά βιβλία. Το ίδιο το Σχολικό βιβλίο εδώ χρησιμοποιεί τον ορισμό (II) αντί του ορισμού (I) που έχει δώσει. Αποφεύγουμε λοιπόν σχετικά λάθη εφαρμόζοντας τον ορισμό (IV). Ση μείωση

Οφείλουμε να παρατηρήσουμε ότι ανάλογο πρό­ βλημα εμφανίζεται και στις γραφικές παραστάσεις συναρτήσεων, χωρίς την έννοια της συνέχειας συ­ ναρτήσεων. π. χ. για τη γραφική παράσταση της f(x) I χ-2 1 +2χ . με τον ορισμό (I) του σχολικού - 2 ' αν χ 2': 2 οπότε βιβλίου έχουμε: f( x ) = {3χ χ + 2, αν χ < 2 δεν είναι φανερό ότι οι δυο κλάδοι της γραφικής παράστασης της f διέρχονται από το σημείο Α(2,4). Με τον ορισμό όμως (IV) έχουμε: - 2 αν χ 2': 2 και αυτό είναι προφανές. f( x ) = {3χ χ + 2, αν χ � 2 =

'

Τέλος εκεί που δεν μπορούμε να αποφύγουμε τον ορισμό (IV) είναι στην Γ Λυκείου για την εύρεση π.χ. του ολοκληρώματος ι ι χ�χ . ψ

χ'

χ

t

ψ' I

Άσκηση 6" Να βρεθούν τα χ Ε JR για τα οποία ισχύει:

2� l -2 x+6 1 �.

Λ ύ ση

Ουσιαστικά έχουμε να λύσουμε το σύστημα των δυο ανισώσεων: 2� l -2x+6 1 (I) και l -2x+6 I �4 (II) (I) � -2χ + 6 � -2 ή -2χ + 6 ?: 2 �

ΕΥΚΛΕΙΔΗΣ Β' τ. Ι /29


Μαθηματικά για την Α ' Λυκείου

17 ή χ = 1 3 Τελικά λοιπόν: Α = 4 <:::> χ = . 5 (Π)<=> -4 :5 -2χ+6:54 <:::> -4 -6 :::; -2χ+6 -6:5 4- 6<:::> Εντελώς ανάλογα: α) Στο (-οο,3] έχουμε: -10:5 - 2x:S: - 2<:::> 5 2: χ 2:1 <::> I :::; χ :::; 5. Α < 4 <=> -χ + 9 < 4 <=> -χ < -5 <=> χ > 5 Πρέπει να συναληθεύσουμε τις δυο ανισώσεις: οι τιμές αυτές απορρίπτονται διότι δεν ανήκουν στο (-οο,3]. -(1) ( Ι )(Π) β) Στο [3, 5] έχουμε: 17 χ' ο 2 4 5 χ Α < 4 <:::> -5χ + 21 < 4 <:::> -5χ < -1 7 <:::> χ > 5 Άρα 2 s l -2x + 61 s 4 <:::> χ Ε [Ι, 2 ] υ [4,5 ] <:::> Χ Ε ·; , 5 -2χ :5 - 8 ή -2χ 2: -4 <=> χ2:4 ή χ :5 2

ι

Ά σκ η σ η 7ΙJ

( ]

-

-

Να απλοποιηθεί η παράσταση : Α= 3 1 χ 5 1 -2 1 3 χ I και να λυθ εί η εξίσωση : Α=4 και η aνίσωση Α<4

γ) Στο [5, +οο) έχουμε: Α < 4 <=> χ - 9 < 4 <=> χ < 13 <=> χ Ε [ 5, 1 3 ) - οο

3

Τελικά Α < 4 <=> χ Ε

Λύση

Για τη λύση μιας εξίσωσης ή μιας ανίσωσης, στην οποία οι παραστάσεις που βρίσκονται μέσα στα απόλυτα είναι διαφορετικές, πρέπει να κάνουμε πίνακα προσήμων αυτών των παραστάσεων να λύσουμε την εξίσωση ή την ανίσωση στα επιμέ­ ρους διαστήματα του πίνακα και να εξετάσουμε αν οι τιμές που βρήκαμε ανήκουν στα αντίστοιχα δια­ στήματα. Όπως και στην άσκηση 5 βρίσκουμε: -χ + 9 αν x s 3 Α = -5x + 2 l αν 3 s x s 5 χ - 9 αν χ � 5 Οπότε: α) Στο ( -οο,3] έχουμε: Α = 4 <=> -χ + 9 = 4 <:::> χ = 5 η τιμή αυτή απορρί­ πτεται διότι δεν ανήκει στο (-οο,3]. β) Στο [3, 5] έχουμε: Α = 4 <=> -5χ + 21 = 4 <=> χ = .!2 5 Δεκτή τιμή αφού ανήκει στο [3, 5]. γ) Στο [5, +οο) έχουμε: Α = 4 <:::> χ - 9 = 4 <:::> χ = 13 Δεκτή τιμή αφού ανήκει στο [5, +οο).

{

1 7/5

5

13

(ι; , 5J υ [ 5, 13) <=> χ (ι; , 1 3) Ε

Ρ Ι ΖΕΣ Π Ρ Α ΓΜ Α τ t ΚΩ Ν Α Ρ Ι ΘΜΩΝ

Για κάθε α 2 Ο και ν Ε Ν * , αποδεικνύεται ότι υπάρχει ένα μόνο χ 2 Ο τέτοιο ώστε χν=α και συμβολίζεται με χ = � , δηλαδή:

( ]

Χ = � � Χν = . ορσ χ 2 0 α

Άμεση συνέπεια αυτού είναι η ισότητα:

Άσκ η ση 8'1

Να χαρακτηρίσετε με (Σ), Σωστό ή (Λ) Λάθος τις παρακάτω προτάσεις και να αιτιολογήσετε την απάντησή σας. .J25 = -5 ί) ίί)

�(-2) 2 = -2

ίίί)

Για κάθε α ε JR ισχύει

iv)

Για κάθε α � Ο ισχύει

ν)

Υπάρχει α Ε ffi. με

Ν=α,

( .rαγ = α

if;;! = � α � Ο ισχύει � = α ,

vi) Για κάθε vii) Για κάθε α, β Ε ffi. ισχύει:

ΕΥΚΛΕΙΔΗΣ Β ' τ. t /30


Μαθηματικά για την Α · Λυκείου

J;! = Jii <::> α = β ,

viii) Για κάθε α, β Ε IR , με α β � Ο ισχύει: ix) χ)

Jεψ = vfai.Jιii

Για κάθε α � Ο ισχύει .J4α * -2-.Γα Για κάθε α < Ο, β > Ο ισχύει

α1β = - -α3β

ii)

3) α= -2 και β = 5 � Η ισότητα α 2 + β 2 = α + β , με α,β Ο ισχύ­

ει: 1 ) πάντοτε 2) μόνο όταν α = 3) μόνο όταν α = Ο ή β = Ο

Ο

και β =

Ο

Λύση

i) Ι και 2, διότι (1) => α = 2 και β = -5 => xi) =:> α = 2 ή β = -5 α Ε IR ώστε να ισχύει αifi = -� . ii) 3, διότι �α 2 + β 2 = α + β <=> ( 2 + β 2 ) 2 = (α + β) 2 <=> α 2 + β 2 = α2 + 2αβ + β 2 α � Λύση i) Λάθος, διότι η τετραγωνική ρίζα ενός θετι­ <=> 2αβ = Ο <=> α = Ο ή β=Ο. κού αριθμού α, είναι η μη αρνητική λύση της εξίσωσης χ2=α. Άρα J25 = +5 Άσκη σ η 1 0'1 ii) Λάθος, διότι για το λόγο που προαναφέραμε Να λύσετε τις εξισώσεις: i) � = .J3 - 4x ισχύει: �( -2 ) 2 = 1-21 = +2 ii) .,/9 χ ' 6 χ + 1 + χ 2 ο iii) Λάθος, π.χ. αν α = -3 τότε �( -3 )2 = .J9 = 3 * -3 . Γενικά για κάθε Λύση α Ε JR ισχύει: Ν = Μ = lαl i) Για να ορίζεται η εξίσωση πρέπει και αρκεί: 3χ - 2 � ο 2 3 ίν) Σωστό, διότι α � Ο -� χ �- . δηλαδή , 3 - 4χ � Ο 3 4 ν) Σωστό, π. χ. για α = 5 , αλλά και για κάθε Τότε έχουμε: α � Ο. 5 νί) Σωστό, διότι α �0<=> α�Ο. .J3x -2 = .J3 -4x <=> {.J3x - 2 ) 2 = {.J3 -4x ) 2 <=> νίί) Λάθος, διότι αν π.χ. α=3 και β= -3 τότε η J3i = �( -3 2 είναι αληθής ενώ η 3 = -3 εί- <=> 3χ - 2 = 3 - 4χ <=> 7χ = 5 <=> χ = -5 . ) 7 2 ' 5 ' Η ναι ψευδής. Γενικά ισχύει: ' - � -5 � -3 . τιμη χ = - ειναι δεκτη' διοτι 3 7 4 7 Ν = # <=> Ι α l = Ι β l <=> α = ± β , 2 Ι 5 3 5 (αφου - - - = - > 0 και - - - = - -Ι < 0 ) νίίί) Σωστό, διότι 7 4 28 7 3 2Ι � = νfαβί = νraϊίί3ί = νfαί ν1βί ii) Για να ορίζεται η εξίσωση πρέπει και αρκεί 9χ2 - 6χ + Ι � Ο πράγμα που ισχύει για κάθε ix) Λάθος, αφού για α=Ο είναι ίσα. χ Ε JR , αφού 9χ2 - 6χ + Ι = (3χ - Ι)2 � Ο. χ) Σωστό, αφού o@=-{-α)<ffi =�=�, Άρα καθόσον -α > Ο . .J9x 2 - 6χ + Ι + 7χ - 2 = 0 <=> x i ) Λάθος διότι μόνο για α=Ο ισχύει. �( 3χ - Ι ) 2 + 7χ - 2 = 0 <=> l3x - II + 7x - 2 = 0 Υπάρχουν του λάχιστον δύο τιμές του

7

-

-

=

}

.

Άσκηση 9η Ποιες από τις παρακάτω προτάσεις είναι σω­ στές και γιατί; i) Για κάθε α Ε [ 2, + οο ) και β Ε [ -5, + οο ) αν ισχύει Vα - 2 + �β + 5 = 0 (Ι) τότε θα ι­

σχύει: 1 ) α = 2 ή β = -5 2) α = 2 και β = -5

α) Αν χ Ε

( j] τότε -οο ,

(I) <=> ( -3χ + Ι) + 7χ - 2 = Ο <=> χ = .!. δεκτή 4 ' '4Ι Ε -οο, '3 τιμη' διοτι

ΕΥΚΛΕΙΔΗΣ Β' τ.1/3 1

( 1]


β) Αν χ

ε

Μαθηματ ικά για την Α· Λυκείου

[�, + ) τότε

= Ι Ι - 2J3 1 = 2J3 - Ι , αφού 2J3 > Ι .

οο

<::::> ( 3 χ - Ι) + 7 χ - 2 = Ο <::::> χ = 2_ Η τιμή 10 3 Ι αυτη. απορριπτεται διοτι . . 10 -3 , + Τελικά .J9 x2 - 6χ + Ι + 7χ - 2 = Ο <::::> χ = ..!_ 4 (I)

[

οο

)

Λ σ κ η ση 1 3 '�

Να μετατρέψετε τα παρακάτω κλάσματα σε ι­ σοδύναμα κλάσματα με ρητό παρονομαστή. 2 αβ α) β) , co ' αβ > Ο ,ναβ � -

ι α :;e Ο �α2 ι �' α>Ο , r---:; ,

γ)

λ (F Κψ:J η 1 I ' 1

Αν Ι χ Ι <3 τότε να απλοποιηθεί η παράσταση :

.J4x 2 - 24χ + 3 6 .Jx 2 + 6χ + 9 + Α= χ+3 χ-3

ε)

δ) ζ)

α-3 ι-;; , α > 3 να - 3 2J3 + ιο 2J3 - ιο

----

/

Λίιση

α) Αν ο παρονομαστής είναι της μορφής W με α>Ο, ν>κ, τότε πολ/με τους όρους του κλάσματος με την παράσταση vαν-κ έτσι ώστε ο παρονομαστής να γίνει W = α , δηλαδή 2 = 2ifiί = 2 if24 = 2 if24 = J.fϊ6 2 7

�·. ��; ιυ Ί�-�

( χ - 3-)2 + �( χ + 3)2 = 2 lx -3l )χ + 3ι Α �..--4χ--3χ+3 χ -3 χ +3 = -2(χ - 3) + χ + 3 = -2 + Ι = -Ι , επειδή χ-3 χ+3 χ -3 <0 lxl < 3 � -3< x < 3} � χ+3>0

}

ifi3 ifi3 if24

αβ αβ�α2β2 = αβ�α2β2 = αβ�α2β2 = β) αβ � ��α2β2 ψαβγ

Ά σ κ η (ιη 1 2 '�

Ν ' απλοποιηθούν οι παραστάσεις: Α=

��α3 �α 2 Ν , Β = � ,

�α2 β 2 Ι I{[af {[af = �α� {[af \fσf = {[af w= =

Γ = {Γσ!θ

Δ = �1 3 - 4J3 ΛiΗ'Η!

Εφαρμόζοντας τις ιδιότητες α4β = � , ffα = μ� και μ� = � με α,β :::: Ο έχουμε: Για να ορίζονται οι Α, Β πρέπει και αρκεί α ::=:: Ο . Τότε Α = �α3 �.Jα4 α2 = �α3 � = Wa = � = -Γα Β = �α5·4+3 = � · W = α4W . Η παράσταση Γ όμως ορίζεται για κάθε α R οπότε Γ = νfar = �ι αι 7·S +) = Ια1 5 fαr αφού πι­ θανόν να είναι α<Ο . Χρειάζεται λοιπόν προσοχή όταν ο εκθέτης του υπόριζου είναι άρτιος. Δ = �Ι2 + ( 2 J3 ) 2 - 2 · Ι · 2J3 = �( Ι - 2 J3 )2 = ....----:-= -

ε

if2ί

'

γ) δ)

Μ

α- 3 ( α- 3)νΓα"=3 - ( α- 3)νΓα"=3 .Jα- 3 .Jα- 3 -Jα- 3 �( α- 3) 2 _

_

=

= (α - α3)--Γα="3 = �α - 3 3

1 ε) �

=

� - � -� α2 � α2 �� α2 if;! αJ 1

=

ή

Ι {Γα! {Γα! συντομοτερα 1 Γ7 = 1 ΓQ = . ν α7 να9 α3 ζ) Όταν ο παρονομαστής είναι της μορφής � ± .Jβ , πολ!ζουμε και τους δυο όρους του κλάσματος με τη συζυγή παράσταση δηλαδή -Γα + Jβ , έτσι ώστε να δημιουργηθεί η ταυτότητα: (α + β)(α - β) α2 - β2 Άρα:

ΕΥΚΛΕΙΔΗΣ Β ' τ. Ι /32

=


Μαθηματ ικά για τη ν Α ' Λυκείου

2 2ν'3 + 10 ( 2ν'3 + 10)( 2ν'3 + 10) ( 2ν'3 +10) = 2ν'3 -10 ( 2ν'3 -10)( 2ν'3 + 10) ( 2ν'3 )2 -102 ( 2../3 ) 2 + 2 · 2../3 · 10 + 102 12 +40../3 + 100 = = 12 - 100 88

i) 2χ6=32χ2 ii) χ4=-27χ iii) χ4+256=0 ίν) (x+l)3= -64 ν) (2χ+4)4=81 vi) χ4=α6 vii) χ3=λ-2

iii) χ 4 + 256 = Ο <=> χ 4 = -256 αδύνατη. 3 ίν) { χ + 1 ) = -64 <=:> χ + 1 = --V'64 <=> χ + 1 = -4 <=> χ = -5 ν) { 2χ + 4 ) 4 = 8 1 <=:> 2χ + 4 = 3 ή 2χ + 4 = -3 <=> 2χ = -1 ή 2χ = -7 <=> χ = _ .!. ή χ = _ 7._ 2 2 νί) χ 4 = α 6 <=> χ = ± W <=:> χ = ± Μ <=:> x = l α iM ή χ = - Ι α Ι Μ · νίί) αν λ - 2 � Ο , δηλαδή, λ � 2 , τότε χ 3 = λ - 2 <=> χ = �λ - 2 αν λ - 2 < Ο , δηλαδή, λ < 2 , τότε χ 3 = λ - 2 <=> χ = -�2 - λ

Λ ίJ ση

Άσκηση

1 1 2 + 40

.J3

88

=

14 +

s.J3

11

Άσκηση 1 4'1

(Λύση εξισώσεων της μορφής χν=α) Να επιλύσετε τις εξισώσεις:

Έχουμε μάθει στα προηγούμενα μαθήματα ότι για να λύσουμε μια εξίσωση πρέπει να δούμε τι βαθ­ μού είναι. Εάν είναι πρωτοβάθμια χωρίζουμε γνω­ στούς και αγνώστους, Εάν όμως είναι από 2 ου βαθ­ μού και πάνω εξετάζουμε αν είναι στη μορφή χν=α ( 1 ) . Εδώ υπάρχουν οι εξής περιπτώσεις: i) Αν α=Ο τότε (1) <=:>χ=Ο ii) Αν α>Ο και ν άρτιος τότε (1) <=:> χ = ± ifO. (2 λύσεις αντίθετες) iii) Αν α>Ο και ν περιττός τότε ( 1) <=> χ = ifO. (1 λύση θετική) ίν) Αν α<Ο και ν άρτιος τότε είναι αδύνατη ν) Αν α<Ο και ν περιττός τότε ( 1) <=> χ = -� ( 1 λύση αρνητική) Στην περίπτωση όμως που δεν είναι έτοιμη η εξί­ σωση στη μορφή χν = α, τότε φέρνουμε όλους τους όρους στο α ' μέλος έτσι ώστε το β ' μέλος να είναι μηδέν, παραγοντοποιούμε και εφαρμόζουμε την αβ = Ο <=> α = Ο ή β = Ο. Άρα: i) 2χ 6 = 32χ 2 <=> 2χ 6 - 32χ 2 = ο <=> <=:> 2χ 2 ( χ 4 - 16 ) = 0 <=:> χ 2 = 0 ή χ 4 - 1 6 = 0 <=:> χ = Ο ή χ 4 = 1 6 <=:> χ = 0 ή χ = � ή χ = -� <=> χ Ε { Ο,- 2, 2 } ίί) Χ 4 = -27χ <=> χ 4 + 27χ = ο <=> χ ( χ 3 + 27 ) = 0 <=:> χ = 0 ή Χ 3 = -27 <=:> χ ο ή χ = -ifii <=> χ = ο ή χ = -3 . =

! 5'1

Για κάθε ν ε ifi + ifi > ifS

Ν

με ν � 2 ν α δείξετε ότι:

Λ\Jση

ο < χ < 1 � χ κ < 1 � χ κ+μ < χμ ' για κάθε κ ' μ

ω ω οπότε 'm δή Η Jf Ομοίως JI Ji

Άρα

.

Ε

Ν *

'm δηλα­

Άρα: ν {Ι + vfi � {Ι + fi Αλλά � + {i > 1 , vs vs \15 \15 vs vs 2 αφού ( .J2 + .J3 ) =5+2J6 >5� � fi + ν'3 > J5 � Οπότε:

Η + )Ι > ι

Jf + Ji > Ι, δηλαδή ifi + if3 > if5

Αν θεωρηθεί γνωστό το ανάπτυγμα του διωνύμου του Νεύτωνα, τότε η παραπάνω aνί­ σωση αποδεικνύεται ευκολότερα ως εξής: Γνωρίζουμε ότι για κάθε α,β>Ο έχουμε: Π α ρ ατή ρ η σ η :

Αν α = ifi και β = if3 τότε: ( 42 + 43 )" > 42-ν + 43ν = 2+3 = 5 � 42 + 43 > :15

ΕΥΚΛΕΙΔΗΣ Β ' τ. l /33


Μαθηματικά για την Α ' Λυκείου

Άσκ ησ η 1 6'1

Αν

χ+

x,y θετικοί α ριθμοί να αποδείξετε ότι: y � �χ 2 y 2 + 2 FY - JhY (I) και να βρεί­ +

τε πότε ισχύει το ίσον

(

)

(x - I ) .J2x - x2 + � + l = o � χ = Ι, αφού .J2x-x2 + � + Ι > Ο Π ρό β λη μα

Δίνετ αι ορθογώνιο τρίγωνο ΑΒΓ

Λύ ση

Υψώνουμε και τα δυο μέλη στο τετράγωνο, εφό­ σον είναι μη αρνητικά και προσπαθούμε να την αναγάγουμε σε μια σχέση που ισχύει: (I) � χ + Υ + JbY � �χ 2 + y 2 + 2.,ΓχΥ � [ (χ + Υ) + .fbYT � �χ 2 + y 2 + 2.,ΓχΥ τ � (χ + Υ ) 2 + 2( χ + y )J2xy + 2xy � χ 2 + y 2 + ___ +4�χ 2 + y2 FY + 4xy � J2 (χ + Υ) � 2J,...-x2 + y2 � 2χ2 + 2y 2 + 4xy � 4χ 2 + 4y 2 � � 2xy � x2 + y2 � χ2 +I -2xy2:: 0 � ( x-y)2 2:: 0 που ισχύει για κάθε x,y>O . Η ισότητα ισχύει μόνο όταν (χ - ψ)2 = Ο, δηλαδή χ = ψ

[

Άσκη ση 1 7'1

Θεωρούμε το κλάσμα Α

= .J2x -

(Α= 90°)

Αν

τα μήκη των πλευρών του ΑΒ και ΒΓ είν αι ίσα με κάποιου ς απ' τους αριθμούς - I α2-2 1 , α2+2 , I α+2 1 - l -2-α I , α2-2, να βρείτε α) τ α μήκη των πλευ ρών του β) το εμβ αδόν του τριγώνου Λύ ση

Α) Τα μήκη των πλευρών του είναι θετικοί αριθ­ μοί. Αφού - l α2-2 I �O και l α+2 1 - l -2-α l = = Ι α +2 1 - 1 α+2 1 =Ο για κάθε α Ε 1R , αποκλείεται αυτοί να είναι μήκη πλευρών Γ

2 2χ - 3χ + l χ2 χ2 -

α) Για ποιες τιμές του χ ορίζεται; β) Ν α λυ θεί η εξίσωση Α= 1 - χ

.Jι -

Β

Παρατηρούμε εξάλλου ότι: 2 - 2 � Ο � α2 � 2 � Ι αl 2 � 2 � α ση Λύ Ι αΙ � J2 � -J2 � α � J2 α) Για να ορίζεται η Α πρέπει και αρκεί: Αν λοιπόν α ε [ .J2 .J2 ] τότε και ο αριθμός α2-2 2χ -χ2 2:: 0(1), l - x 2 2:: 0(2), .J2x - x2 - � :;t O (3) αποκλείεται να είναι μήκος πλευράς Αν όμως α2 - 2 > Ο δηλαδή α > J2 ή α < -J2 . (Ι) � χ2 - 2χ � ο � χ(χ - 2) � ο � Τότε ( ΑΒ ) = α 2 - 2, ( ΒΓ ) = α 2 + 2 αφού ΒΓ> ΑΒ. Εφαρμόζουμε το Πυθαγόρειο θεώρημα και έχου­ �ο ή 2:: ο � χ Ε [0, 2] . Ομοίως ( 2) � χ Ε [ -1, Ι] . Οι ( 1 ), (2) συναληθεύ­ με: ( ΑΓ)2 = ( ΒΓ)2 - ( ΑΒ)2 = ( α2 + 2 / - ( α 2 - 2 / = ουν στο [0,1], οπότε: = ( α2 + 2 + α2 - 2 )( α2 + 2 - α 2 + 2 ) = 2α2 · 4 = 8α2 (3) �2χ-χ2 :;t}-x2 �x:;t �x E Ο, Άρα ( ΑΓ ) = J&1 = J8.N = 2J2Ι α l > Ο αφού (2χ-Ι)(χ-ι)(� +Μ = Ι αΙ > J2 . β) A=l-x� -{ ) 1-χ � 2x-i 1-i β · υ (ΑΒ)(ΑΓ) ( α 2 - 2) · 2J2i α l = = = Ε β) 2 2 2 (x - l ) .J2x - x 2 + � + ( χ - 1) = 0 � 2 = J2Ι α l ( α - 2) . -

,

(:�� ) (:�� )

� [ �)υΘ, ι] )

(

)

....:.____ ...___ _ _:_ _

ΕΥΚΛΕΙΔΗΣ Β ' τ.t/34


Μαθηματικά για την Α ' Λυκείου

.J2x - x2 - .Jx2 - 4 = 3x - λ

Άσκ ηση 1 8'1

Να λυθεί η εξίσωση: 2 ( χ2 + 2χ) 2 + 2 1 χ2 + χ - 21 + 5Jx3 - 6χ - 4 = 0

(1),

όπου λ παράμετρος (1)

Λύ ση

Για να ορίζεται η (Ι) πρέπει και αρκεί χ3 - 6χ - 4 :=:: Ο (2). Τότε έχουμε: --3 ( χ 2 + 2χ ) 2 + 2 Ι χ 2 + χ - 2 1 + 5 .J'"x.,--3- 6χ - 4 = ο <=:> { χ 2 + 2χ ) 2 � 0 χ 2 + 2χ = Ο (3) χ : + χ - 2 = ο (4) , αφού Ι χ 2 + χ - 2 1 � ο χ - 6χ - 4 _- Ο (5) .Jx 3 _ 6χ _ 4 � ο ( 3 ) <=> χ ( χ + 2 ) = 0 <=:> χ = 0 ή χ = -2 Παρατηρούμε ότι οι (4) και (5) επαληθεύονται μό­ νο για χ= -2, η οποία είναι δεκτή, διότι για χ= -2 έχουμε χ3-6χ--4 = 02:0, δηλαδή ισχύει η (2). Άρα (Ι) <=:> χ=-2

}

Άσκ η ση 1 9'1

Λύ ση

Για να ορίζεται η (Ι) πρέπει και αρκεί 2χ - χ 2 � ο (2) (3) χ2 - 4 � 0 Έχουμε: ( 2 ) <=:> χ ( χ - 2 ) � Ο <=:> χ Ε [ Ο, 2 ] . ( 3 ) <:::> x z � 4 <=> l x l z � 4 <:::> <=:> l x l � 2 <=:> χ Ε ( -οο,-2 ] υ [ 2,+οο ) - οο χ

'

(3 )

-2

ο

1

1

(2 )

!

:

+οο

2

χ

(3)

Επομένως η εξίσωση (Ι) ορίζεται μόνο για χ=2. Για να επαληθεύεται για χ=2 πρέπει και αρκεί JO - JO = 6 - λ, δηλαδή λ = 6 Άρα όταν λ :j:. 6 είναι αδύνατη, ενώ όταν λ = 6 έχει μοναδική ρίζα την χ = 2.

Ν α λυθεί η εξίσωση:

Γεω μετρία

Ευθ ύ γραμμα Τ μή ματα

Σ αμπ άς Θ εό δω ρος

ΑΚ=ΑΔ-ΔΚ=α+β+γ-δ ( I ) Για να έχει λύση, Σε μια ευθεία (ε) παίρνουμε διαδοχικά σημεία πρέπει να ισχύει: Α, Β,Γ, Δ ώστε: ΑΒ=α, ΒΓ=β, ΓΔ=γ. ΒΔ<ΔΚ<ΑΔ <=:> β+γ<δ<α+ β+γ (2) i) Να βρείτε σημείο Κ εσωτερικό του ΑΒ ώ­ ίί) ΔΛ=ΑΔ-ΑΛ=α+ β+γ-δ (3) Για να έχει λύση, στε: ΔΚ=δ. Ποια συνθήκη πρέπει να ικανο­ πρέπει να ισχύει: ποιείται ώστε να έχει λύση το πρόβλημα; ΑΓ<ΑΛ<ΑΔ <=:> β+γ<δ<α+ β+γ (4) ii) Όμοια να βρείτε σημεί Λ εσωτερικό του ΓΔ iii) Θα πρέπει να ισχύουν ταυτόχρονα οι συνθήκες ώστε: ΑΛ=δ. Ποια συνθήκη πρέπει να ικα­ (3) & (4) <=:> maχ{β+γ,α+ β }<δ<α+ β+γ ΚΛ=ΑΔ - ΑΚ - ΛΔ = νοποιείται ώστε να έχει λύση το πρόβλημα; iii) Βρείτε τη συνθήκη πρέπει να ικανοποιείται = α+ β+γ - (α+ β+γ - δ) - (α + β + γ - δ) ΚΛ = α+ β +γ-α-β-γ+ δ-α-β-γ+ δ= ώστε να ισχύουν οι ερωτήσεις Ι & 11. Στη 2δ-(α+ β+γ) συνέχεια υπολογίστε το μήκος του τμήματος (5) Άσκηση 1 '1

ΚΛ.

Ε

ί)

(α,β,γ,δ γνωστά μήκη)

Λύ ση Α

κ

Β

Γ

Λ

Δ

Η (5) έχει νόημα γιατί: δ > β + γ => 2δ > α + + + > α + + β γ β γ β δ>α+β

ΕΥΚΛΕΙΔΗΣ Β' τ. Ι/35

}


Μαθη ματικά για την Α· Λυκείου

Α

ί\σκηση 211

Επί ευθείας χ ' χ, δίνονται τέσσερα διαφορετικά ση μεία Α,Β,Γ,Δ με οποιαδήποτε σειρά. Θεωρούμε τις προτάσεις: p: «τα ευθύγραμμα τμήματα ΑΒ, Γ Δ έχουν κοι­ νό μέσο>>. q: «ΑΓ=ΒΔ>> i) Δείξτε τη συνεπαγωγή p=> q ii) Ισχύει η συνεπαγωγή q=> p ; iii) Πώς πρέπει να διαμορφωθεί η υπό θεση, ώστε να ισχύει η ισοδυναμία: p<::>q /\1:Jση

Α

Ο

Β

Δ

,__�- --Ι----- ----Ι---------.----1---τ--

Γ

ί)

-

χ· χ Έστω τα ευθύγραμμα τμήματα ΑΒ, ΓΔ & Ο το κοινό μέσον τους αν ΓεΑχ' τότε: ΟΓ>ΟΒ άρα ΔεΒχ Για το σχήμα (Σ): ΓΑ=ΟΓ-ΟΑ=ΟΔ-ΟΒ=ΒΔ όμοια, όταν ΔεΒχ Αν Γ είναι εσωτερικό σημείο του ΑΒ, τότε: ΟΓ<ΟΑ &ΟΔ<ΟΒ. Έχουμε το σχήμα (Σ') Α

Ο

Β Γ Δ -4--l------t------------1--1x

'

χ

Έτσι: ΑΓ=ΟΑ - ΟΓ=ΟΒ - ΟΔ=ΒΔ Σε κάθε περίπτωση ισχύει η συνεπαγωγή p=>q ίί) Στο σχήμα Σ", έχουμε ΑΓ=ΒΔ, όμως τα ευθύ­ γραμμα τμήμτα ΑΒ & ΓΔ δεν έχουν κοινό μέσο. Α Γ Ο ---1Δ--Β---1 ----ι-----1 χ' !Ί ρ ιiγματ ι : αν Ο είναι κοινό μέσο των ΑΒ & ΓΔ τότε: ΟΓ>ΟΑ=> ΟΔ>ΟΒ (άτοπο) Άρα δεν ισχύει η συνεπαγωγή q=> p ίίί) Η υπόθεση πρέπει να διαμορφωθεί ως εξής: «Δίνονται τα σημεία Α,Β,Γ,Δ της ευθείας χ'χ ώστε Γ, ΔεΑΒ ή Γ,Δίι!ΑΒ, τότε ισχύει η ισο­ δυναμία p<::>φ>. Π ρfη μ ατ ι : Το πρόβλημα υπάρχει με τη συνε­ παγωγή q=>p. Έχουμε τις περιπτώσεις, των σχημάτων: ΑΓ=ΔΒ & ΟΓ=ΟΔ (Ο μέσο του ΓΔ) ----e

--χ

Γ

ο

Β

Δ

-�·�----�---+�

χ' χ Άρα ΑΓ+ ΟΓ=ΔΒ +ΟΔ=>ΟΑ=ΟΒ, δηλαδή Ο μέσο του ΑΒ. Όμοια και για το σχήμα (Σ2) Β ο Α χ' Γ Δ χ

ΑΓ=ΒΔ & ΟΑ=ΟΒ (Ο μέσο του ΑΒ) Άρα ΑΓ+ ΟΑ+ΟΒ +ΒΔ => ΟΓ=ΟΔ, δηλαδή Ο μέσο του ΓΔ Έτσι, ισχύει η συνεπαγωγή q=> p Λ σ κηση 3 '1

Δίνεται ευθεία χ ' χ, στην οποία παίρνουμε τα σημεία Α, Β 'ώστε ΑΒ=α (α γνωστό μήκος) Να βρείτε τα σημεία Μ, Ν της ευθείας χ ' χ, ώ­ στε: ΜΑ-ΜΒ= ΝΑ-ΝΒ Λ ίJση

Επειδή ΜΑ-ΜΒ & ΝΑ-ΝΒ?::Ο , δηλαδή ΜΑ?::ΜΒ & ΝΑ?::ΝΒ (1) Έχουμε: χ'

Α

ο

Β

χ

Τα Μ, Ν θα ευρίσκονται στην ημιευθεία Οχ' (Ο το μέσο του ΑΒ), γιατί μόνο τότε ισχύουν οι συνθή­ κες (1). Αν το Μ συμπίπτει με το Ο, τότε: ΜΑ-ΜΒ & ΝΑ-ΝΒ=Ο. Επομένως και το Ν συμπίπτει με το Ο, συνεπώς τα Μ, Ν συμπίπτουν με το Ο. Αν το Μ ανήκει στην ημιευθεία Βχ', τότε: ΜΑ-ΜΒ =ΑΒ <=> ΝΑ-ΝΒ=ΑΒ, άρα και το Ν α­ νήκει στην ημιευθεία Βχ'. Είναι προφανές ότι τα Μ, Ν δεν συμπίπτουν υποχρεωτικά, έχουμε απει­ ρία ζευγών σημείων Μ, Ν (κάθε ζεύγος σημείων της ημιευθείας Β χ'). Απομένει, τα Μ, Ν να είναι εσωτερικά σημεία της ΟΒ. Τότε έχουμε:

ΕΥΚΛΕΙΔΗΣ Β' τ. l/36


Μαθηματικά για την Α ' Λυκείου

ΜΑ+ΜΒ=ΝΑ+ΝΒ� fJ1(. +ΟΜ+ΟΜ- )dB = ΟΑ+ΟΝ+ ΟΝ - )dB �ΟΜ=ΟΝ� Μ, Ν συ­ μπίπτουν

ΜΑ-ΜΒ = ΝΑ-ΝΒ <::::> (ΟΑ+ΟΜ) -(ΟΒ-ΟΜ)=(ΟΑ+ΟΝ) <::::> ΟΜ=ΟΝ Άρα τα σημεία Μ, Ν συμπίπτουν.

Ι η Λ υ κείου ( τ Ο Ξ Α )

Ά σκη σ η 4 '�

Δίνεται ευθεία χ 'χ, στην οποία παίρνουμε τα σημεία Α, Β 'ώστε ΑΒ=α (α γνωστό μήκος) Να βρείτε τα σημεία Μ, Ν της ευθείας χ ' χ, ώ­ στε: <�

χ'

χ

--+----.----+---

0

Α

Β

Ά σ κη ση 5 η

Δίνεται κύκλος (O,R) και τα διαδοχικά σημεία Α, Β, Γ, Δ ώστε: -- -- -ΑΒ + ΒΓ + Γ Δ < 360° Αν είναι α · ΑΒ = β · Β Γ (α, β γνωστοί θετικοί), τότε: --ί) Ν α εκφράσετε το ΒΔ σε σχέση με τα ΑΔ και ΓΔ ίί) Αν είναι ΑΟΓ = 90° , τότε να βρείτε τα τόξα ---ΑΒ και ΒΓ ίίί) Αν τα σημεία Α, Γ είναι aντιδιαμετρικά να ---βρείτε τα τόξα ΑΒ και ΒΓ

ΜΑ2:ΜΒ & ΝΑ2:ΝΒ (1), για να έχει νόη­ μα η σχέση που δίνεται προς απόδειξη. Τα σημεία Μ, Ν ανήκουν στην ημιευθεία Οχ ' (Ο το μέσο του ΑΒ), τότε μόνο ισχύουν οι σχέσεις (1). i) Αν το Μ συμπίπτει με το Ο, τότε και το Ν συ­ Λύσ η μπίπτει με το Ο. Γιατί: ΜΑ2-ΜΒ2=0<::::> ΝΑ2- i) ΒΔ = ΑΔ - ΑΒ ΝΒ2=0 ΒΔ = ΒΓ στην συνέχεια: + ΓΔ ------ii) Λν \I είνω �;σωτr.ρ ι κό το υ Ο Β , τότε: α · ΒΔ = α · ΑΔ - α · ΑΒ -ΜΑ - ΜΒ < ΑΒ � ΜΑ 2 - ΜΒ 2 < ΑΒ2 (2) + β · ΒΔ = β · ΒΓ + β · ΓΔ ΜΑ + ΜΒ = ΑΒ ( α + β) Μ = α · Μ + β · fΔ και το Ν είναι εσωτερικό του ΟΒ, σε άλλη πε­ --Γιατί: β · ΒΓ = α · ΑΒ ρίπτωση (δηλαδή Ν Ε Βχ ' ), έχουμε: ΝΑ - ΝΒ = ΑΒ � ΝΑ 2 ΝΒ 2 - � ΑΒ 2 (3) ΝΑ + ΝΒ � ΑΒ από (2) & (3) καταλήγω σε άτοπο (ΑΒ2<ΑΒ2) Έτσι: (ΜΑ-ΜΒ)(ΜΑ+ΜΒ)= (ΝΑ - ΝΒ )(ΝΑ + ΝΒ) � (ΜΑ - MB) .}df = (ΝΑ - ΝΒ) ΑΒ� ΜΑ - ΜΒ=ΝΑ - ΝΒ(ΟΑ+ΟΜ)--(ΟΒ - ΟΜ)= (ΟΑ+ΟΝ)-(ΟΒ-ΟΝ) � ΟΜ=ΟΝ άρα τα Μ, -Ν συμπίπτουν. ..---Δ = -α · ΑΔ---'+ β· ΓΔ- (1) τελικα: , Β α+β iii) Λ το Μ Β τ6τ:; κ α ι το :\! Ε Β χ ' (σύμφω­ ii) ΑΟΓ = 90° � ΑΓ = 90° . να και με την (11)) Έχουμε: Έτσι: (ΜΑ-ΜΒ)(ΜΑ+ΜΒ)= AB + Br = 90° (ΝΑ-Ν Β)(ΝΑ+ΝΒ) � ..--- = β · ΒΓ επιλύω το σύστη μα και έχω: }df (ΜΑ+ΜΒ) = }df (ΝΑ+ΝΒ) � α · ΑΒ Γl ρ{:πε ι :

}

το

}

ν

Ε

χ ·,

__

ΕΥΚΛΕΙΔΗΣ Β ' τ. Ι/37

}


-- β α και ΑΒ = --90° ΒΓ = --90° α+β α+β

Μαθηματικά για την Α ' Λυκείου

α 1 � 45° α2 � 45° α3 � 45° α4 � 45°

}

Α, Γ aντιδιαμετρικά � ΑΓ = 1 80° . Έχουμε: -ΑΒ + ΒΓ = 1 80° επιλύω το σύστημα και έχω: α · ΑΒ = β · ΒΓ

ίίί)

__

α 1 :s; 45o α2 ::; 45° α3 ::; 45° α4 ::; 45°

ή

Βr = -α-1 80° και ΑΒ = -β-180° α+β α+β

Az

Ά σκη ση 6'1

Μια ευθεία γωνία διαιρείται σε διαφορετικές γωνίες με ημιευθείες από την κορυφή της. i) Αν οι γωνίες είναι τρεις, τότε: «μια τουλάχι­ στον είναι μικρ ότερη των 60° και μια του­ λάχιστον είναι μεγαλύτερη των 60° » ii) Αν οι γωνίες είναι τέσσερις, τ ότε: «μια του­ λάχιστον είναι μικρότερη των 45° και μια τουλάχιστον είναι μεγαλύτερη των 45°» iii) Αν οι γωνίες είναι ν (ν�5) τότε: «μια τουλά­ χιστον είναι μικρότερη των 180°/ν και μια τουλάχιστον είναι μεγαλύτερη των 180°/ν» Λ1J ση

Αι

As

Σε κάθε περίπτωση άτοπο, η ισότητα ισχύει όταν α = αz = α3 = «4 = 45° (απορρίπτεται από την υπόθεση), άρα απομένει η πρόταση ι

(ίί) ίίί) (Η Γενίκευση), ν= ακέραιος θετικός �5

απορρίπτονται τα δυο ενδεχόμενα:

}

Θα εργαστώ με απαγωγή σε άτοπο, έστω: α 1 � 60° α 2 � 60° � α 1 + α2 + α3 > 1 80° (άτοπο, η ισότητα α3 � 60° δεν ισχύει από την υπόθεση) ί)

Αι

180° αι � -­ ν 180° αz � -­ ν

Az

180° α > -ν v -

}

ή

180° αι :s; -­ ν 180° αz :s; -­ ν 180° α < -ν v -

Αι

α 1 � 60° α 2 � 60° � α 1 + α2 + α3 < 180° (άτοπο, όμοια α3 � 60° όπως προηγούμενα) άρα ισχύει η πρόταση (ί) ίί) Όπως και στην απάντηση (ί) έχουμε: αν ισχύει:

Σε κάθε περίπτωση άτοπο, η ισότητα: 1 80° (απορριπτεται απο α ι = αz = . . . = α ν=-ν την υπόθεση), άρα απομένει η πρόταση (ίίί)

ΕΥΚΛΕΙΔΗΣ Β ' τ. l/38

'

'


Μαθηματικά για τη ν Α· Λυκείου

- Ασκήσεις στην Ισότητα

και τις Ανισοτικές Σχέσεις Τριγώνων Γιώ ργος Τ ριάντος

Φίλοι μαθητές Τα θέματα που αναπτύσσουμε στη συνέχεια, καλύπτουν το περιεχόμενο κυρίως του τρίτου κεφαλαίου της Γεωμετρίας. Κρίνεται απαραίτητη η γνώση: Των κριτη ρίων ισότητας τριγώνων ( ορθογωνίων και μη ) Των ιδιοτήτων της διχοτόμου γωνίας και της μεσοκαθέτου ευθυγράμμου τμήματος . Των βασικών ανισοτικών σχέσεων μεταξύ των γωνιών και των πλευρών τριγώνου . Για περισσότερα θέματα - στη συγκεκριμένη ύλη - μπορείτε να ανατρέξετε σε τεύχη παλαιοτέρων ετών του Ευκλείδη Β·, όπου υπάρχουν πολύ καλές επιμελημένες εργασίες άλλων συναδέλφων . Άσκη ση 1 . η ΒΔ ταυτίζεται με τη μεσοκάθετη της ΑΓ. • •

Σε κυρτό τετράπλευρο ΑΒΓΔ είναι: ΑΒ = ΒΓ και A = r . Να αποδείξετε ότι: ι. ΔΑ = ΔΓ 2. Η διαγώνιος ΒΔ είναι η μεσοκάθετη της δι­ αγωνίου ΑΓ.

Απόδειξη

Άσκηση 2

Σε τρίγωνο ΑΒΓ είναι ΒΓ = 2 ΑΒ ·

, Β= 2· Γ,

Α

ΒΔ η διχοτόμος της γωνίας Β και Ε το μέσον της ΒΓ. Να αποδείξετε ότι: ι . Η Β Δ είναι μεσοκάθετη της ΑΕ. 2. Το τρίγωνο ΑΒΓ είναι ορθογώνιο στο Α. Απόδειξη Γ

Δ Ι . Επειδή το τρίγωνο ΑΒΓ είναι ισοσκελές με κορυφή Β, οι γωνίες που πρόσκεινται στη βάση ΑΓ είναι ίσες, δηλ. ΒΑΓ = BrΑ (Ι)

Β

Ι . Επειδή το Ε είναι το μέσον της ΒΓ, έχουμε ΒΕ = ..!_ ΒΓ = ..!._ 2 ΑΒ = ΑΒ οπότε το τρίγωνο 2 2 Από την υπόθεση είναι: ΒΑΔ = BrΔ (2). ΑΒΕ είναι ισοσκελές με κορυφή το Β με αποτέλεσμα η διχοτόμος της γωνίας Β να είναι διάμεσος Απο τις σχέσεις (Ι),(2) προκύπτει ότι: και ύψος του, δηλ. μεσοκάθετος της ΑΕ. Α Γ Α Β Δ - Β Γ = Βf'Δ - Β Α δηλ. Ι = ΔΒΓ , το τρίγωνο ΔΒΓ είναι ΓΑΔ = ArΔ (3) . Από τη σχέση (3) συνάγε­ 2. Επειδή Γ = -Β 2 ται ότι το τρίγωνο ΔΑΓ είναι ισοσκελές με κο­ ισοσκελές με κορυφή το σημείο Δ και διάμεσο ΔΕ ρυφή το Δ και άρα ΔΑ=ΔΓ. ,που είναι και ύψος του. Δηλ. ΔΕ j_ ΒΓ . Τα τρίγω­ να ΑΒΔ, ΒΕΔ (από το κριτήριο Π-Γ-Π) είναι ίσα , 2. Επειδή είναι ΒΑ=ΒΓ και ΔΑ=ΔΓ τα σημεία αφού έχουν κοινή την πλευρά ΒΔ, ΒΑ=ΒΕ και Β, Δ ανήκουν στη μεσοκάθετη της ΑΓ, οπότε ·

·

ΕΥΚΛΕΙΔΗΣ Β' τ. Ι/39

·


Μαθηματικά για τη ν Α ' Λυκείου

ΑΒΔ = ΕΒΔ . Απο την ισότητα των τριγώνων αυ­ τών προκύπτει η ισότητα: ΔΑΒ = ΔΕΒ = I ορθή, δηλ. το ζητούμενο. Ά σκηση 3 "

Δύο μη ορθογώνια τρίγωνα ΑΒΓ, Α 'Β 'Γ έχουν τις γωνίες Α,Α' παραπληρωματικές και ΑΒ = Α 'Β ', ΑΓ = Α'Γ' . Να αποδείξετε ότι τα ύ ψη τους, που άγονται από τις κορυφές Γ, Γ είναι ίσα. Το αυτό για τα ύ ψη που άγονται από τις κορυφές Β,Β '.

Γ '�,- - - - - - υ ·\χ

\� \

\

ι

'"'--

'·"

ΑΒ = Α 'Β ' ΑΒ Δ = � Β = � Β ' = Α 'Β 'Δ ' ΒΔ = Β 'Δ ' Σύμφωνα με το κριτήριο Π-Γ-Π , τα τρίγωνα ΑΒΔ, Α ' Β ' Δ ' είναι ίσα οπότε απέναντι των ίσων πλευρών του βρίσκονται ίσες γωνίες. Άρα, θα είναι Α = Α · .

!

Δ' Α

ι

\\ >�Α' \' I \\ I'

··\

{

Συγκρίνουμε τα τρίγωνα ΑΒΓ, Α ' ΒΤΌ Έχουν � = � 'Β ' Α = Α' \jΒ' Β = Β' Είναι: Α + Α : = 2 ορθές .Υποθέτουμε ότι Α < I οπότε σύμφωνα με το κριτήριο Γ-Π-Γ τα τριγω­ ορθή, τότε Α ' > I ορθή , δηλ. το τρίγωνο Α ' Β Τ ' να ΑΒΓ, Α ' Β ' Γ ' είναι ίσα. είναι αμβλυγώνιο στο Α ' , με αποτέλεσμα το ίχνος Δ ' του ύψους Γ ' Δ ' να βρίσκεται στην προέκταση Ά σκψηι 5. Δίνεται ισοσκελές τρίγωνο ΑΒΓ με βάση ΒΓ και της ΒΆΌ τυχαίο σημείο Ρ της διχοτόμου ΑΤ της γωνίας Τα 1ψ Ο ()JγΙ.iJνΗκ τρ ίγΗη'σ ΑΓΔ, ΑΤ ' Δ ' είναι iσΗ. Α του τριγώνου. Αν η ΒΡ τέμνει την πλευρά ΑΓ αφού έχουν τις υποτε ίνουσες ΑΓ, Α ' Γ ' ίσες και τις στο Δ και η ΓΡ τέμνει την πλευρά ΑΒ στο Ε, τό­ οξείες γωνίες: ΓΑΔ = 2ορθ - Β 'Α 'Γ ' = Γ 'Α 'Δ ' τε να αποδείξετε ότι: 1 . ΒΔ=ΓΕ. 2. Η ΑΤ είναι μεσοκάθετος της ΔΕ. (δηλ. ίσες) Απόδειξη Από την ισότητα των τριγώνων αυτών προκύπτει Επειδή ΑΒ=ΑΓ η διχοτόμος ΑΤ είναι διάμεσος και ότι: ΓΔ=Γ ' ΔΌ ύψος δηλ. η ΑΤ είναι μεσοκάθετος της ΒΓ. Όμοια, αποδεικνύεται και το δεύτερο σκέλος (Ά­ σκηση ) 11)

λσκηση .:ι

Αν για δύο τρίγωνα ΑΒΓ, Α 'Β 'Γ είναι ΑΒ=Α Ή ', Β = Β ' και ισχύει ΒΔ=Β 'Δ ', όπου ΒΔ, Β ' Δ ' είναι οι διχοτόμοι των γωνιών Β , Β ' αντί­ στοιχα, τότε τα τρίγωνα αυτά είναι ίσα.

Γ Β Απ6δ:.:ιξη Ι) Συγκρίνουμε τα τρίγωνα ΑΒΔ, Α ' Β ' Δ ' . Έχουμε: Το σημείο Ρ ανήκει στη μεσοκάθετο ΑΤ της ΒΓ και συνεπώς είναι ΡΒ=ΡΓ, οπότε από το ισοσκελές τρίγωνο ΡΒΓ προκύπτει η ισότητα ΡΒΓ = Pf'B .

ΕΥΚΛΕΙΔΗΣ Β' τ. Ι/40


Μαθηματικά για την Α ' Λυκείου

Στη συνέχεια, συγκρίνουμε τα τρίγωνα ΑΒΔ, ΑΓΕ. με Β = Β' > Ι ορθή. Α Τα τρίγωνα αυτά έχουν: ΑΒ Α κοινή. ΑΒ=ΑΓ και Δ = ΑfΈ , σαν διαφορά ίσων γωνιών. Από το κριτήριο Γ-Π-Γ προκύπτει η ισότητα των τριγώνων αυτών με αποτέλεσμα την ισότητα των πλευρών ΒΔ=ΓΕ, που βρίσκονται απέναντι στη κοινή γωνία Α , καθώς και την ισότητα ΑΔ=ΑΕ. 2. Από την ισότητα ΑΔ=ΑΕ, στο ισοσκελές τρίγω­ νο ΑΕΔ η διχοτόμος ΑΤ είναι διάμεσος και ύψος του, δηλ. μεσοκάθετος της ΔΕ. Ά σκη ση 6.

Αν για δύο τρίγωνα ΑΒΓ, Α'Β'Γ ισχύουν Β = Β', Γ = Γ' και υα = υα. , τότε να αποδείξετε ότι τα τρίγωνα αυτά είναι ίσα. Απ ό δ ειξη

Ι) γποθέτουμε ότι ίσες γωνίες των τριγώνων είναι οξείες.

Φέρουμε τα ύψη ΑΔ = υα, Α'Δ' = υ a· · Τα ίχνη Δ, Δ' των υψών τους είναι σημεία εσωτερικά των πλευρών ΒΓ,Β'Γ αντίστοιχα. Α . Τα ο ρ Ο ογι:ί) νια τ ρ ίγωνα ΑΒΔ, Α'Β'Δ' είναι ίσα, αφού έχουν ΑΔ=Α 'Δ' και Β = Β ' . Άρα είναι και ΒΔ = Β'Δ' . Τα ορθογώνια τ ρ ίγωνα ΑΓ Δ, Α'Γ Δ' είναι ί­ σα, αφού έχουν ΑΔ = Α'Δ' και Γ = Γ · . Άρα εί­ ναι και ΓΔ=Γ Δ'. Β.

Α

Γ Α'

Γ

Β'

Τότε, Τα ίχνη Δ, Δ' των υψών τους είναι σημεία που βρίσκονται στις προεκτάσεις των πλευρών ΓΒ, ΓΒ' αντίστοιχα. Α. Τα ορθο γ ώνια τρ ίγωνα ΑΓΔ, Α'ΓΔ' είναι ίσα, αφού έχουν ΑΔ=Α' Δ' και Γ = Γ · . Άρα είναι και ΓΔ=Γ'ΔΌ Β . Τα ο ρ θ ογώνια τρ ίγων α ΑΒΔ, Α 'Β' Δ' είναι ίσα, αφού έχουν ΑΔ=Α'Δ' και ΑΒ Δ = 2 = 2ορθ - Β = 2ορθ - Β ' = Α 'Β 'Δ' Άρα, ΒΔ=Β'ΔΌ Παρατηρούμε ότι: Β = Β ', Γ = Γ· και ΒΓ=ΔΓ-ΒΔ=Δ'Γ-Β' Δ'=Β'Γ. Από το κριτήριο ισότητας Γ-Π - Γ προκύπτει ότι τριγ ΑΒΓ = τριγ Α 'Β 'Γ . Π α ρ ατή ρ η ση :

Το θέμα που ακολουθεί έχει «πολιτογραφηθεί» σαν το 4" Κ Ρ ΠΗ P I O ι σότητας τριγι!η·ω ν . Ά σκη ση 7

Αν για τα τρίγωνα ΑΒΓ , Α 'Β'Γ ισχύουν : ΑΒ=Α 'Β , ΑΓ=Α 'Γ, ΑΒ < ΑΓ , Β = Β ' , τότε τα τρίγωνα αυτά είναι ΙΣΑ.

Γ

Β Α'

Απ ό δ ε ιξη

Β'

Γ

Παρατηρούμε ότι: Β = Β ' , Γ = Γ · και ΒΓ = ΒΔ + ΔΓ = Β'Δ' + Δ'Γ' = ΒΤ . Από το κριτήριο ισότητας Γ -Π-Γ προκύπτει ότι τριγ ΑΒΓ = τριγ Α'Β'Γ . 11) γποθέτουμε ότι τα τρίγωνα είναι α μ β λυγ ώνια

Α'

Α

"

'

'

'

'

'

'

'

' ' ----- � Δ _:::::,. Β '--

8 , '----�

ΕΥΚΛΕΙΔΗΣ Β' τ. Ι /41

'

Γ

Γ


Μαθηματικά για την Α ' Λυκείου

Αρκεί να αποδείξουμε ότι τα τρίγωνα ΑΒΓ, ΑΈ ' Γ ' έχουν και την τρίτη πλευρά ίση. Δηλ. ΒΓ=Β ' Γ ' . Εφαρμόζουμε την μέθοδο απαγωγής στο άτοπο. Δεχόμαστε ότι ΒΓ =ι:. Β 'Γ ' . Αν υποθέ­ σουμε ότι ισχύει ΒΓ > Β 'Γ ' , τότε υπάρχει σημείο Δ εσωτερικό του τμήματος ΒΓ τέτοιο, ώστε να είναι ΒΔ = Β 'Γ ' . Συγκρίνοντας τα τρίγωνα ΑΒΔ, Α Έ ' Γ ' διαπιστώΑΒ = ΑΈ ' νου με ότι: Β = Β ' ΒΔ = Β 'Γ ' Από το κριτήριο ισότητας Π-Γ -Π τα τρίγωνα ΑΒΔ, Α Έ ' Γ ' είναι ΙΣΑ. Άρα, θα είναι και ΑΔ=Α ' Γ ' . Επειδή όμως είναι και ΑΓ=Α ' Γ ' θα έ­ χουμε ότι ΑΔ=ΑΓ. Από το ισοσκελές τρίγωνο ΑΔΓ είναι ΑΔΓ = Γ . Σύμφωνα με την υπόθεση, η ανισότητα ΑΒ<ΑΓ οδηγεί στην Β > Γ και λόγω της ΑΔΓ = Γ καταλήγουμε ότι Β > ΑΔΓ . Δηλ. σε άτοπο, αφού η γωνία ΑΔΓ είναι μεγαλύτερη της Β σαν εξωτερική γωνία του τριγώνου ΑΒΔ. Σε άτοπο καταλήγουμε και πάλι με παρόμοιο τρό­ πο, αν δεχθούμε ότι ισχύει ΒΓ<Β ' Γ ' . Τελικά είναι : ΒΓ=Β ' Γ ' , οπότε με εφαρμογή του κριτηρίου Π-Π-Π τα δύο τρίγωνα είναι ΙΣΑ.

{

Άσκ ηση 8.

Σε οξυγώνιο τρίγωνο ΑΒΓ με ΑΒ<ΑΓ η διχοτόμος της Α τέμνει τη μεσοκάθετο t της πλευράς ΒΓ σε σημείο Ρ. Να αποδείξετε ότι: Ρ Β Α + PrΑ = 2 ορθές.

Απόδειξη

Α

τέλεσμα ΡΓΕ = ΡΒΔ = 2ορθ - ΡΒΑ , απ όπου προ­ κύπτει το ζητούμενο. Άσκ ηση 9 .

Δίνεται αμβλεία γωνία χόy , δύο σημεία Α, Α' στην Οχ, δύο σημεία Β, Β ' στην Oy τέτοια, ώ­ στε: ΟΑ < ΟΑ ', ΟΒ < ΟΒ ' . Ν α αποδείξετε ότι : ΑΒ < Α 'Β ' .

Απόδειξη

Φέρουμε την ΑΈ. Παρατηρούμε ότι η γωνία Α 'ΒΒ ' είναι εξωτερική γωνία του τριγώνου ΟΑ ' Β, οπότε: Α 'ΒΒ ' > Α 'ΟΒ > Ιορθ . ο

φ \...ω_ _ _ _ _

Α'

Υ

χ

Στο τρίγωνο ΑΈΒ ' η γωνία Α 'ΒΒ ' είναι η μεγα­ λύτερη γωνία του (αφού είναι αμβλεία) και συνεπώς ισχύει : Α 'Β Β ' > Α 'Β Έ . Σύμφωνα με γνωστο θεώρημα, ισχύει ΑΈ ' > ΑΈ (1). Η γωνία φ είναι εξωτερική γωνία του τριγώνου ΟΑΒ , οπότε ισχύει: φ > x Oy > 1 ορθ και άρα φ > & . Από το ίδιο θεώ­ ρημα στο τρίγωνο ΑΑ ' Β προκύπτει ΑΈ > ΑΒ (2). Από τις σχέσεις (1), (2) έχουμε ΑΒ<Α'Β'. Άσκη ση 1 0.

Δίνεται τρίγωνο ΑΒΓ : ΑΒ < ΑΓ , η διχοτό μος ΑΔ της γωνίας Α και Ρ τυχαίο σημείο της ΑΔ. Ν α αποδείξετε ότι: 2) ΔΒ < ΔΓ 1) ΡΒΑ > ΡΓΑ 3) ΡΓ > ΡΒ 4) ΡΓ - ΡΒ < β - γ "

"

Απόδειξη

"

Α

Ρ

Φέρουμε ΡΔ .l ΑΒ , ΡΕ .l ΑΓ και συγκρίνουμε τα ορθογώνια τρίγωνα ΡΒΔ, ΡΓΕ. Έχουμε : Α . ΡΒ=ΡΓ , αφού το Ρ ανήκει στη μεσοκάθετο της ΒΓ ΡΔ=ΡΕ, αφού το Ρ ανήκει στη διχοτόμο της Α Συνεπώς ,τα τρίγωνα ΡΒΔ, ΡΓΕ είναι ίσα, με αποΒ.

Ε

ΕΥΚΛΕΙΔΗΣ Β' τ. Ι/42

Γ


Μαθηματικά για την Α ' Λυκείου

Η κάθετος από το Γστην διχοτόμο ΑΔ τέμνει την προέκταση της ΑΒ στο Ε. Διαπιστώνουμε ότι τα τρίγωνα ΑΓΕ, ΡΓΕ, ΔΓΕ είναι ΙΣΟΣΚΕΛΉ με κο­ ρυφές Α,Ρ,Δ αντίστοιχα. (Να το δικαιολογήσετε) ι ) Η γωνία Ρ Β Α είναι εξωτερική του τριγώνου ΡΒΕ οπότε ισχύει ΡΒΑ > ΡΕΒ = θ = ΡΓΑ . Άρα, Ρ ΒΑ > ΡΓΑ 2) Τα τ ρ ίγωνα ΑΔΕ, ΑΔΓ είναι ίσα, αφού έχουν: ΑΕ = ΑΓ και την ΡΑ κοινή πλευρά. Από ΡΕ = ΡΓ την ισότητα των τριγώνων αυτών προκύπτει ΑΕ Δ = ΑΓΔ ή το αυτό ΒΕΔ = Γ . Η γωνία ΔΒ Ε είναι εξωτερική του τριγώνου ΑΒΓ ά­ ρα, ισχύει ΔΒΕ > Γ = ΒΕΔ , οπότε ΔΕ>ΔΒ και τέλος ΔΓ>ΔΒ.

{

4)

Η γωνία ΡΒ Ε είναι εξωτερική του τριγώνου ΑΒΡ οπότε ισχύει Ρ Β Ε > ΑΡ Β (1 ). Η γωνία ΑΡΒ είναι εξωτερική του τριγώνου ΒΡΔ οπότε ισχύει ΑΡΒ > Ρ ΔΒ (2). Η γωνία ΡΔΒ είναι εξωτερική του τριγώνου ΑΔΓ οπότε ισχύει Ρ ΔΒ > Γ > θ = Ρ ΕΒ (3 ). Από τις σχέσεις (1 ), (2), (3) προκύπτει : ΡΒΕ > Ρ ΕΒ με αποτέλεσμα ΡΕ>ΡΒ δηλ ΡΓ>ΡΒ. Στο τρίγωνο ΡΒΕ από την τριγωνική ανισ ό­ τητα έχουμε:

Τα τρίγωνα ΑΡΕ, ΑΡΓ είναι ΙΣΑ (να το αιτιο­ λογήσετε)

3)

ΒΕ > ΡΕ - ΡΒ <:::> ΑΕ - ΑΒ > ΡΓ - ΡΒ <:::> ΑΓ - ΑΒ > ΡΓ - ΡΒ <:::> β - γ > ΡΓ - ΡΒ

Οι Πανελλήνιοι Μαθηματικοί Διαγωνισμοί

της Ελληνικής Μαθηματικής Εταιρείας θα πραγματοποιηθούν ως εξής: «0 ΘΑΛΗΣ» 4 Νοεμβρίου 2006

«0 ΕΥΚΛΕΙΔΗΣ» 20 Ιανουαρίου 2007 «0 ΑΡΧΙΜΗΔΗΣ)) 24 Φεβρουαρίου 2007

'Όπως

γνωρίζετε ιι Ε.Μ.Ε. χρηματοδοτεί τις δραστηριότητές της διαμέσου οικονομικών πό­

ρων, που προέρχονται ι<υρίως από τις συνδρομές των μελών της.

Η ετήσια συνδρομή, που ανέρχεται για το 2006 στα 2 0 Ευρώ, καλύπτει την αποστολή του Ευκλεί­ δη Α και Β δηλαδή τέσσερις αποστολές το χρόνο από 2 περιοδικά κάθε φορ ά. Για ενη μέρωσή σας,

επάνω στην ετικέτα με το όνομα σας αναγράφεται το συνολικό π οσ ό το οποίο οφείλετε στην Ε . Μ . Ε. Η τακτοποίηση των συνδρομών μπορεί να γίνει:

Στα γραφεία της Ε.Μ.Ε. 2. Στα γραφεία των παραρτημάτων της Ε.Μ.Ε. 3. Με ταχυδρομική επιταγή σε διαταγή, ΕΛΛΗΝΙΚΉ ΜΑΘΗΜΑτΙΚΉ ΕΤΑΙΡΕΙΑ, ΤΑΧ. ΓΡΑΦΕΙΟ ΑΘΗΝΑ 54, Τ.Θ. 30044 4. Με κατάθεση του αντιτίμου της συνδρομής στους παρακάτω λογαριασμούς: Τράπεζα ΕΘΝΙΚΉ, λογαριασμός όψεως 080/48002300 Τράπεζα ALPHA , λογαριασμός όψεως 10 100 200 20 1 9 988 Στην περίπτωση που η πλη ρωμή θα γίνει σε τράπεζα πρέπει να αποστείλετε στο Fax της ΕΜΕ (21 0-3641 025) την απόδειξη κατάθεσης συμπληρωμένη με τα πλήρη στοιχεία του καταθέτη (ονομα­ τεπώνυμο, διεύθυνση, τηλέφωνο, περιοδικό για το οποίο ενδιαφέρεστε) ώστε να είναι εφικτή η ορθή αποστολή των περιοδικών. Για περισσότερες πλη ροφορίες μπορείτε να καλέσετε στα γραφεία της ΕΜΕ ι.

ο

ο

2ι Ο-36 ι 6532 και 2ι Ο-36ι 7784. ΕΥΚΛΕΙΔΗΣ Β' τ. Ι/43


•-•ιιιι-�ι-•

!Jοω rJQJ!J lif uc!J�m rJ(j)rJJ ώrJJωe5fJ(J)rJJ Αλ - Ή γεβ. . ·- ρα -

____

_

_

Τριγωνομετρία.

Νίκος Σ . Τ απεινό ς

ε την έναρξη της νέας σχολικής χρονιάς εύχομαι απόκτη ση γνώσεων, που θα σε οδηγή σουν στην εκπλήρωση των ονείρων σου και στην κατάκτη ση της επιτυχίας. Η τριγωνομετρία που ακολουθεί, ελπίζω να σε βοηθή σει στην κατανόη ση της ενότητας αυτής, καθώς περιλαμβάνει ασκή σεις που αναφέρονται στην ύλη της Α' και Β ' τάξης του Ενιαίου Λυκείου. 1.

Αν ημχ =

και χ ε

(; ) ,π

να υπολογίσε-

τε την τιμή της παράστασης 2εφχ - 3ημχ Α= 1 Οσφχ + 6συνχ Λύ ση

(

2.

)

Επειδή χ �,π δηλαδή είναι τόξο του 2°υ τεταρτημορίου έχουμε: � � συνχ = -�1 - ημ2χ = - 1 - % = - = -15 ημχ -15 και 3 = -=εφχ = -συνχ 2 2 3 Ι 2 2-/5 -= = -σφχ = εφχ -/5fi 5Ji 2 - 25 - 3 5 3 - -15 = : Η) = ---15 Ά ρα Α = 4./5 - 4 2 +6 ε

-%

( ) ιο(- )

-15 ( -15 - ι ) = 2-/5 = -15 = = 4 ( -15 + ι ) 2(-/5 + 1 ) 2(-!5 + ι ) (-!5 - ι ) = 2(5 - -/5 = 5 - -/5 5 - 1) 8 Αν ημχ+συνχ=κ (1), χ ε R να υπολογίσετε την τιμή της παράστασης: Α= ημ6χ+συν6χ Λ ύσ η

Από ( 1) έχουμε: ( ημχ + συνχ ) 2 = κ2 ημ2χ + συν2χ + 2ημχσυνχ κ2 Ι + 2ημχ + συνχ κ2 κ2 - Ι ημχσυνχ = (2) 2Έχουμε: 3 3 = Α ( η μ 2 χ ) + ( συν 2 χ ) = = η μ 2 χ συν 2 χ ( + )( η μ4 χ - η μ 2 χσυν 2 χ συν4 χ ) = = ι [ ημ2 χ 2 ( ) + ( συν 2 χ )2 - η μ 2 χσυν 2 χ J = = { ημ2χ + συν2χ) - 2ημ2χσυν2χ - ημ2χσυν2χ = = 12 - 3ημ2χσυν2χ = Ι - 3( ημχσυνχ ) 2 =

ΕΥΚΛΕΙΔΗΣ Β' τ.l/44

<::::>

=

=

<::::>

<::::>

+


Μαθηματικά για την Β ' Λυκείου

��-{ ) �

κ' - ι ' 2 ( κ2 - 1 ) 2 = 1 - 3 -'-----'--4 4 - 3 (κ 4 - 2Κ2 + 1) -3κ 4 + 6κ2 + 1 = 4 4 3.

)

(

iii)

Η

ίν)

Ισχύει συν( α-β)=συνα--συνβ για κάθε α,β e R Η συνάρτηση f(χ)=3ημ(χ-π) έχει ελά­ χιστη θετική περίοδο Τ=π , Ι σχυει συν 2 χ=ημ2 χ-συν2 χ σφσφβ + 1 Ισχύει σφ β - α = για ( ) σφα - σφ β

vi)

)

(

εφχ=εφα

ν)

Να απλοποιηθεί η π αράσταση : 1 π 3π ημ + χ φ 9π + χ συν +χ ) ( 2 � Α = --� ���----��--� 9 1 π συν - χ σφ - χ συν 5π - χ

( ;� } � ( )(

ii)

)

για όλες τις τιμές του χ, που ορίζονται οι αναγραφόμενοι τριγωνομετρικοί αριθμοί.

( ) ( ) ( ) = -ημ (� - χ ) = -συνχ εφ( 9 π + χ) = εφ(8π + π + χ) = εφ( π + χ) = εφχ συν ( 3; + χ ) = συν ( 2π - � + χ ) = = συν ( - % + χ ) = συνχ ( � - χ = ημχ ) συν ( 92π - χ ) = συν ( 4π + % - χ ) = = συν ( % - χ ) = ημχ σφ ( 1 �π χ ) = σφ ( 6π -%- χ ) = = σφ ( -%- χ ) = -σφ + χ ) = (% [ π-(;)] ( ; ) συν(5π - χ) = συν( 4π + π - χ) =

Λ ύ ση

Είναι: ημ 1 �π +χ =ημ 8π- � + χ =ημ -� + χ =

νίί)

χ

= σφ

4 . Να χαρακτηρίσετε τις παρακάτω προτάσεις με Σωστό (Σ) ή Λάθος(Λ) i) Η εξίσωση συνχ=3/2 είναι αδύνατη

εξίσωση εφ χ +

�)

= 8 έχει λύσεις

5. Να επιλέξετε τη σωστή απάντηση : i ) Περίοδος της συνάρτησης

(

f(x) = 2συν 3χ -

�)

είναι:

π � π α) Τ = - , β) Τ = π, γ) Τ = 2π, δ) Τ = - , ε) Τ = 2 3 3 ii) Μια λύση της εξίσωσης 2ημχ = J2 είναι

α) -

π

4

, β)

π

2

, γ)

3

, δ)

4

ί ί ί ) Η τιμή της παράστασης

= εφχ

= συν( π - χ) = -συνχ -συνχεφχημχ = 1 Άρα: Α = ημχεφχ ( -συνχ)

(

χ=2κπ + α με κ e Ζ

όλα τα α,β που ορίζονται οι παραστά­ σεις 1 + συν2α , 2 για ολα τα α ν ίίί ) Ισχύει σφ α = 1 - συν2α που ορίζονται οι π α ραστάσεις ix) Μια από τις λύσεις της εξίσωσης 2ημ2χ=1 είναι χ=π /1 2 α 2εφ 2 για όλα τα α Ισχύει εφ2α = χ) α 1 - εφ z 2 που ορίζονται οι π αραστάσεις

_

= -σφ

<=>

J2

Γ-

α) - , β) ν 3 , γ) ι, δ)

, ε)

4

π 2εφ -

____.:8 �

_

1 - εφ z Π

Jj

είναι:

S

, ε) ο

3 Τ iv) ο ημ4α αeR, είναι ίσο με: α) 2ημα·συνα, β) 4ημα·συνα.συν2α, α α γ) 2ημ - συν - , δ) 1-2ημ2 2α, 2 2 ε) κανένα από τα προηγούμενα 2 ν) Αν χ=2ημ3α·συν3α και y=1-2ημ 3α τότε , , , η παρασταση χ 2+ y2 ειναι ιση με:

ΕΥΚΛΕΙΔΗΣ Β' τ. Ι/45

2


Μαθηματικά για την Β ' Λυκείου

J3 J2 γ) α) 1, β) ε) Ο , -1, δ) 2 2 6. Να aντιστοιχίσετε κάθε τριγωνομετρικό α­ ριθμό της στήλης Α στην παράσταση της στή­ λης Β, με την οποία είναι ίσος Στ ' λ Α

1.ημ4χ

2. συν9χ 3. ημ5χ 4.συν7χ

Στ ' λ

Β

α. ημχ.συν5χ+ημ5χ. συνχ β. συν8χ. συνχ-ημ8χ. ημχ γ. ημ12χ. ημ5χ+ συν12χ. συν5χ δ. συν12χ. ημ5χ+ ημ12χ.συν5χ ε. ημ3χ.συν2χ+ συν3χ.ημ2χ στ. ημ5χ. συνχ-ημχ. συν5χ χ. συν9χ

7. Να aντιστοιχίσετε κάθε τριγωνομετρικό α­ ριθμό της στήλης Α στην παράσταση της στή­ λης Β, με την οποία είναι ίσος

Στήλ,η Α

1.ημ22χ 2.

Στήλη Β

1 - συν4χ 1 + συν4χ συν4χ - 1 β. 2 συν4χ + 1 γ. 2 συν4χ - 1 δ. συν4χ + 1 1 + συν4χ ε. 1 - συν4χ -1 - συν4χ στ. 2 ζ. 1 - συν4χ 2 α.

συν22χ

3. εφ22χ 4.σφ22χ

π κΕΖ α) ημχ = -1 <=>ημχ = ημ-π <=> χ = 2κπ+-, 2 6 6 5π η' Χ = 2 ΚΠ + -, Κ Ε ι.ι. 6.

'71

( )

β)ημχ=- � <=>ημχ= ημ � � <=> χ=2κπ- � , ΚΕΖ 7π Κ Ε ι.ι. η' Χ = 2 ΚΠ + -, 6 Αν y= -2 τότε έχουμε . ι ημχ Ι = -2 που είναι αδύνατον γιατί . ι ημχ Ι 1 '71

s

Να βρεθεί το πεδίο ορισμού της συνάρτη­ σης: 2ημχ f(x) = εφ2χ - 3 9.

Λύ ση

Για να ορίζεται η εφχ πρέπει χ * κπ + -π2 με κ

ΕΖ.

Ακόμα πρέπει: εφ2χ - 3 * Ο Αν εφ2χ = 3 <=> εφχ = J3 ή εφχ = -J3 εφχ = ν-' <=> εφχ = εφ-π <=> χ = κπ +-π με κ Ε Ζ 3 3 r:;

( )

εφχ =.J3 <=> εφχ = εφ -� <=> χ = κπ- � με κ Ε Ζ

Να λυθεί η εξίσωση συν2χ - J3ημ2χ = Ο (1) 3 αν χ ε Ο, 1 Ο.

[ ;)

Λύ ση

Να λυθεί η εξίσωση: 2ημ2χ+3· 1 ημχ l -2=0

Jj ( 1 ) <=> ν -'ημ2χ = συν2χ <=> εφ2χ = 3 (2) με κπ + -π με Ε Ζ συν2χ * Ο <=> 2χ * κπ +-π2 <=> χ * Λύ ση 2 4 (1) <=> 2 1η μχ l 2+3 · 1 ημχ l -2=0 3π <=> Ο � κπ + -π < 3π <=> Επειδη:' Ο � χ < Θέτουμε I ημχ I =y και έχουμε 2 2 4 2 2/+3y-2=0 που αληθεύει για y=1/2 ή y=-2 --41 � -κ2 < -45 <=> --21 � κ < -25 1 ' ' εχουμε: ' Αν l τοτε I ημχ I = 2 <=> ημχ = 2 η Επειδή κ Ε Ζ έχω κ = Ο <=> χ * π4 1 ημχ = --2 3π η' κ = 1 <::::> x :;t: -π2 + -π4 <=> x :;t: 4 8.

r:;

(1)

κ

y

I

=

1

ΕΥΚΛΕΙΔΗΣ Β' τ. l/46


Μαθηματικά για την Β ' Λυκείου

5π η' κ = 2 <:::::> x :;t: π + -π4 <:::::> x :;t: 4

=

Τότε (2) <=:> εφ2χ = εφ� <=:>2χ κπ+� 6 6 κπ π <=:> χ = - + - ' κ ε Ζ 2 12 Επειδή 3π κπ π 3π 1 17 ο ::; χ < - <:::::> ο ::; - + - < - <=> -- :::;; κ < 2 2 12 2 6 6 κ ε Ζ έχω κ Ο <:::::> χ = � δεκτή 12 π <:::::> χ = 7π δεκτη' η' κ = 1 <:::::> χ = -π + 12 2 12 π 13π η' κ = 2 <:::::> χ = π + U <:::::> χ U δεκτη' 3π Άρα χ ε �;, \ 2

---

2 εφ 2 α

=

=

=

ι�,

}

2 1 1 . Να λυθεί η εξίσωση : συν2 χ+ ημ 2χ=l (1)

1 3. Αν εφχ=2 /5 να υπολογισθεί η τιμή της πα­ ράστασης Α=2ημ2χ+5συν2χ Λύ ση

2 Α = 2ημ2χ + 5συν2χ = 2 2εφχ2 + 5 Ι - εφ 2 χ Ι + εφ χ 1 + εφ χ 4 � + 5 - 5 __±_ 4εφχ + 5 - 5εφ 2 χ ---=-. 5 ----=2c:;_5 = 2 Ι + εφ χ Ι + __±_ 25 � + 5 _± 29 = 5 29 5 = _2_ 29 = 5 . 25 25 =

_

Αν ημχ+συνχ=S/4 (1) να υπολογίσετε το

1 4. Λύ ση

( 1 ) <:::::> συν2 χ-1 + (2ημχ. συνχ) 2 = 0 <=:>-ημ2 χ +4ημ2 χ. συν2χ = Ο <=:>ημ2 χ(4συν2 χ-1 ) = Ο <:::::> ημ2 χ = Ο ή 4συν2 χ-1 = Ο Αν ημ2 χ = Ο<=:> ημχ = Ο <:::::> ημχ =ημΟ<=:>χ= κπ με κε Ζ 2 Αν 4συν χ-1 = 0 <:::::> 4συν2 χ = 1 <:::::> συν2χ = 1/4 <=> συνχ 1 2 π <:::::> συνχ = συν- <:::::> χ = 2κπ±-π με κ ε Ζ 3 3 . 2π με 2π <:::::> χ = 2κπ ± η' συνχ = - 1 <:::::> συνχ = συν 3 2 3 κεΖ = -

-

1 2 . Να αποδείξετε ότι

1 + συν8α + συν4α 1 - εφ 2 2α ------- = -----'--ημ8α + ημ4α 2εφ2α

Λ ύ ση

1 + συν8α + συν4α 1 + 2συν 2 4α - 1 + συν4α ημ8α + ημ4α 2ημ4ασυν4α + ημ4α Ι = = συν4α ( 2συν4α + Ι ) = σφ4α = -ημ4α ( 2συν4α + 1 ) εφ4α

ημ2χ Λύ ση

Από την (1) έχω: 25 <=:>ημ2 χ +συν2 χ + 2η χσυνχ = 25 <:::::> ( ημχ + συνχ )2 = μ Ι6 16 25 <:::::> ημ2χ = -. 9 Ι + ημ2χ = Ι6 Ι6 1 5. Αν α+β= � ν α δείξετε ότι: 3

(2συνα+συνβ) 2 +(2ημα-η μβ) 2 =7 Λύ ση

(2συνα+συνβ)2+ (2ημα-ημβ) 2= =4συν2 α+ συν2 β+4συνασυνβ +4 ημ2 α + ημ2 β-4 ημαημβ = =4( συν2α+ ημ2 α) + ( συν2 β+ ημ2 β) +4 ( συνα.συνβ-ημα.ημβ)= =4. Ι + Ι +4συν(α+ β)= 5 +4· συν- = 5 +4· -Ι = 7 2 3 π

(

)(

1 6. Να δείξετε ότι:

π 1 - εφ 2 32

Λύ ση

ΕΥΚΛΕΙΔΗΣ Β ' τ. Ι/47

π 1 - εφ 2 16

)(

)

π π 1 - εφ 2 - = 8εφ 32 8


Μαθηματικά για την Β' Λυκείου

Γνωρίζουμε ότι: εφ2χ = 2εφχ2 <:::::> 1 - εφ 2 χ = 2εφχ 1 - εφ χ εφ2χ Τότε έχουμε: 1 - εφ 2 3� 1 - εφ 2 : 1 - εφ 2 = 1 π εφ-π εφ-π εφπ εφ= 2 ____21_ _R = 8εφ � . 2 ___l§_ 2 π8 = 8 ____ π π π 32 εφ- εφ- εφ-4 εφ-4 16 8

(

)(

)(

�)

Α Β Γ Α Β Δ εφ -εφ - εφ - + εφ -εφ - εφ - + 2 2 2 2 2 2 Α Γ Δ Β Γ Δ +εφ -εφ - εφ - + εφ -εφ -εφ 2 2 2 2 2 2 Λ ύ ση

εφΒ = - και εφΓ = 2 5 Να υπολογίσετε τη γωνία Α

Σε κάθε κυρτό τετράπλευρο ΑΒΓ Δ γνωρίζουμε ό­ τι: Α + Β + Γ + Δ = 360° <:::::> Α + Β + Γ + Δ = 1 80ο <:::::> 2 2 2 2 Α + Β = 1 80ο Γ + Δ 2 2 2 2 Οπότε έχου με: εφ � + � = -εφ + Α + εφ-Β Γ + εφ-Δ εφεφ2 = 2 2 2 <:::::> Β Γ Α Δ 1 - εφ-εφ1 εφ-εφ2 2 2 2 Α + �-Β - �-�-�Α Γ Δ - �-�-�Β Γ Δ= �2 2 2 2 2 2 2 2 Γ Α -εφΒ Γ +�-�-�Α Β Δ <:=> =-�- -εφ -Δ +�-� 2 2 2 2 2 2 2 2 Α εφ- + εφ-Β + εφ-Γ + εφ-Δ = 2 2 2 2 Α Β Γ + εφ-εφ-εφΑ Β Δ+ = εφ-εφ-εφ2 2 2 2 2 2 Α Γ Δ + εφ-εφ-εφΒ Γ Δ εφ-εφ-εφ2 2 2 2 2 2

Λύ ση

20. Αν σε τρίγωνο ΑΒΓ ισχύει ότι:

-

17. Αν σε τρίγωνο ισχύει: συν2Α-συν2Α=ημΑ(1ημΑ) (1) να δείξετε ότι: Β+Γ= 1 50 ° ή Β+Γ=30 ° Λύ ση

( 1 ) <:::::> συν2 Α-συν2Α=ημΑ-ημ2Α <:::::> 1 - συν2Α=ημΑ <:::::> 1-( 1-2 ημ2 Α)= η μΑ <:::::> 2 ημ2 Α= η μΑ <:::::> ημΑ(2ημΑ-l )=Ο Επειδή Α γωνία τριγώνου ισχύει ότι ημΑ;CΟ Οπότε έχουμε: 2ημΑ-1 =0<:=> ημΑ= l/2 <:::::> ημΑ=ημ30° <:=>Α=30°<:=> Β +Γ= 1 50° ή ημΑ=ημ1 50° <:=>Α=1 50°<:=> Β + Γ=30° 18. Σε τρίγωνο ΑΒΓ είναι: J3

J3

Σε τρίγωνο ΑΒΓ γνωρίζουμε ότι Α+Β + Γ= 1 80°<:=>Α=1 80°-(Β + Γ) Οπότε εφΑ= -εφ(Β + Γ) <:::::> εφΑ = εφΒ + εφΓ <:::::> 1 - εφΒεφΓ Jj Jj 7 J3 +5 --- εφΑ = - 10 ...-. εφΑ = 2 Jj Jj 1 0 - 3 -..-.. .... l - 25 1() εφΑ = -J-3 <:::::> εφΑ = εφ120 <:::::> Α = 120° 1 9. Σε κυρτό τετράπλευρο ΑΒΓΔ να αποδείξετε Α Β Γ Δ , οτι: εφ - + εφ - + εφ - + εφ - = 2 2 2 2

( ) ) ( � �)<=> _

(

ημΒ(συνΓ-ημΒ)=συνΒ(συνΒ-ημΓ) (1), aποδείξτε ότι αυτό είναι ορθογώνιο Λ ύ ση

( 1 ) <:=>η μΒσυνΓ-η μ2 Β=συν2 Β-συνΒη μΓ <:=>η μΒσυνΓ+ συνΒη μΓ= η μ2 Β +συν 2 Β <:=>η μ(Β + Γ)= 1 <:=>η μΑ=l (γιατί Α+Β+Γ= 1 80°) οπότε Α=90° Απαντή σεις : Άσκη ση 4 . Ά σκη ση 5 . Άσκη ση 6 .

ΕΥΚΛΕΙΔΗΣ Β' τ.l /48

Σ, Λ, Σ, Λ, Λ, Λ, Σ, Σ, Σ, Λ δ, ε, γ, β, α 1-στ, 2-β, 3-ε, 4-γ


Μαθηματικά για την Β ' Λυκείου

Γεω μετρία

Μετρικές Σχέσεις στο Τρίγωνο του Αποστόλη Κακα β ά

Οι Μετρικές σχέσεις στο τρίγωνο βασίζονται στο πυθαγόρειο Θεώρημα (Π.Θ.) με τις συναφείς προτάσεις και στις συνέπειές του, δηλαδή στα θεωρήματα οξείας και αμβλείας γωνίας που αποτελούν την επέκταση του Πυθαγορείου θεωρήματος (Ε.Π.Θ.) καθώς και στα θεωρήματα διαμέσων ( Θ.Δ.) ι. Έστω τρίγωνο ΑΒΓ με πλευρές α,β,γ. Άντι­ Τα τρίγωνα ΑΒΓ και ΒΜΔ είναι όμοια στοιχίστε τη σχέση που συνδέει τις πλευρές ( Α = Μ = 90° και Β κοινή) άρα του, της στήλης Α με τη γωνία Α του τριΔΒ = ΜΒ <:::::> ΔΒ = 10 <:::::> ΔΒ = 200 <:::::> ΔΒ = 25 γώνου της στήλης Β. ΒΓ ΑΒ 20 16 16 2 Το Δ σημείο της μεσοκαθέτου του ΒΓ άρα Στήλη Α Στήλη Β 2 + 2 + 2 25 και ΑΔ =ΑΒ-ΔΒ = 16-25 = -7 ι. α = β γ βγ J3 α. Α = 30° ΔΓ = ΔΒ = 2 + 2 2 2 2 2 2. α = β γ β . Α = 4 5° Α

3. 4. 5. 6. 7.

α2 α2 α2 α2 α2

= β 2 + γ 2 βγ .fi = β 2 + γ 2 + βγ = β 2 + γ 2 _ βγ J3 = β 2 + γ 2 βγ = β 2 + γ 2 + βγ .fi _

_

γ. Α = 50° δ. Α = 60° ε. Α = 90° στ. A = l20° ζ. Α = ι25° η. A = l35° θ. Α = ι5ο0

Λίι ση

1 � θ , διότι α2 = β 2 + γ 2 + βγ .J3 � � -2βγσυνΑ = βγ../3 �συνΑ = - .J3 � Α = 150° 2 Ομοίως 2 � ε, 3 � β, 4 � στ, 5 � α, 6 � δ, 7 � η

3. Δίνεται ορθογώνιο τρίγωνο ΑΒΓ με υποτεί­ νουσα ΒΓ=5 και ύψος ΑΔ=2,4. Να υπολογίσε­ τε τα μήκη των τμημάτων ΔΒ, ΔΓ, ΑΒ, ΑΓ. Λύ ση

Έχουμε ΔΒ+ΔΓ = ΒΓ = 5 και ΔΒ ΔΓ = ΑΔ2 = 2,42 = 5,76. Άρα τα ΔΒ, ΔΓ είναι ρίζες της εξίσωσης: χ2-5χ+5,76=0 με Ο<χ<5 Η εξίσωση αυτή έχει διακρίνουσα Δ=25-23,04=1 ,96 και ρίζες χ 1 = 5 + 1 • 4 = 3,2 2 5 -1,4 = I , 8 που προφανως , ανηκουν , στο και χ 2 = 2 (0,5). ·

Γ

2. Δίνεται ορθογώνιο τρίγωνο ΑΒΓ με Α = 90° και μήκη πλευρών ΒΓ=20 και ΑΓ=ι2. Στο μέσο Μ της υποτείνουσας ΒΓ φέρω ευθεί­ α(ε) κάθετη στη ΒΓ που τέμνει την ΑΒ στο εσωτερικό της σημείο Δ. Να υπολογιστούν τα μήκη των τμημάτων ΔΓ και ΔΑ. JL.l._____

Λ ύ ση

Εφαρμόζουμε το Π. Θ. στο τρίγωνο ΑΒΓ οπότε ΑΒ2Γ=ΒΓ2- ΑΓ2 = 400-144=256 Άρα: ΑΒ= 16

Α

Β

Άρα (ΔΒ= 1 ,8 και ΔΓ=3,2) ή (ΔΒ=3,2 και ΔΓ=1 ,8) Αν ΔΒ=Ι ,8 και ΔΓ=3,2 τότε ΑΒ 2=ΔΒ" Β Γ=\ ,8.5=9 οπότε ΑΒ=3 και ΑΓ2=ΔΓΒΓ =3,2· 5 = 1 6 οπότε ΑΓ=4 Αν ΔΒ=3,2 και ΔΓ=1 ,8 τότε ομοίως ΑΒ=4 και ΑΓ=3 •

β ' τρό π ος

Β

}

}

2 2 β 2 + γ 2 = 25 � β2 + γ 2 + 2βγ = 49 � βγ = αυα = 5 · 2,4 = 12 β + γ - 2βγ = 1

ΕΥΚΛΕΙΔΗΣ Β ' τ. t /49


}

Μαθηματικά για την Β ' Λυκείου

}

β + γ = 7 η' β + γ = 7 κ.λπ. β-γ=Ι γ-β = Ι 4.

Δίνεται ορθογώνιο και ισοσκελές τρίγωνο ΑΒΓ με Α = 90° και ΒΓ=α, η διάμεσός του ΑΜ και Ν το μέσο της. Στη ΒΓ έστω σημείο 3α Κ με ΓΚ= . Να δειχθεί ότι η ΚΝ είναι 8 κάθετη στη ΝΒ. Απόδ ειξη

}

Α = 90° =� => ΑΜ = ΒΓ Μ 2 2 Α διάμεσος Μ Ά ρ α ΝΜ = Α = � 2 4 3α = -α . ΜΚ = ΜΓ - ΓΚ = -α - 2 8 8 Εφαρμόζουμε το Πυθαγόρειο Θεώρημα στο 2 2 Κ Μ Ν οπότε ΚΝ2 = ΚΜ2 + ΝΜ2 = � + � = 5α2 α2 + α2 = =(1) 64 16 64

() ()

Γ

Κατασκευάζουμε εξωτερικά του τριγώνου, τετράγωνα ΒΓ ΔΕ πλευράς α, ΑΒΘΗ πλευ­ ράς γ και ΑΓΖΙ πλευράς β. Φέρω ΔΚ κάθε­ τη στην ΖΓ που τέμνει την προέκτασή της στο Κ. Να δειχθεί ότι ί) τα τρίγωνα ΑΒΓ και ΔΚΓ είναι ίσα και ii) ΖΔ 2+ΙΗ2+Ε Θ2=6α2 Λύ ση

i) Τα τρίγωνα ΔΚΓ και ΑΒΓ είναι ίσα, διότι είναι ο ρ θογώνια και έχουν ΒΓ=Γ Δ=α και AfB = 90° - φ = κfΔ . ii) Από (i) παίρνουμε ΓΚ=ΑΓ=β και ΔΚ=ΑΒ= γ Εφαρ μόζουμε το Π. Θ. στο Δ Κ Ζ και έχουμε: ΔΖ2=κz2+ΚΔ2=(2β)2+γ2 =4β2+ γ2 (Ι) Ομοίως ΕΘ2 =4γ2+β2 (2), ενώ ΙΗ2=β2+γ2 (3) Από τις (Ι), (2), (3) έχουμε: ΔΖ2+1Η2+ ΕΘ2=4β2+γ2+(β2+γ2)+4γ2+β2= =6β2+6γ2=6(β2+γ2)= 6α2 . Δ

Δ

----

r"""'-

I I I I I I I I I

--'-<r ---------------Ω κ φ

α

Ε

β

Β

β ' τρόπο ς:

() ()

Εφαρ μόζουμε το Πυθαγόρειο Θεώρημα στο 2 2 Μ Ν Β οπότε ΒΝ2 = ΜΝ2 + ΜΒ2 = � + � = 5α2 α2 + α2 = =(2) Ι6 4 Ι6 2 2 Εξάλλου: ΒΚ2 = ( α - ΓΚ )2 = 5; = 2

() :

(3)

Δ

Από (Ι), (2) έχουμε στο Κ Ν Β : 5 2 +� 5 2 =� 25 2 (=3 ) ΒΚ2 ΚΝ2 + ΒΝ2 = � 64 Ι6 64 Ν Άρα Κ Β = 90° δηλαδή ΚΝ 1_ ΝΒ . 5. Έστω ορθογώνιο τρίγωνο ΑΒΓ με Α = 90°

ΔΖ2 = α 2 + β 2 - 2αβσυνΔf'Ζ = = α 2 + β 2 - 2αβσυν ( Ι 80° - Γ) = = α 2 + β 2 + 2αβσυνΓ = α 2 + β 2 + 2αβ · � = α = α 2 + 3β 2 = β 2 + γ 2 + 3β 2 = 4β 2 + γ 2 κ.λ.π.

6. Δίνεται ισόπλευρο τρίγωνο ΑΒΓ πλευράς α. Προεκτείνουμε τις πλευρές ΑΒ, ΒΓ, ΓΑ κα­ τά ΒΖ=ΓΔ=ΑΕ =α. Να βρεθούν τα μήκη των πλευρών του τριγώνου ΔΕ Ζ. Λύ ση

Β εξ = 120° Εφαρμόζουμε το νόμο συνημιτόνων στο Ι3 = 60°

ΕΥΚΛΕΙΔΗΣ Β' τ.l/50


Μαθηματικά για την Β ' Λυκείου Δ

Α

ΒΖΔ . ΔΖ2 = ΒΖ2 + ΒΔ2 - 2ΒΖ · ΒΔσυνΖΒΔ = = α 2 + ( 2αγ - 2α2ασυνΙ20° = 5α 2 - 4α2 - � = = 5α2 + 2α2 = 7α 2 .

( )

Ε

Β Δ

Δ

z

Άρα ΔΖ = α-fi . Ομοίως ΔΕ = ΕΖ = αJΊ .

ίί)

7. Δίνεται ισόπλευρο τρίγωνο ΑΒΓ πλευράς α και σημείο Δ της ΒΓ ώστε ΒΔ = ! . Αν Κ 3 σημείο της πλευράς ΑΒ ώστε ΑΚ=ΚΔ τότε: i) Να υπολογίσετε τα τμήματα ΚΔ , ΚΓ ii) Να δείξετε ότι δεν υπάρχει τρίγωνο με πλευρές ΚΒ, ΚΓ, ΒΔ. ί)

Απ ό δ ε ιξ η

Εφαρμόζουμε το νόμο συνημιτότων στο Κ ΒΔ και έχουμε: ΚΔ2 = ΚΒ 2 + ΒΔ2 - 2 · ΒΚ · ΒΔσυνΒ = 2 = ( α - ΑΚ )2 + � - 2 ( α - ΑΚ ) � συν60° = 9 3 2 α α = ( α - ΚΔ )2 + 9 - 2 ( α - ΚΔ ) "3 · "2Ι = α2 - α 2 + α · ΚΔ = α 2 - 2αΚΔ + ΚΔ2 + 3 9 3 2 2 ΚΔ α · α α Άρα: 2 αΚΔ - -3- = α2 + 9 - 3 , δηλαδη' ΚΔ = α + -α - -α οποτε 7α (Ι) , ΚΔ = 2ΚΔ - ' 3 9 3 Ι5 Δ

--

Ομοίως από το νόμο συνημιτότων στο Α Γ Κ έχουμε: ΓΚ 2 = ΑΚ 2 + ΑΓ 2 - 2 · ΑΚ · ΑΓ · συνΑ = = ΑΚ 2 + α 2 - 2 · ΑΚ · α-Ι = 2 49α2 + α2 - α 7α = = ΚΔ2 + α2 - α · ΚΔ <=Ι J -225 Ι5 2 2 2 2 = 49α + 225α - 1 05α = Ι69α 225 225 Οπότε: ΓΚ = Ι 3α (2) Ι5 Έχουμε: 7α = ­ 8α ΚΒ = α - ΑΚ = α - ΚΔ = α - (3) Ι5 Ι5 � = Ι3α � ΚΓ + Άρα ΚΒ + ΒΔ = 8α Ι5 3 Ι5 δηλαδή δεν υπάρχει τρίγωνο με πλευρές ΚΒ, ΚΓ, ΒΔ.

8.

Έ στω τρίγωνο ΑΒΓ με μήκη διαμέσων μα = ν� -'-' ' μ μ =

Jϊii5 -

2-,

m

,

μ1 = 2- , να βρεθει

το είδος του τριγώνου ως προς τις γωνίες του. Λύ ση

}

μα2 = 2β2 + 2γ2 - α2 4 2β 2 + 2γ2 - α2 = 132 (Ι) 2 2 2 μβ2 = 2α + 2γ -β <::::> 2α2 + 2γ2 - β 2 = 105 ( 2 ) 4 2α2 + 2β2 -γ2 = 33 ( 3 ) 2 2 2 2α -γ + 2β μγ2 = 4 (+) ( Ι ),( 2 ), (3 )=:>3α2 + 3β2 +3γ2 = 270 =:> α2 +β2 +γ2 = 90 2α2 + 2β2 + 2γ2 = Ι80 (4) ( ) ( 4 ),( Ι )=:> 3α 2 = 48 :::::> α2 = Ι6 ( ) ( 4), ( 2 )=:> 3β 2 = 75 =:> β 2 = 25

ΕΥΚΛΕΙΔΗΣ Β ' τ. Ι /5 1

-


-

Μαθηματικά για την Β ' Λυκείου

( )

( 4 ),( 3 )=> 3γ 2 = 147 => γ 2 = 49 Άρα α2 +β2 = 16+ 25 = 41 < 49 = γ2 οπότε: f > 90° . 9. Αν σε ένα τρίγωνο ΑΒΓ ισχύουν οι σχέσεις 4μ2α=α2+2γ2 και α2+β 2=2γ·μy τότε το τρίγωνο είναι ορθογώνιο και ισοσκελές.

ΜΔ2 - ΑΔ2 = 2ΜΑΚΟ = ΜΑ2(ΚΑ+ΑΟ) = ΜΑ (2ΚΑ+2ΑΟ) = ΜΑ (ΜΑ+ΑΓ) = ΜΑΜΓ 1 1 . Δίνεται τραπέζιο ΑΒΓΔ με (ΑΒ>ΓΔ). Να δειχθεί ότι: ΑΓ2+ΒΔ 2=ΒΓ2+ΔΑ2+2ΑΒ·ΓΔ

ΑΒ//ΓΔ

Α πό δε ι ξη

Έστω Μ το μέσο της ΑΓ και Ν το μέσο της ΒΔ τότε ΜΝ = ΑΒ - ΓΔ (l) 2

Α πό δε ι ξη

2 2 2 4 μα2 = α 2 + 2 γ 2 => 4 2 β + 2γ - α 4 Δ

α = β => Α Β Γ ισοσκελές. Από υπόθεση έχουμε: α2 + β 2 = 2γ · μ r

(l) Β

Α

Από (1° Θ. Δ.) έχουμε: 2 α 2 + β 2 = 2 · μ 2 + r_ 2

Δ

(2)

Ύ

1 0. Δίνεται ρόμβος ΑΒΓΔ και Μ σημείο της προέκτασης της διαγωνίου ΓΑ προς το Α. Να δειχθεί ότι ΜΔ2 - ΑΔ 2 = ΜΑ · ΜΓ Α πόδ ε ι ξη :

Έστω Κ το μέσο του ΜΑ και Ο το σημείο το­ μής της διαγωνίου του. Τότε ΟΑ < ΟΜ => ΔΑ < ΔΜ . Μ

Δ

Εφαρμόζουμε το {1° Θ . Δ.) στα Α Β Δ, ΒΓ Δ , ΝΑ Γ οπότε ΑΒ 2 + ΑΔ2 = 2ΑΝ 2 + ΔΒ 2 (2) 2 2 ΒΓ 2 + ΓΔ2 = 2ΓΝ 2 + ΔΒ (3) 2 2 ΑΝ 2 + ΓΝ 2 = 2ΜΝ 2 + ΑΓ (4) 2 Δ

(+) ( 2 ),( 3 )=> ΑΒ 2 + ΒΓ2 + ΓΔ2 + ΔΑ2 = <4> ΑΓ-2 + ΔΒ 2 = 2 ( ΑΝ 2 + ΓΝ 2 ) + ΔΒ 2 = 2 2ΜΝ 2 + 2 2 (ι) = 4ΜΝ2 +ΑΓ2 + ΔΒ2 =4 ΑΒ -ΓΔ + ΑΓ2 + ΔΒ2 2 Άρα: ΑΒ2 + ΒΓ2 + ΓΔ2 + ΔΑ 2 = = ΑΒ2 - 2ΑΒ · ΓΔ + ΓΔ2 + ΑΓ2 + ΔΒ2 Οπότε: ΑΓ2 + ΔΒ2 = ΒΓ2 + ΑΔ2 + 2ΑΒ · ΓΔ

(

)

(

)

12. Έστω ισόπλευρο τρίγωνο ΑΒΓ πλευράς α και ευθεία ε//ΒΓ που διέρχεται από το κέ­ ντρο βάρους θ του τριγώνου ΑΒΓ. Αν Μ σημείο της ε, να δειχθεί ότι ΜΒ 2 + ΜΓ2 = 2ΜΑ2 Α πό δε ι ξη

Γ Εφαρμόζουμε (2° Θ. Δ.) στο Μ Δ Α : Δ

Έστω Δ το ίχνος του ύψους ΑΔ=υ. Τότε 2υ <=> α 2 = αJ3 <=> α = 4υ 2 (l) υ = -3 2 J3 ΑΘ = � υ (2) 3

ΕΥΚΛΕΙΔΗΣ Β ' τ. t /52


Μαθηματικά για την Β ' Λυκείου

1 (3) ΘΔ = -υ 3

χουμε: B = f > fΊ � ΜΓ > ΜΒ � ΜΓ2 - ΜΒ 2 = 2α · ΚΛ , οπότε αρκεί 2 ΚΛ = 1 , η, ΚΛ = 21 ΜΑ ΜΑ

Α

Α Μ

Β

( ε)

Δ

Εφαρμόζω το (1° Θ. Δ.) στο Μ Β Γ οπότε: Β Γ Λ Κ ΒΓ 2 ΜΒ 2 + ΜΓ 2 = 2ΜΔ2 + -ΚΛ ΚΒ 2 Πράγματι: ΜΚ // ΑΛ � = ΜΒ = 21 , αφού ΜΑ α 2 (=ι ) 2ΘΔ2 + 2ΜΘ 2 + 2 υ 2 = 2 ( ΘΔ2 + ΜΘ 2 ) + 2 1 } Μ, = 30° � ΚΒ = 2 ΜΒ 2 � 2 � + 2 ( ΑΜ 2 - ΑΘ 2 ) + % υ 2 ii) Από την ( 1) παίρνουμε: ΜΓ + ΜΒ = α = ΒΓ > 1 (=2 ) 2 υ2 + 2ΑΜ 2 - 2 3 2υ 2 + 2 υ 2 ΜΑ ΜΓ - ΜΒ ΜΓ - ΜΒ 3 9 ΜΓ - ΜΒ = α = ΒΓ < 1 ΜΑ ΜΓ + ΜΒ ΜΓ + ΜΒ = � υ 2 + 2ΑΜ 2 - � υ 2 + � υ 2 = 2 · ΑΜ 2 • 9 9 9 τριγωνική ανισότητα στο Μ Β Γ 13. Δίνετ αι ισόπλευρο τρίγωνο ΑΒ Γ με πλευρά Άρα ΜΓ+ΜΒ>ΜΑ>ΜΓ - ΜΒ, οπότε υπάρχει Μ τυχα ίο εσωτερικό σημείο της πλευ­ α και τρίγωνο με πλευρές ΜΑ, ΜΒ, ΜΓ ράς ΑΒ. Ν δείξετε ότι:

}

Α

()

( )

(

)

α

i)

ΜΓ2 - Μ 2 Β Μ

Α

β ' τρ όπος :

(1)

και ii) υπ ά ρχει τρ ίγωνο με πλευρ ές ΜΑ, ΜΒ, Μ Γ Απ ό δ ε ι ξη

Έστω ΑΛ, ΜΚ .l ΒΓ i) Εφαρμόζουμε το (1° Θ. Δ.) Μ ΒΓ και έ-

Δ

Αν ΜΝ//ΒΓ, τότε το Α Μ Ν είναι ισόπλευρο και το ΜΒΓΝ ισοσκελές τραπέζιο. Επομένως ΜΝ=ΜΑ, ΝΓ=ΜΒ. Άρα το Μ Ν Γ είναι το ζητούμενο τρίγωνο. Δ

Δ

a Μετρικές Σχέσεις

στον

Κύ κ λ ο Θ ανάσης Τ σιούμ ας

Τέμνουσ ε ς Κ ύ κλου

Αν θεωρήσουμε έναν κύκλο (O,R) και σημείο Σ του επιπέδου του, τότε αποδεικνύεται ότι το γινόμενο ΣΑΣΒ (όπου ΣΑΒ τέμνουσα του κύκλου) είναι σταθ ε ρό, δηλαδή ανεξάρτητο από τη θέση της τέμνουσας, αλλά εξαρτάται από τη θέση του Σ ως προς το κέντρο του κύκλου. Έτσι έχουμε τις εξής περιπτώσεις: Ι. Αν Σ ε σωτε ρικό του κύκλου και ΟΣ=δ τότε

ισχύει

ΕΥΚΛ ΕΙΔΗΣ Β' τ. Ι/53

Δ

Β Σχ. 1


Μαθηματικά για την Β ' Λυκείου

Π.

Σ εξωτερικό του κύκλου τότε ισχύει

και ΑΣ εφ απτόμενο τμήμα . Σ

Σ Μ Σχ. 2

Β

ΣΑ-ΣΒ=ΣΓ ·ΣΔ= ΣΜ2= δ2 -R2 όπου δ=ΟΣ και ΣΜ το εφαπτόμενο τμήμα Απ άντη ση

Ση μείωση

Η τέμνουσα ΣΑΒ κατά την κίνησή της γύρω από το Σ παίρνει την οριακή θέση ΣΜ γι' αυτό έχουμε ΣΑ · ΣΒ = ΣΜ·ΣΜ= ΣΜ2 Π αρατη ρή σεις

Ι . Η ισότητα ΣΑ ΣΒ = ΣΓ ΣΔ εκφράζει το θεώρημα των τεμν όμενων χορδών, που έχει εφαρμογή σε υπολογισμούς τμημάτων που τε­ λειώνουν σε κύκλο. ·

·

Α

Είναι ΒΓ · ΒΡ=ΒΑ2 (μετρικές σχέσεις στα ορ­ θογώνια τρίγωνα) Και ΒΔ· ΒΣ= ΒΑ2 (μετρικές σχέσεις στα ορθο­ γώνια τρίγωνα) Άρα ΒΓ·ΒΡ=ΒΔ·ΒΣ οπότε τα Σ,Ρ,Γ,Δ είναι ομοκυκλικά. Ση μείωση

Αν ΣΡ2 =ΣΑ · ΣΒ όπου Σ,Α,Β συνευθειακά τότε ο κύκλος που διέρχεται από τα Ρ ,Α,Β είναι ε­ φαπτόμενος της ΣΡ. Δ ύ ναμη ση μείου ως προς κύ κλο.

ΑΓ=5 ΒΓ=6 ΑΣ=χ ;

Γ

Λύ ση

Έχουμε ΓΣ·ΓΑ=ΓΜ·ΓΒ ή (5-χ) -5=3·6� x=l ,4 2. Ισχύει και το αντίστροφο του θεωρήματος των δηλαδή αν χορδών, τεμνόμενων ΣΑ·ΣΒ=ΣΓ · ΣΔ όπου Σ, Α, Β και Σ, Γ, Δ συ­ νευθειακά σημεία, τότε τα Α, Β, Γ, Δ είναι ο­ μοκυκλικά.

Η δύναμη σημείου Σ ως προς κύκλο (O,R) ορίζεται ως εξής: IΔ�o, R > = ΟΣ 2 - R 2 1 Έχουμε τις περιπτώσεις: Αν Σ εξωτερικό του κύκλου (δ>R) i) (σχ.2) τότε η Δ�ο, R > συμπίπτει με το γινόμενο ΣΑ-ΣΒ και το ΣΜ2 , δηλαδή έ­ χουμε ΔΣ(O, R ) = ΣΑ · ΣΒ = ΣΓ · ΣΔ = δ 2 - R 2 = ΣΜ 2 > 0 Αν Σ εσωτερικό του κύκλου (δ<R) ii) (σχ. Ι) τότε Δ�ο, R > = -ΣΑ · ΣΒ = -ΣΓ · ΣΔ = δ2 - R 2 < Ο iii) Αν Σ είναι π άνω στον κύ κλο τότε Δ�Ο, R ) = ο Π αράδειγμα

Π αρ άδειγμα

Γιατί στο π α ρ ακάτω σχήμα το ΣΡΓΔ είν αι εγγράψ ιμο σε κύκλο; όπου ΑΒ διάμετρος

Έ στω Ρ τυχαίο σημείο χορδής ΑΒ κύκλου (0, R) να αποδείξετε ότι: ΟΒ 2 = 0Ρ 2 + ΡΑ · ΡΒ

ΕΥΚΛΕΙΔΗΣ Β' τ.Ι /54


Μαθηματικά για την Β ' Λυκείου

(Ι). Ομοίως το ΑΡΣΔ είναι εγγράψιμο άρα Το Ρ είναι εσωτερικό σημείο του κύκλου άρα ΒΣ·ΒΔ=ΒΡ·ΒΑ. Με πρόσθεση κατά μέλη των (1), Δ�ο,R> = ΟΡ 2 - R 2 = -ΡΑ · ΡΒ . (2) έχουμε: ΑΣ· ΑΓ +ΒΣ· ΒΔ=ΑΒ(ΑΡ+ΡΒ)=ΑΒ· ΑΒ=ΑΒ 2= Επομένως ΟΡ 2 + ΡΑ · ΡΒ = R 2 = 08 2 (2R2 )= 4R2 Λύσ η

2.

Σχόλιο .

Λύση

Οι μετρικές σχέσεις του κύκλου, μας βοηθούν να κατασκευάσουμε γεωμετρικά τις θετικές ρίζες ορισμένων Β/θμιων εξισώσεων. Γνωστό είναι το πρόβλη μα της χρυ σής τομής: δηλαδή να διαιρε­ θεί ένα δοσμένο τμήμα ΑΒ=α σε μέσο και άκρο λόγο, δηλαδή χ= ; (χ<α) ώστε χ2=α(α-χ) προκύπτει:

Από το θεώρημα τέμνουσας και εφαπτομένης έχουμε ΣΜ2 =ΣΒ·ΣΑ (1)

ι� � � � φι (χρυσός λόγος) αρχικό της

λέξης Φειδίας. Επειδή ο χρυσός λόγος καθορίζει τις τέλειες αναλογίες σε ένα σώμα, χρησιμοποιή­ θηκε κυρίως στην τέχνη. Ακόμα και ο αριθμός των κερκίδων, του κάτω διαζώματος, προς των αριθ­ μών των κερκίδων του πάνω διαζώματος του Αρ34 Φ ' λογο ' 2J , θ ε ατρου της Ε πι δ α υ ρου εχει χαιου Το παραπάνω θέατρο όπως είναι γνωστό έχει εκ­ πληκτική ακουστική. '

'

=

Α

Σ

Έστω ΣΒ=χ τότε ΣΜ= 2χ και ΣΑ= 6+ χ οπότε η (1) γράφεται (2χ)2= χ· (6+ χ) <::::> 4χ=6+ χ<::> 3χ=6<::> χ=2. Άρα το Σ απέχει από το κέντρο απόσταση ΟΣ=5.

.

3.

Λ υ μένες Ασκήσει ς Ι.

Δίνεται κύκλος (0,3). Στην προέκταση της διαμέτρου ΑΒ, να βρεθεί σημείο Σ, τέτοιο ώστε το εφαπτόμενο τμήμα ΣΜ, να είναι δι­ πλάσιο του ΣΒ.

Δίνετ αι κύκλος διαμέτρου ΑΒ=2 και οι χορ­ δές του ΑΓ και ΒΔ που τέμνονται στο Σ. Ν α αποδείξετε ότι ΑΣ·ΑΓ+ΒΣ·ΒΔ=4 R2

Δίνεται τρίγωνο εγγεγραμμένο σε κύκλο (0, R). Έ στω ΑΜ η διάμεσος του , που τέμνει τον κύκλο στο Ρ και Θ το βαρύκεντρο του τριγώνου. Να αποδειχτεί ότι: α2 i) ΜΑ · ΜΡ = 4 _ .!_ ( 2 β 2 γ 2 ) e α + + ii) Δ (O,R) = 9 Λύση

Από το θεώρημα των τεμνόμενων χορδών έ­ α2 (1) χουμε: ΜΑ · ΜΡ = ΜΒ · ΜΓ = -α2 · -α2 = 4 ii) Το Θ είναι εσωτερικό του κύκλου (0, R). Άρα Δ�ο . R > = -ΘΑ · ΘΡ = -ΘΑ ( ΘΜ + ΜΡ ) = Λύσ η Φέρουμε ΣΡ l. ΑΒ τότε το τετράπλευρο ΣΡΒΓ = -ΘΑ · ΘΜ - ΘΑ · ΜΡ = είναι εγγράψιμο σε κύκλο διότι και f = 90° (εγγε­ α2 (2) 2 2 -α2 = --μα 2 2 --2 Ι --2 ΜΑ · .ΜΡ=ι• >--μα 2 α ·-μα =--μ γραμμένη σε ημικύκλιο) οπότε ΑΣ·ΑΓ=ΑΡ-ΑΒ 3 3 3 3 34 3 6 i)

ΕΥΚΛΕΙΔΗΣ Β ' τ. t /55


Μαθηματικά για την Β ' Λυκείου

=2ΡΕ 2 + 2ΑΡ 2 =2(ΡΕ 2 + ΑΡ 2 )=2ΑΕ2 = =ΑΕ2 + ΑΖ2 αφού ΑΕ=ΑΖ 5

Ρ ,Όμως μα2 = 2β 2 + 2γ 2 - α 2 (3) 4 Από (2), (3) έχουμε : 2 2β 2 + 2γ 2 - α2 - 6 α2 - -91 ( α 2 + β 2 + γ 2 u < O . R J - -3 4 λ0

4.

_

(

)

Έ στω κύκλος (Ο, R) και ορ θ ογώνιο τρίγωνο ΑΒΓ εγγεγρ α μμένο στον κύκλο, με ΑΒ=2. Προεκτείνου με την ΓΑ κατά ΑΣ=2. Αν το εφαπτόμενο τμήμα ΣΜ είναι ΣΜ=4 να υπο­ λογίσετε α) τη χορδή ΑΓ β) την ΟΣ γ) την προβολή του ΑΒ πάνω στη ΒΓ Σ

_

Γ

Δίνετ αι τρίγωνο ΑΒΓ (ΑΒ<ΑΓ) εγγεγρ α μμέ­ νο σε κύκλο (0, R). Από το μέσο Μ τη ς ΒΓ φέρου με ΜΡ κάθ ετη στη διάμετρο ΑΔ, που τέμνει τον κύκλο στ α Ε και Ζ. Ν α αποδείξε­ τε ότι: i) το τρίγωνο ΑΕΖ είναι ισοσκελές ii) ΑΒ2+ΑΓ2=ΑΕ 2+ΑΖ2

Λ ύσ η

α) Από το θεώρημα τέμνουσας και εφαπτομέ­ νης έχουμε: ΣΜ 2 =ΣΑ . ΣΓ<:::>4 2=2(2 +Α Γ)<:=> I ΑΓ=6 1 β) Υπολογίζουμε τη διάμετρο από το Π. Θ. στο ορθογώνιο τρίγωνο ΑΒΓ είναι ΒΓ2=ΑΒ2+ΑΓ2 ή ΒΓ2=2 2+62<:::> IBΓ =2MI δηλαδή R = Μ . Επειδή ΣΜ2=ΣΟ2-R2<:::> 42 =Σd -fι02 �ΣΟ=J261

Λ ύ ση

Επειδή η ΟΡ, άρα και η ΑΡ είναι κάθετη στη χορδή ΕΖ τότε το Ρ θα είναι το μέσο της ΕΖ , επομένως η διάμεσος ΑΡ είναι και ύψος του τριγώνου ΑΕΖ οπότε αυτό θα είναι ισοσκελές. ί)

ΒΔ = προβ �� . γ) Έστω Έχουμε 2 ΑΒ = ΒΔ · ΒΓ (μετρικές στα ορθογώνια τρί­

Ι ��·

γωνα) ή 2 ' � ΒΔ · 2 Ji0 <ο> ΒΔ 6

Εφαρμόζουμε το 1 ο θεώρημα της διαμέσου στο τρίγωνο ΑΒΓ και έχουμε ΑΒ 2 +ΑΓ2 =2ΑΜ 2 +2ΒΜ 2 ( 1 ) Είναι ΒΜ 2 =ΒΜ · ΜΓ=ΜΕ·Μ Ζ= =(ΡΕ - ΜΡ)(ΡΖ + ΜΡ)= =(ΡΕ - ΜΡ)(ΡΕ + ΜΡ)=ΡΕ2 - ΜΡ2 (2). (ΡΕ = ΡΖ ) Από (1 ), (2) προκύπτει ότι ΑΒ 2 +ΑΓ2 = =2ΑΜ2 + 2ΡΕ2 - 2ΜΡ2=2ΡΕ2 + 2(ΑΜ2 -ΜΡ2) ii)

Θεωρούμε κύκλο (0, R) και ΑΒ μια διάμε­ τρ ός του. Αν Μ τυχαίο σημείο του κύκλου (διαφορετικό των Α,Β) και Ρ,Σ τα μέσα των ακτίνων ΟΑ και ΟΒ αντίστοιχα, να αποδει­ χθ εί ότι:

i) ii)

3R2 --

R2

3 και ΣΛ = -4ΜΡ 4ΜΣ ΜΡ ΜΣ 1 Ο + = ΡΚ ΣΛ 3

ΡΚ =

Λ ίJ σ η

ί)

Από το θεώρημα των τεμνόμενων χορδών έ­ χουμε:

ΕΥΚΛΕΙΔΗΣ Β' τ. Ι /56


Μαθηματικά για τη ν Β ' Λυκείου

1 3 R = -R 3 2 ΡΚ · ΡΜ = ΡΑ · ΡΒ = -R 2 2 4 ' 3R 2 οπότε ΡΚ = -(1). 4ΜΡ 3 1 = -R 3 2. ' ΜΣ · ΣΛ = ΑΣ · ΣΒ = -R-R Ομοιως 2 2 4 2 3R Άρα ΣΛ = (2) 4ΜΣ · -

Αν φέρουμε τη ΒΓ τότε B rΑ = 90° ως εγγεγραμ­ μένη σε ημικύκλιο, άρα το τετράπλευρο ΓΒΣΕ εί­ ναι εγγράψ ιμο, αφού και ΑΣΕ = 90° , Επομένως: ΑΒ · ΑΣ = ΑΓ · ΑΕ (3). Από (2), (3) προκύπτει ότι ΑΓ · ΑΕ = ΣΑ2 - ΣΓ 2 •

Μ

ii)

8) Θεωρούμε οξυγώνιο τρίγωνο ΑΒΓ και ΑΔ α2 το ύψος του. Αν ΑΗ · ΑΔ = (1) όπου Η 2

Από (1), (2) έχουμε:

το ορθόκεντρο του τριγώνου, να αποδείξετε α J3 2 2 2 ότι: i) β + γ = α 2 ii) μα = 2-

Όμως η ΜΟ είναι διάμεσος του τριγώνου ΜΡΣ, άρα από το 1 ο θεώρημα της διαμέσου είναι: 2 2 2 MPz + ΜΣz = 2ΜΟ2 + ΡΣ = 2R 2 + R = 5R (4 ) 2 2 2 Από τις (3), (4) προκύπτει ότι ΜΡ + ΜΣ = 4 · 5R 2 = 10 . 2 ΡΚ ΣΛ 3 · 2R 3 7

Σε κύκλο (0, R) η εφαπτομένη σε τυχαίο του, σημείο Γ, τέμνει την προέκταση της δι­ αμέτρου ΑΒ στο Σ. Αν η κάθετη στην ΑΒ στο Σ, τέμνει την ευθεία ΑΓ στο Ε, να απο­ δείξετε ότι: ΑΓ ΑΕ = ΣΑ 2 - ΣΓ 2 ·

Λύση

Το Σ είναι εξωτερικό του κύκλου οπότε Δ�ο, R > = ΣΓ2 = ΣΒ · ΣΑ (1 ). Άρα ΣΑ 2 - ΣΓ 2 = ΣΑ 2 - ΣΒ · ΣΑ = = ΣΑ ( ΣΑ - ΣΒ ) = ΣΑ · ΑΒ (2).

Λύση

i) Φέρουμε το ύψος ΒΕ, επειδή Α < 90° από το θεώρημα της οξείας γωνίας θα έχουμε: α2 = β 2 + γ 2 - 2ΑΕ · ΑΓ <::> 2 2 2 <:::> β + γ = α + 2ΑΕ · ΑΓ (2). Α

Β

Όμως το τετράπλευρο ΗΔΓΕ είναι εγγράψιμο <ι>= αz οπόΔ Ε = 1 80° άρα ΑΕ · ΑΓ = + ΑΗ · ΑΔ τ ( ) τε η (2) γίνεται: β2 + γ 2 = α2 + 2 �2 ή β2 + γ 2 = 2α2 • 2 2 2 2 + 2 2γ α ·· ' μα2 = β ι ι ) Ε ιναι η, 4 2 ( β 2 + γ 2 ) - α 2 2 · 2α 2 - α 2 3α2 , . 2 = = 4 απο (ι). μα = 4 4 . Άρα μα = α.J3 2

ΕΥΚΛΕΙΔΗΣ Β ' τ. Ι /57


Μαθηματικά για την Β ' Λυκείου

Μαθη ματικά Κατεύ θυνση ς Διανύσματα

Θάνος Χαραλάμπους - Βαγγέλης Ευσταθίου

Ένα πλήθος ασκήσεων του Διανυσματικού Λογι­ σμού αντιμετωπίζεται με την χρήση της άσκησης 4, σελίδα 28 του Σχολικού βιβλίου, σύμφωνα με την οποία:

β2 γ2 ΟΓ ΟΔ = 2 + ΟΒ β + γ2 β2 + γ2 Σχόλιο :

Αν ΔΒ = μ και Δ στην προέκταση του ΒΓ ΔΓ λ δηλαδή ΔΒ ίί ΔΓ τότε ΔΒ � ΔΓ οπότε λ λ λΔΒ - μΔΓ = δ και ΟΔ = ---ΟΒ - -μ-ΟΓ λ μ λ-μ Γ 'Ε , , ΔΒ = λμ με τσι γενικοτερα αντι, της σχεσης Αν Δ είναι σημείο μεταξύ των Β και Γ τέτοιο ΔΓ = ώστε �� t (Ι) όπου μ,λ>Ο, τότε για κάθε ση­ μ,λ>Ο μπορούμε να θεωρήσουμε την χΔΒ + ψΔΓ = δ με χ,ψ ε JR* μείο Ο, θα έχουμε: ο

=

� = t <=> ΔΒ = - t: ΔΓ <::> λΔΒ + μΔΓ = δ (1) ΔΒ i-1- ΔΓ <=> λ ( ΟΒ - ΟΔ ) + μ ( ΟΓ - ΟΔ ) = δ <::> ΟΔ = λ +λ μ ΟΒ + λ +μ μ ΟΓ <=> ΟΔ = λ � μ ( λΟΒ + μΟΓ )

}

- -

Άρα από την λΔΒ + μΔΓ ΟΔ // ν

=

=

Ο

προκύπτει ότι

λΟΒ + μΟΓ .

Άσκηση Ι η

Έστω τρίγωνο ΑΒΓ και Ι το έγκεντρό του. Να υπο λογισθεί το διάνυσμα θέσης του εγκέντρου Ι, συναρτήσει των διανυσμάτων θέσης των κορυ­ φών του τριγώνου. Λ ύση

Σύμφωνα με τα προηγούμενα, β-ΟΒ + -γ-ΟΓ ΟΔ = β+γ β+γ όπου Δ το ίχνος της εσωτερικής διχοτόμου της γωνίας Α . �

Ε φ αρμογές

Ομοίως από το τρίγωνο ΑΒΔ, έχουμε:

α) Αν ΟΔ διάμεσος τότε μ = λ οπότε:

- λ - λ - 1- 1ΟΓ = l OB + l OΓ ΟΒ + ΟΔ = lλ lλ ΔΒ β , , , , β ) Αν ΟΔ δ ιχοτομος, τοτε ισχυει: = γ , αρα ΔΓ λ β = . μ γ Επομένως από την (1), θα έχουμε: γ β ΟΔ = __ ΟΒ + __ Ο Γ β+γ β+γ

ΔΓ

ΟΓ

--

-

Δηλαδή αΑΙ + (β + γ)ΔΙ

=

Ο

α ( ΟΙ - ΟΑ ) + (β + γ) ( ΟΙ - ΟΔ ) = δ <=> ΟΙ = α + (βα + γ) ΟΑ + α +β(β+ +γ γ) ΟΔ <=> ΟΙ = α + αβ + γ ΟΑ + α + ββ + γ ΟΒ + α + γβ + γ or = + γΟΓ = αΟΑ α+ +βΟΒ β+γ

(

Bor = 90° και ΟΔ είναι το ύψος τότε: μ γ2 ΔΒ ΟΒ 2 , = = pz · . Ετσι λ 2

γ) Αν

αγ α ΙΔ ΒΔ = β + γ = -ΙΑ = ΑΒ γ β + γ

- - -

Επομένως από την (1 ), προκύπτει: ΕΥΚΛΕΙΔΗΣ Β ' τ. l/58

)


Μαθηματικά για την Β ' Λυ κείου

Επομένως: - + ΑΓ 5ΑΒ ΓΟ = 265 263 ( 3ΑΒ ΑΟ · )( - - 7ΑΓ ) - 2 - 16ΑΒ -·1 5 ( 1 5ΑΒ -2 ) =ΑΓ 7ΑΓ 2 26 1 5 ( 1 5α2 - 16α2 συν60 - 5α2 ) =261 52 21 2 =0 =262 1 5α - 1 6α -2 - 5α -

(

Άσκη ση 2 '1

Δίνεται ισόπλευρο τρίγωνο ΑΒΓ και σημείο Ο του επιπέδου του, ώστε να ισχύει: 60Α + 1 50Β + 50Γ = Ο (1), Να αποδείξετε ότι: ΟΑ .l ΟΓ . -

-

-

-

-

z)

zος τρόπος ( Μ ε σ υντεταγμένες)

Θεω ρούμε ορθοκανονικό σύστημα συντεταγα ./3 ) με, νων χΒψ με Γ(α,Ο) και Α ( 2α , -2 - . Υ

Λύση 1 "' τρόπος

Αν v = 1 50Β + 50Γ τότε σύμφωνα με τα όσα α­ ναφέραμε προηγουμένως θα είναι ν// ΟΔ όπου Δ σημείο.τέτοιο ώστε: 1 5ΔΒ + 5ΔΓ = Ο (2), (προφανώς επί της ΒΓ) και μάλιστα Γ(α,Ο) χ 1 > -6ΟΑ 50Β 1 5 5 1 < 5ΟΓ + ΟΔ = 20 ΟΒ + 20 ΟΓ = 20 = ----w--Τότε από την 60Α + 1 50Β + 50Γ = δ = - 103 ΟΑ. προκύπτει : 6ΑΟ + 1 5ΒΟ + 5ΓΟ = δ , οπότε - 3Άρα ΟΔ = - 10 ΟΑ (3). 6 χ - % , ψ - α + 1 5(χ,ψ) + 5(χ - α, ψ) = (0, 0) Α 3J3α 4α , Άρα : χ = u Ψ = 13 . -

-

-

(

�)

'Ετσι ΑΟ = - �:( 1, 2./3 ) και ΓΟ = �:( -6, ./3 ) 2 ΓΟ = - 1265α2 ( -6 + 2 · 3 ) = Ο ΑΟ Επομένως : ·

Β

Γ

Έχουμε: - (4) 1 0 ΑΔ -= 3 = ΑΟ � ΑΟ ΑΔ - ΑΟ (3) � Ϊ3 10 -+ Ι­ 5 15 Επίσης ( 2 ) � ΑΔ = 20 ΑΒ + 20 ΑΓ = 43 ΑΒ 4 ΑΓ και - = 10 3 5 ( 3ΑΒ -+1= Ar ΑΓ ) + AB ( 4 ) � ΑΟ 4 4 u 26

(

)

Άρα: ΓΟ = ΑΟ - ΑΓ = ;6 ( 3ΑΒ + ΑΓ ) - ΑΓ = = �6 ( 5ΑΒ - 7ΑΓ ) .

Ά σκ η ση 3 η Αν για τυχαίο σημείο Ο στο επίπεδο ισο­ πλεύ ρου τριγώνου ΑΒ Γ ισχύει κΟΑ + λΟΒ + μΟΓ = Ο (1) τότε να β ρεθεί η ικανή και αναγκαία σχέση μεταξύ των κ,λ,μ ώστε ΟΑ .l ΟΓ Λύ ση

κΑΟ + λΒΟ + μΓΟ = δ � κ χ - % ,ψ- α + λ( χ,ψ) + μ( χ - α,ψ) = (Ο,Ο) �

(

ΕΥΚΛΕΙΔΗΣ Β ' τ.l/59

�)


κα + μα (κ + λ + μ)χ = 2 κα.J3 (κ + λ + μ)ψ = -2

}

Μαθηματικά για την Β ' Λυκείου

( � + μ)α χ=

Ά σκη ση 511

Δίνονται τα μη συγγραμμικά διανύσματα

α, β

κ+λ+μ με I α I 2 . Να δειχθεί ότι: α.J3 κί) προ β� -ψ = κ + λ2+ μ 2 Επειδή Α, Β, Γ είναι μη συγγραμμικά θα ισχύει ίί) Αν για ένα διάνυσμα ισχύει: κ + λ + μ Ο . Έτσι: ( α · � ) � = 12προ β� + 4β (t) α .J3 �2 + μ α κ ι . χ = 3α + λ β ΑΟ = κ + λ + μ - �2 ' κ +-2λ-+ μ _ α.J3 2 = 2. Αν το διάστυμα που ικανοποιεί την ( ) (1 ) είναι μοναδικό τότε να βρεθεί το λ. = 2 ( κ :� + μ) λ - μ, (λ + μ) J3 Α πόδειξη �+μ α κ αJ3 Έστω � = u + ν , όπου u προβ� , άρα l_ α . και ΓΟ = κ + λ + μ - α' κ + λ2+ μ = Τότε u 1 1 α άρα υπάρχει λ ]R* ώστε u = λα ή α · u = λα 2 . i ) Ό μω ς α · � = α ( u + ν ) = α · u + α · ν , αλλά � + λ α κ α/J3 = κ+λ+μ 'κ+ +μ = α . ν = ο , άρα α . � = α . υ i ) : ( = - 2(κ + + μ) κ + 2λ,-κ J3 Τότε ΑΟ l_ ΓΟ ( λ - μ)( κ + 2λ) - 3κ( λ + μ) = Ο ά κλ + 2λ2 - κμ - 2λμ - 3κλ - 3κμ = Ο α·χ Έτσι έχουμε α- · χ- = λα- z ή λ = --=-z · λ2 - (κ + μ)λ - 2κμ = Ο (Ι) α π . χ. Για κ = 6, λ = 1 5 παίρνουμε μ = 5 - α·χ- α·χΤελικά u = --=-zα ή προ β � = --=-zα α α Άσκη ση 4η ίί) ( 1) ::::} αχ Ο διαφορετικά θα έχουμε β = Ο Δίνονται τα συνεπίπεδα διανύσματα β, γ για δηλαδή βιια πράγμα άτοπο . Άρα: τα οποία ισχύει ι α ι = j β j = IΎI = .J2 και (1) ::::} (α�)� = 12 αχ4 α + β ::::} � = 3α + _4_ β 2r ( α + β ) = α · β + 6 Να αποδειχθεί ότι α + β = 2r αχ - - ::::} χ = 3α + λβ Απόδειξη 2 . Ισχύει: ( α · � ) � = 12προβ� + 4β Αρκεί να αποδείξουμε ότι α + β - 2r = Ο . ( α · � ) � · α = ι 2α · προ β � + 4α . β Όμως για να αποδείξουμε ότι ένα διάνυσμα ισού­ ( α . � ) � . α = ι 2α . � + 4α . β ται με το μηδενικό διάνυσμα, αρκεί να δείξουμε ότι το μέτρο του είναι ίσο με το μηδέν . ( α . � ) 2 = 12α . � + 4α . β Αρκεί λοιπόν να δείξουμε ότι I α + β - 2y I = ο . ( α . � ) 2 - 12α . � - 4α . β = ο Έχουμε: Θέτουμε α · � = ω και η τελευταία γίνεται 1 α + β - 2::ΥΙ 2 = α2 + β 2 + 4::Υ 2 + 2α . β - 4β · ::Υ - 4::Υ · α ω2 - 12ω - 4α . β ο Αφού το χ είναι μοναδικό θα είναι και το = Ι α1 2 + Ι β 1 2 + 4Ι Ύ Ι 2 + 2[ α . β - 2::Υ( α + β )J α· χ = ω μοναδικό . Συνεπώς η εξίσωση = 2 + 2 + 4 . 2 + 2( -6) = ο

[(

*

=

-

α

α . χ α α

χ

)

[( [(

=

J

)

)

J

χ

J

ν

=

Ε

::;Ζ:

α,

=

ΕΥΚΛΕΙΔΗΣ Β ' τ. Ι /60


Μαθηματικά για την Β ' Λυκείου

ω2 - 12ω - 4α · β = Ο θα έχει διπλή ρίζα, άρα Δ = ο � ( 12 ) 2 - 4 ( -4α . β ) = ο � 144 + 16α . β = ο � α . β = -9 Άρα έχω ω2 - 12ω + 36 = Ο � ( ω - 6 / = Ο � ω=6 � α·χ = 6 -

α, β

είναι δύο μη συγγραμμικά διανύσμα-

τα, να αποδείξετε ότι για κάθε διάνυσμα επιπέδου των α, β ισχύει η σχέση : -2

α·χ α·β Χ=

}

-

'

Ά σκηση 6'1

Εάν

Απόδε ι ξη

Εφόσον τα α, β είναι μη συγγραμμικά, άρα θα υπάρχουν μοναδικοί- κ, λ-ώστε-: χ = κα + λβ χ . = κ α 2 + λα . β \ + α . βλ = α . χ α α ' Άρα: - - - - � - -2 α · βκ + λβ = β · χ β · χ = κα · β + λβ 2 ως Λύνουμε - προς - - κ,- λ: - 2 το- σύστημα -2 α·χ α·β α α · β α α·χ = =- D= D D λ ' β·χ β α·β β·χ α . β β2 Όμως D = α 2 β 2 - ( αβ ) 2 > Ο , αφού

β·χ -2

α

-2

β

α+

α·χ

α·β β·χ -2

α

α·β

α

α·β

α·β

β

α·β

β

-2

β

χ

του

K

-

-2

α � β 9� =l � l βl ω.(�)l < l � l βl =>cα.βy < a2 . β2•

DK , λ = Dλ 'Ετσι: κ = 0 D D λ βDκ α + Οπότε: χ = D D

-2

.,. ΕΠ ΙΣΤΗΜΟΝΙΚΑ Για ΑΕΙ, ΤΕΙ, ΙΕΚ, ΑΣΕΠ ��>

Ι>

ΜΑθΗΜΑΥΙΚΑ & ΠΟΙΧΕΙΑ ΠΑτΙΠΙΚΗΣ Γ' Ενιαfαυ Λυκεfου Διαφορικό<; Αοyιομός. Σtιmιmι<ή. Πιθανότητες

ΕΚΠΑΙΔΕΠΙΚΑ Για Δημοτικό, Γυμνάσιο, λύκειο, ΤΕΕ ΛΟΓΟΤΕΧΝΙΑ, ΜΕΛΕΤΕΣ, ΛΕΥΚΩΜΑΤΑ

ΜΑθΗΜΑΥΙΚΑ θετικ4ς & τεχνολ. Κατεuθ. Γ' Ενισfσυ Λuκεlοu Τdμας 1: Μιyαδtκοl οριθμοl, Όριο & συνέχεια σννόρτησης

-

ΜΑΟΗΜΑΥIΚΑ θετικ4ς & τεχνολ. Κστευθ. Γ' Ενισlου Λuκεlου Τόμος 2: Διαφορικό<; λΟ'(ΙΟμός. Ολοιιληρωnκός λο'(Ιομός

θΟΜΑΣ ΚΥΙΙΕΝΠΔΗΣ, ΛΙΙΙΠΕΡΛ ΜΛ8ΗΜΛΠΙΙΛ, 3 θlθΛΙΟΠΩΛ'ΕΙΟ • Ι<ΕΝΊ'ΡΙΚΗ ΑΙΑΘΕ:ΣΗ: ΑρμενοnοuλοU 'D · 546 35 Θεοσαλοli(κη • Τηλ. 2310..203.720 • Fax 231Q-2 1 1 .305 • e-mai: saleι@zili.g • θlθΝΟΠΩΛΕΙΟ .ΑΘΗΝΩΝ • •ΕΝΩΣΗ ΕΚΔΟmΝ ΒΙβΛΙΟV θΕΣΣΑΛΟΝΙΚΗLο Σtοό τοιι Βιβλίου (ΠεσμοζόγλοU 5) - 105 84 ΑθΗΝΑ • Τηλ..faχ 210-321 1 .097 • ΑΠΟΘΗΚΗ ΑΘΗΝΩΝ - ΠΩΛΗΣΗ ΧΟΝΔΡIΚΗ: Ασι<ληπιού 80 - Εξάρχεια 1 1 4 7 1 . Αθήνα • Τηλ.-Faχ 210-3818.1150 • e-mail: athinι@zili.gr •

ΕΥΚΛΕΙΔΗΣ Β ' τ.l /61


�-•ιιιι8-ιι8• [YO(JJ fJ[Jjf!J r fJc!J&j[[J f](f)[!) !Jr!Jωe56(f)r!J Μαθη μα,τικά Κατεύθυνσης Μιγαδικοί Αριθμοί Γεράσιμος Κουτσανδρέας

Λίγο πρ ιν : Είναι χρήσιμο να έχου με μια γενική εικόνα στους μιγαδικούς αριθμούς μέσα από μερικές ασκήσεις που θε ω ρούμε ότι έχουν ενδιαφέρον σε ση μεία που παρουσιάζουν δυ σκολίες . Έτσι οι κάποιες πρώτες παρατηρήσεις δίνουν ένα δείγμα ουσιαστικής παρέμβασης στο ξεκαθάρισμα εννοιών. Ακολουθούν ασκήσεις - Θέματα που υλοποιούν το γενικότερο πνεύ μα της συνολικής κάλυψης μέσα από θέματα πανελλαδικών εξετάσεων (εξετάσεις 2006) αλλά και επισημάνσεων που διαμορφώνουν μια καλή θεώρηση στους μιγαδικούς αριθμούς και βοηθούν πάρα πολύ τους αναγνώστες να έχουν μια πιο πλήρη κάλυψη ουσιαστικού ενδιαφέροντος. Π αρατηρή σ εις : •

Οι δυνάμεις ενός μιγαδικού αριθμού z με εκθέτη ακέραιο, ορίζονται ακριβώς όπως και στους πραγματικούς αριθμούς. Επίσης για z ο ορίζουμε z o = Ι και z - ν = _..!_ν _ , για κάθε ν Ε Ν . Ζ Δεν έχει νόημα η διάταξη στους μιγαδικούς αριθμούς. Δεν έχει νόημα το σύμβολο της � στους μιγαδικούς αριθμούς, δηλαδή δεν γράφουμε :;z:

..Γz .

αριθμοί να έχουν άθροισμα πραγματικό αριθμό χωρίς να είναι συζυγείς π. χ. z 1 = 2 + 3ί , Ζ 2 = 5 - 3ί . ii) z - z = 2βί .Δηλαδή η διαφορά δυο συζυγών μιγαδικών αριθμών είναι φανταστικός αριθμός. Το αντίστροφο δεν ισχύει. Υπάρχουν μιγαδικοί αριθμοί που έχουν διαφορά φανταστικό αριθμό χωρίς να είναι συζυγείς π. χ. z 1 = 5 + 3ί , Ζ 2 = 5 - i . iii) Αποδεικνύονται εύκολα οι προτάσεις z · � = α2 + β 2 z E R {:} z = z z ε Ι {::} Ζ = -z Δυο ίσοι μιγαδικοί έχουν ίσα μέτρα. Το αντίστροφο δεν ισχύει. Υπάρχουν άπειροι μιγαδικοί αριθμοί με το ίδιο μέτρο. · z = Ι + 2 ι· , z 3 = 2 - ι· κ.τ. λ . π .χ. z = 2 + ι, 1 2 -

-

Για τους συζυγείς μιγαδικούς αριθμούς z = α + βί και z = α - βί ισχύουν : i) z + z = 2α . Δηλαδή το άθροισμα δυο συζυγών μιγαδικών αριθμών είναι πραγματικός αριθμός. Το αντίστροφο δεν ισχύει: μπορεί δυο μιγαδικοί

ΕΥΚΛΕΙΔΗΣ Β' τ.Ι /62


Μαθηματικά για την Γ Λυκείου •

Για το μέτρο των μιγαδικών αριθμών ισχύουν: i) l z l = ι � ι = 1 -z l = ι -�ι ii) l z l 2 = z · � iii) l zΙ · z 2 Ι = l z � ll z 2 1 ί ν) l z v l = l zl v ν) l zl = Ο � z = Ο νί ) l l z � l - l z 2 1 1 � Ι z 1 + 2 2 l � l z Ι I + l z 2 1 Το μέτρο της διαφοράς δυο μιγαδικών αριθμών είναι ίσο με την απόσταση των εικόνων τους στο μιγαδικό επίπεδο. Βασικοί γεωμετρικοί τόποι των σημείων M(z) για τους οποίους ισχύει: i) l z - z0 1 = ρ, ρ > Ο , κύκλος με κέντρο το σημείο Α ( ) και ακτίνα ρ. ii) l z - z 1 1 = l z - z 2 1 , η μεσοκάθετη του ευθυγράμμου τμήματος με άκρα τα σημεία •

z0

A (z1 ) , B(z� ) .

ίίί ) l z - z 1 1 = ρ · lz - z 2 1 , ρ 1, ρ > Ο , απολλώνιος κύκλος. ίν ) l z - z 1 1 + 1z - z 2 1 = 2α, α > Ο, και lz 1 - z2 1 < 2 α , έλλειψη. ν) l lz - z 1 1 - l z - z 2 1 1 = 2α, α > Ο, και l z 1 - z2 1 > 2 α , υπερ βολή. 7:

Θ έμα 1 "

Θεωρούμε z1

τους

= _!_ + J3 i και -

2

2

μιγαδικούς z2

=

ι+

αριθμούς

z1

z

••. 11 1)

:=ι

2 ν+ Ι z2

= - Ζ ιν+2 '

z2 I

=

[-.!_ ί]2 2

+ J3 2

I

z22 // - ( -ΖΙ2 )2ιι -- ΖΙ4 // - ΖΙ ΖΙ - ΖΙ 1 - ΖΙ β) Λόγω της (ίί ί) έχουμε: z 326 = z 11 8 = ( z31 )6 = 1 6 = 1 + 0ι. 2 1 9 - 2 z.9+ Ι - 29 + 2 2 2 . 291 - 2 2 1 2 2 J3 ι = -z 2 = -21 + 2 μ

_

_

_

I

-

_

Jμ -

_

I

μ

_

Ι/

I

_

_

.

ο

I

Θ έμα 2"

+

Δίνεται ο μιγαδικός αριθμός z = χ yi, ..ι. , x,y Ε R και εστω . , z .,... =

f ( z) 2z-i:Zι z-

ι

__

α) Αποδείξτε ότι

f{l - i)

=

3 + 3ί.

β) Αποδείξτε ότι ο αριθμός

( f ( ι - i)}

2 004

είναι

πραγματικός αριθμός. γ) Έστω Α, Β οι εικόνες των μιγαδικών αριθμών και στο μιγαδικό

f (ι- i)

f (l +i)

2ν z2

=

z1

ν

=

ε

Ν*

)

.!_ - 2 . .!_ _ J3 i - i 4

3 - 3i = 3 + 3ι α) f ( 1 - ι ) = 2 ( 1 -1 i)- -i -i (l1 + i) = --i β) (f(l - i) ) 2004 = (3 + 3i) 2004 = (3(1 + i) ) 2004 = 0 3 2004 [ (1 + i)2 Γ 2 = 3 2004 (2i) I 002 = -2 1 002 3 2004 Ε R "

9 z •2

Είναι

I

Λύση :

β) Να γράψετε στη μορφή α + βί τους z�6 και Λύ ση :

Άρα J3 ί = Ο - .!_ (ί) 1 + z + z 2 = 1 - .!_2 + J3 ί 2 2 2 (ii) 1 + z 1 + z � = Ο {::} ( Ζ1 - 1 ) ( ι + Ζ1 + z � ) = Ο {::} {::} z� - 1 = Ο {::} z � = 1 . ( ίίί ) Είναι 1 + z 1 = -z � δηλαδή Επομένως : Ι z;v+l = (-� ) 2ν+ =-Ζι4ν+2 =-zι"+2 . z;v =-zι"+2 ( zΠ =

επίπεδο. Αποδείξτε ότι το τρίγωνο ΟΑΒ είναι ορθογώνιο στο Ο . (Ο η αρχή των αξόνων)

α) Αποδείξτε ότι: ί) ι + Ζ1 + z ; = 0

ii)

J3 . = --21 - -ι. 2

2 2

4

=

ο

ο

γ) f ( 1 +ι" ) = 2 (1 + i) - i(l - i) = 1+i-1 2 + 2i - i + i 2 = 1 -:- ί = 1 - ί ι

ΕΥΚΛΕΙΔΗΣ Β ' τ. Ι /63


Μαθηματικά για την Γ Λυκείου

άρα Α (3, 3), Β ( 1 , - 1 ) είναι Λύ σ η : ΟΑ · ΟΒ = (3,3)( 1 , - 1) = 3 - 3 = 0 άρα το τρίγωνο α) lz l 2 - ( Im(z) + 1 ) 2 + 1 = 0 {::} ΟΑΒ έχει Ο = 90° . {:} ( �x 2 + y2 ) 2 - (y + 1) 2 + 1 = 0 {::} 2 χ + y 2 - y 2 - 2y - 1 + 1 {::} = ο {::} Χ 2 - 2y = ο {::} Θέμα 3 " 1 Δίνεται ο μιγαδικός αριθμός z και έστω η {::} Υ = - χ 2 (I) 2

συνάρτηση f (v) =

iv Χ z , ν

ε

Ν*.

β) Η δοθείσα σχέση γράφεται:

( J8 )2 -(y + l)2 + 1 = 0 {:} 8 - (y + l)2 + 1 = 0 {::}

α) Αποδείξτε ότι:

f(I) +f(6) +f(7)+f(16)=0

β) Αποδείξτε ότι:

f(v) + f(v + 2) + f (v + 4) + f (v + 6)

=

Ο

γ) Αν ΙzΙ = 2 aποδείξτε ότι

lf(2001) +f(2004�=2ν'2.

Λύση :

α) f(1) + f(6) + f(7) + f(1 6) = = i · z + i6 · z + i7 · z + i 16 · z = = iz + i 2 z + i 3 z + z = = i z - z - iz + z = Ο β) f(ν) + f (ν + 2) + f(ν + 4) + f(ν + 6) = ί ν · z + i v + 2 · z + i v +4 · z + i v +6 · Ζ = ivz - ivz + ivz - ivz = O γ) j f(2001) + f(2004)j = J i 2001z +i 2004zJ = l iz + zl = j z(1 +i)j = lzl fi = 2fi Θέμα 4 "

Δίνεται ο μιγαδικός αριθμός z = χ + yi, x,y Ε R α) Αποδείξτε ότι αν lzl2 - (Im (z) + t)2 + 1 = 0 ,

{:} (y + l) 2 = 9 {:} y + l = ±3 {:} {::} y = 2 ή y = -4 (απορρίπτεται) βρίσκουμε z 1 = 2 + 2i, z 2 = -2 + 2i γ) ρ2 -(y+l ) 2 + 1 =0 {:}ρ2 -1 -2y-1+1 =0 {:} y2 + 2y -ρ2 = 0 Δ = 4 + 4ρ 2 > Ο και Υ ι · Υ2 = - ρ 2 < Ο άρα το τριώνυμο έχει δυο ρίζες ετερόσημες από τις οποίες δεχόμαστε λόγω της (I) μόνο τη θετική, η οποία μας δίνει δυο τιμές για το χ. Θέμα 5"

α) Να περιγράψετε γεωμετρικά το σύν,ολο (Σ) των εικόνων των μιγαδικών αριθμών z που ικανοποιούν τις σχέσεις: Ιz l = 2 και Re(z) > O . β) Αν οι εικόνες του z ανήκουν στο σύνολο (Σ) να βρεθεί ο γεωμετρικό τόπος των εικόνων του μιγαδικού w = z - 4 + 3ί • γ) Να βρεθεί ο μιγαδικός αριθμός w με το ελάχιστο μέτρο.

τότε οι εικόνες του z στο μιγαδικό επίπεδο βρίσκονται στην παρα β ολή y = .!. χ2 •

Λύση :

2

β) Από τους μιγαδικούς αριθμούς z του (α) ερωτήματος να βρεθούν αυτοί που έχουν μέτρο

JS . γ) Αποδείξτε ότι για κάθε θετικό πραγματικό αριθμό ρ υπάρχουν πάντα δυο μιγαδικοί αριθμοί z, που ικανοποιούν το (α) ερώτημα τέτοιοι ώστε να ισχύει Ιz l = ρ.

α) Οι σχέσεις lzl = 2 και Re(z) 2:: Ο ορίζουν το ημικύκλιο με διάμετρο το τμήμα του άξονα y ' y με άκρα τα σημεία Β(Ο,2) και Β ' (Ο, -2) που διέρχεται από το σημείο Α (2, Ο) του άξονα χ'χ. β) Έστω w = α + βi και z = x + yi τότε w = z - 4 + 3i {::} z = w + 4 - 3i {::} {::} χ + yi = (α + 4) + (β -3) i ισχύει όμως Ο � χ � 2 και -2 � y � 2 άρα : Ο � α + 4 � 2 {::} -4 � α � -2 και

ΕΥΚΛΕΙΔΗΣ Β' τ. Ι/64


Μαθηματικά για την Γ Λυκείου

Λ ύση :

α) Έστω z = χ + yi και w = α + βi τότε Α(χ, y) και Β(α, β) οπότε ΟΑ · ΟΒ = (χ,y) · (α,β) = αχ + βy ( Ι ) z . w = (χ + yi) (α - βi) = αχ - βχi + αyi + βy = Απ = (αχ + βy) + (αy - βχ)i (11) ό (I) και (II) προκύπτει R e ( z · w) ΟΑ · ΟΒ . β) z - Ι = (χ - Ι) + yi άρα η εικόνα του είναι το σημείο Γ( χ - l, y) z - i = χ + (y - I)i άρα η εικόνα του είναι το σημείο Δ(χ, y - 1 ) . Δηλαδή οι εικόνες του W κινούνται σε κύκλο με λ = y - I - y = - Ι επομένως η ΓΔ σχηματίζει rΔ χ - χ + Ι κέντρο Κ (-4, 3) και ακτίνα ρ = 2. Επειδή όμως -4 :::; α :::; -2 και Ι :::; β :::; 5 ο γεωμ. με τον άξονα χ' χ γωνία 1 35°. τόπος των εικόνων του w είναι το δεξί ημικύκλιο γ) A (x, y) , Γ(χ - I ,y) , Δ(χ, y - Ι) παρατηρούμε του παραπάνω κύκλου με διάμετρο στην ευθεία ότι ΑΓ // χ ' χ και ΑΔ // y ' y άρα ΑΓ ..l ΑΔ χ = - 4. γ) Η ΚΟ έχει εξίσωση y = λχ και επειδή επομένως το τρίγωνο ΑΓΔ είναι ορθογώνιο στο Α. διέρχεται από το Κ (-4, 3 ) επαληθεύεται από Θέ μα 7° αυτό, δηλαδή 3 = λ( -4) {:} λ = - �4 . Θεωρούμε τους μιγαδικούς αριθμούς z για τους � y = - 4 χ, - 4 :::; χ :::; -2 οποίους ισχύει: Ζ 6 ( z + 1 )6 = 1 . Λύνοντας το σύστημα (χ + 4) 2 + (y - 3) 2 = 4 α) Αποδείξτε ότι: =

ο

j

=

ί) z =

προκύπτει το ζητούμενο. Θ έ μα 6"

Δίνονται οι μιγαδικοί αριθμοί z και w με εικόνες τα σημεία Α και Β αντιστοίχως στο μιγαδικό επίπεδο. α) Αποδείξτε ότι Re( z · w) = Ο Α · ΟΒ .

(Ο η αρχή των αξόνων) . β) Ν α βρεθεί η γωνία που σχηματίζει με τον άξονα χ ' χ η ευθεία που ορίζεται από τις εικόνες των μιγαδικών αριθμών z-1, z-i. γ) Αποδείξτε ότι οι εικόνες των μιγαδικών αριθμών z, z-1, z-i στο μιγαδικό επίπεδο, σχηματίζουν ορθογώνιο τρίγωνο . .

! z

και ίί) Z2 + z + 1 = 0 . (I)

β) Ένα τρίγωνο έχει κορυφές τις εικόνες των ριζών της εξίσωσης (Ι) και την εικόνα του μιγαδικού αριθμού Ζ3 = 1. Να βρεθεί το είδος του τριγώνου ως προς τις πλευρές του. Λ ύ ση :

α)

i) z6 = Ι άρα lzl 6 = Ι {:} l z l 2 = I {:} {:} Ζ · Ζ = Ι {:} Ζ = =.Ι z ii) (z + l)6 = I άρα l z + II 6 = I {:} I z + l i 2 = I {:} {:} ( z + Ι) (� + Ι ) = Ι {:}

ΕΥΚΛΕΙΔΗΣ Β ' τ. Ι/65


Μαθηματ ικά για την Γ Λυκείου -

-

-

{::} z · z + z + z + 1 = 1 {::} 1 + z + z = 0 {:} 2 {::} 1 + z +-1 = Ο {::} z + z + 1 = Ο {::} z2 + z + 1 = Ο. z z β) Οι ρίζες της εξίσωσης z 2 + z + 1 = Ο είναι i +i οι z 2 2 ' z 2 = 2 2 ' άρα κορυφές του τριγώνου θα είναι τα σημεία Α(1,0 ) , 2' 2 ' 2' 2 ' βρίσκουμε ότι: (ΑΒ) = (ΑΓ) = (ΒΓ) = J3 άρα το τρίγωνο είναι ισόπλευρο. I

=

_

Θέμα

_!_ vf3 _!_ - vf3 Β [-_!_ vf3 ] r[- ..!.. -vl3] _

8"

-

δ) lzι Z 2 j lzι I .fi και επειδή το μέτρο της 2 διαφοράς δύο μιγαδικών αριθμών είναι η ' ' τους θ α ειναι ' (ΑΒ) = .fi αποσταση των εικονων =

=

2.

Είναι (ΟΑ) = J2 και (ΟΒ) = 1 . 2 Με το Πυθαγόρειο Θεώρημα αποδεικνύουμε ότι το τρίγωνο είναι ορθογώνιο στο Α και επειδή (ΟΑ) = ( ΑΒ) είναι ισοσκελές.

z2 για Θέμα 1 " Θέματα για λύση α) Να περιγράψετε γεωμετρικά το σύνολο (Σ) των εικόνων των μιγαδικών αριθμών z που ικανοποιούν τις σχέσεις: Ι z l = 3 και Im ( z ) 2: Ο . η εικόνα του μιγαδικού πραγματικός β) Να αποδείξτε ότι αριθμού z ανήκει στο σύνολο (Σ) τότε η εικόνα του μιγαδικού αριθμού ω= z + � κινείται σε

Θεωρούμε τους μιγαδικούς αριθμούς z 1 , τους οποίους ισχύει: I Ζ2 I = και I Ζ ι - Ζ2 I = I Ζ ι I .

ι

α) Αποδείξτε ότι

Re(z1

Ζ2

) = .!.

2

β) Να προσδιοριστεί ο θετικός αριθμός λ, για τον οποίο ισχύει: Ζ 2 = Ζ1

( 1 - 2λί) .

γ) Για λ = .!. aποδείξτε ότι

2

αν

Μ )

. l zι l = J2 2

δ) Αν Α η εικόνα του z1 και Β η εικόνα του z2 στο μιγαδικό επίπεδο, aποδείξτε ότι το τρίγωνο ΟΑΒ είναι ορθογώνιο στο Α και ισοσκελές . ( Ο η αρχή των αξόνων).

ευθύγραμμο τμήμα το οποίο βρίσκεται στον άξονα n'.

Θέμα 2" Δίνονται οι μιγαδικοί αριθμοί z = α + βί όπου α,β ε:R και ω 2z + i z 3 , όπου z ο συζυγής του z. Λ ύ ση : α) Αποδείξτε ότι Re( ω) = 2α + β 3 και α) l z ι - Zz l = l z ι l {::? l z ι - Z z l 2 = l z ι l 2 {::? Ιm(ω) = α + 2β . {::? (z1 - z ) (� - Ζ ) = Ζ1 � {::? 2 2 β) Να αποδείξετε ότι αν οι εικόνες του ω στο {::? z 1 z 1 - z 1 z - Ζ Ζ1 + z z = z 1 z 1 {::? z 1 z + z 1 z = 1 2 2 2 2 2 2 μιγαδικό επίπεδο κινούνται στην ευθεία με {::? 2Re ( z1 z ) = 1 {::? Re ( Ζ1 z ) = � εξίσωση y = 3χ + 2, τότε οι εικόνες του z 2 2 κινούνται στην ευθεία με εξίσωση y = - 5χ + 7. β) z 2 = Ζ1 ( 1 - 2λί ) {::? Ζ 2 = Ζ1 - 2z 1 λί {::? γ) Να βρείτε ποιος από τους μιγαδικούς αριθμούς {::? Ζ1 - z = 2 z λί 2 z οι εικόνες των οποίων κινούνται στην ευθεία άρα l z 1 - z 2 1 = l 2z 1 λi l {::? l z ι l = 2 l z ι l l λi l {::? l λl = 2 y = - 5χ + 7 έχει ελάχιστο μέτρο. άρα λ = � , ( λ > Ο ) . Θέμα 3° γ) Ζ 2 = z 1 ( 1 - 2λί) για λ = _!_ έχουμε α) Να περιγράψετε γεωμετρικά το σύνολο των 2 -

=

1

_!_

ΕΥΚΛΕΙΔΗΣ Β' τ. Ι /66

-

-


Μαθηματικά για τη ν Γ Λυκείου

εικόνων των μιγαδικών αριθμών z που ικανοποιούν τη σχέση Ι z - 3 - i Ι � 2. β) Αποδείξτε ότι : z - (3 + i) - (3 - i) = z - 6 . γ) Αποδείξτε ότι : l z - 6 1 ::; 2 + JlO δ) Αν Ζ1 , Ζ2 είναι δυο μιγαδικοί που ικανοποιούν τη συνθήκη του ερωτήματος (α) να αποδείξετε ότι l z ι - z 2 1 ::; 4 .

β) Να λύσετε στο σύνολο των μιγαδικών αριθμών, ' την εξισωση : 2 z = .fj 3 z +Ι γ) Αν z1, z2 είναι οι ρίζες της εξίσωσης του ερωτήματος (β) να υπολογίσετε την τιμή της 2 , παραστασης : Κ= (z ι · z 2 ) -200i 6 4 + (z 1 + z 2 ) ι •

Θέμα 7" Δίνονται οι μιγαδικοί αριθμοί z ι , z2 , Ζ3 με I z ιl = Θέμα 4 " Για τους μιγαδικούς αριθμούς z και w ισχύουν: I Ζ2 I = I Ζ3 I = 3 . z� + z + z = 3 , w = λ - ( λ + Ι ) i , λ ε R και α) Αποδείξτε ότι z 1 = -9 . Ζι Re (z) � Ο . α) Αποδείξτε ότι οι εικόνες του z στο μιγαδικό β) Αποδείξτε την ισοδυναμία z Ε R {:::} z = z και στη συνέχεια δείξτε ότι ο αριθμός � + � είναι επίπεδο βρίσκονται σε κύκλο (C) με κέντρο z, Ζ2 Κ(-Ι , Ο) και ακτίνα ρ = 2. πραγματικός. β) Αποδείξτε ότι οι εικόνες του w στο μιγαδικό 1 Αποδείξτε ότι Ι z, + z, + z, Ι = 3 Ιz,z, + z,z, + z,z, Ι . γ) = + + επίπεδο βρίσκονται στην ευθεία (ε): χ y Ι Ο. γ) Αποδείξτε ότι η ευθεία (ε) τέμνει τον κύκλο δ) Αν I z, Ι = Ι z, Ι = Ι z, - z, Ι αποδείξτε ότι (C) σε δυο αντιδιαμετρικά σημεία. Re (z, z, ) = � . δ) Να βρεθεί ο μιγαδικός z που έχει το μέγιστο μέτρο. ε) Να βρεθεί η εικόνα Μ του μιγαδικού w που έχει Θέμα 8" Θεωρούμε τους μιγαδικούς αριθμούς z f:. Ο, για το ελάχιστο μέτρο. τους οποίους ισχύει: l z l = I I - z l = (I) . Θέμα 5" + α) Αποδείξτε ότι z=Ι. z Δίνεται ο μιγαδικός αριθμός #Ο και η συνάρτηση β) Αποδείξτε ότι οι μιγαδικοί αριθμοί που f ( ν ) = ( i v - I) · z , ν Ε Ν * . ικανοποιούν τις ισότητες (Ι) είναι οι α) Να δείξετε ότι για κάθε ν Ε Ν * ισχύει: . .fj . .fj z = -Ι - ι. z 1 = -2Ι + ιf (ν) · f (ν + Ι ) · f ( ν + 2) · f (ν + 3) = Ο . 2 ' 2 2 2 β) Αν ισχύει f( 5 ) = -3 + ί , δείξτε ότι z = 2 + ί. γ) Αν Α, Β οι εικόνες των z1 , z2 αντιστοίχως γ) Αν z = 2 + ί. Αποδείξτε ότι στο μιγαδικό επίπεδο, να υπολογίσετε τη κυρτή -ιr ( ν + 3 ) - f ( ν + Ι )I = 2J5 ' για κάθε ν Ε Ν* . γωνία ΑΟΒ , ( Ο η αρχή των αξόνων). δ) Αποδείξτε ότι οι αριθμός ω = ( Ζ ι + z 2 ) είναι z� + z� Θέμα 6" 'Ε z πραγματικός αριθμός. στω ο μιγαδ ικος' αριθ μος z και W =

ι.;ι

v

'

z

2

+

l

α) Να αποδείξετε ότι αν ο w είναι πραγματικός Σχόλιο : Σχετικά με το 3° θέμα των πανελλαδικών εξετάσεων 2006. αριθμός, τότε ο z είναι πραγματικός ή Ι z l = Ι . ΕΥΚΛΕΙΔΗΣ Β ' τ.Ι /67


Μαθηματικά για την Γ Λυκείου

Θέμα 3" ( αίί ) Από την υπόθεση οι εικόνες των Ζ1 , Ζ 2 , z 3 βρίσκονται στον κύκλο με κέντρο Ο και ακτίνα ρ=1 . Από το αί οι αποστάσεις των εικόνων τους είναι ίσες άρα σχηματίζουν ισόπλευρο τρίγωνο. Δηλ .το · ι z ι - z 2 j είναι πλευρά ισόπλευρου τριγώνου εγγεγραμμένου σε κύκλο, άρα l z ι - z 2 1 = R J3 '* l z ι - z 2 1 = J3 '* l z ι - z 2 1 2 = 3 . (γιατί δίνεται το παραπλανητικό ερώτημα l z ι - z 2 1 2 ::; 4 ;) Ομοίως ενώ το Re ( z 1 z 2 ) = - 21 γιατί δίνεται Re ( z 1 z ) � - 1 ; 2 Από τα παραπάνω προκύπτει ότι : Το 3 ° θέμα από πλευράς αξιολόγησης είναι εσφαλμένο διότι: 1 ) Προτάσσει το δύσκολο ερώτημα, το οποίο μάλιστα αν δεν λυθεί γεωμετρικά απαιτεί συγκεκριμένη αντικατάσταση. 2") Το ερώτημα ( αίί ) χωρίς να έχει σφάλμα υπό στενή μαθηματική έννοια, είναι παραπλανητικό για τους μαθητές. 3") Στο τελευταίο ερώτημα δημιουργεί σύγχυση ο όρος γεωμ, τόπος, γιατί από την εκφώνηση η σχέση j z1 j = j z 2 j = j z 3 j = Ι συνδέεται με το και "

Άρα υπάρχει λάθος στις ενδεικτικές λύσεις της ΚΕΕΕΛ. Πιστεύουμε ότι το θέμα θα έπρεπε να διατυπωθεί ως εξής: Δίνονται οι μιγαδικοί αριθμοί z 1 ,zυ z3 για τους οποίους ισχύουν: j z1 j = j z 2 j = j z 3 j = 1 και z 1 + z 2 + z 3 = 0 . Αποδείξτε ότι: α) l z ι + z 2 j 2 + l z ι - z 2 l 2 = 2 l z ι l 2 + 2 j z 2 1 2 β) j z ι - z 2 l = l z ι - z3 1 = Ι z 2 - z3 1 γ) I Ζ ι - Ζ 2 1 2 = 3 και Re ( Ζ ι z 2 ) = - 21 δ) Οι εικόνες των μιγαδικών αριθμών z 1 , Ζ υ z 3 στο μιγαδικό επίπεδο σχηματίζουν ισόπλευρο τρίγωνο και βρίσκονται σε κύκλο του οποίου να βρείτε την εξίσωση. (το πρώτο ερώτημα είναι η άσκηση 9 του σχολικού βιβλίου σελ 1 Ο 1 και βοηθά στην επίλυση του Β. Δεν καταλαβαίνουμε γιατί η επιτροπή παίρνει με μεγάλη ευκολία μια άσκηση από εξωσχολικά βιβλία (και μάλιστα με τα ίδια νούμερα) και δεν αξιοποιεί το σχολικό βιβλίο. Από πλευράς αξιολόγησης είναι απαράδεκτο.) -

·

Μαθη ματικά Γενικής Παιδείας Ανά λυση

1.

Να βρείτε το συναρτήσεων:

σύνολο

α) r ( x ) = � +

J

ορισμού

των

Τ ου Γ ιώ ργου Κοτσιφ άκη

Λύση :

α) Η συνάρτηση f ορίζεται για τους αριθμούς χ ε IR για τους οποίους ισχύουν: χ�1 χ 3 -1 � ο χ3 � 1 x +l :;t: O <::::> x + 1 :;t: O <::::> x :;t: -1 2 - χ � 0 (2 -χ)(χ +1) � 0 (x -2)(x +1) :s; O χ +1 .

Z-x x+t

{

ΕΥΚΛΕΙΔΗΣ Β' τ. Ι/68

{


Μαθηματικά για τη ν Γ Λυκείου

{

χ ;::: 1 <::::> 1 ::; χ ::; 2 . <::::> -1 ::; χ ::; 2 Άρα το σύνολο ορισμού της f είναι: A=[l ,2].

3. Το βάρος της φέτας (τυρί) σε κιλά που

πουλάει ένα νέο σούπερ μάρκετ συναρτήσει των ημερών λειτουργίας του t είναι: 1

(

(

r t ) = 3o 2t - ι + t + 2

-

)

β) Η συνάρτηση g ορίζεται για τους αριθμούς Να βρείτε πόσα κιλά φέτα πούλησε: Χ Ε JR , για τους οποίους ισχύουν: α) Τις 8 πρώτες μέρες 2 2 x l ;::: 1 χ - ι ;::: ο χ ;::: ι 1 ) τη 2 η και την 3 η μέρα μαζί β <=> 6+ χ - χ2 > 0 <=> 2 <=> χ - χ - 6 < 0 -2 < χ < 3 γ) την 4η μέρα. χ ;::: 1 ή χ ::; -ι <::::> Λύση -2 < χ < 3 α) Η φέτα που πούλησε τις 8 πρώτες μέρες είναι: -2 -ι ι 2 3 + οο ι f { 8 ) =30 2 · 8-1 + -- = 3ο ι5 + _!_ = 453 κιλά : � J� 1== =: �1: -J-� ___?_�� 8+2 10 Συμπεραίνουμε ότι το σύνολο ορισμού της g είναι: Β=(-2,-ι] U [ι ,3). β) Η φέτα που πούλησε τη 2η και την 3η μέρα μαζί είναι: 2. Έστω η συνάρτηση : f(x)=ιn(2x-e) . f { 3 ) -f ( I ) 30 6-ι + * -30 2-ι + ι ι 6 κιλά α) Ο αριθμός 1 είναι τιμή της f; β) Η γραφική παράσταση της f τέμνει τον γ) Η φέτα που πούλησε την 4η μέρα είναι: άξονα χ ' χ; γ) Η γραφική παράσταση της f δεν f { 4) -f ( 3 ) = 30 8-ι + -30 6-ι + * 59 κιλά

{

- 00

{

{

{

) ( )

(

=

διέρχεται από το σημείο M(2e, ι n3);

( ) ( i) ( i) ( )

=

4 . Έστω η συνάρτηση :

Λ ύσ η .

=

2 Η συνάρτηση f ορίζεται για τους αριθμούς χ - 5χ + 4 x r( ) = χ Ε R για τους οποίους ισχύει: 2χ - e > Ο <::::> χ > � Fx - 2 2 α) Να βρείτε το πεδίο ορισμού της f. Άρα το σύνολο ορισμού της f είναι: f { χ) β) Να βρείτε το όριο: L = lim Χ --+ 4 Α= � , + οο . α) Ο αριθμός 1 είναι τιμή της f αν, και μόνο αν, Λύ σηα): Η συνάρτηση f ορίζεται για τους αριθμούς υπάρχει χΕΑ με: xER, για τους οποίους ισχύουν: f(x)=1 <::::> ιη(2χ - e)=ιne <::::> 2χ - e=e <::::> x=e χ ;::: ο χ ;::: ο χ ;::: ο :> <::: <=> Η τιμή αυτή του χ είναι δεκτή γιατί e Ε Α . χ :�; 4 Γχ -2 :�; 0 Γχ :�; 2 Άρα το 1 είναι τιμή της f. Άρα, το σύνολο ορισμού της f είναι: Α= [Ο,4) υ (4,+=) β) Η γραφική παράσταση της f τέμνει τον β) Έχουμε: άξονα χ ' χ αν, και μόνο αν, υπάρχει χΕΑ με: 2 -5χ +4 . ( χ 2 -5χ +4) ( Γχ + 2) χ e . + ι l ι m ---:---=-.,.-=------L lιm f(x)=O <::::> Ιη(2χ--e)=Ιηι <::::> 2x--e=l <::::> χ= fx -2 2 ( Γχ -2) ( Γχ...,- +2)--'--,Η τιμή αυτή του χ είναι δεκτή, γιατί ανήκει ( χ - t )( χ -4) { fx +2) στο Α. Άρα, η γραφική παράσταση της f τέμνει τον lim = lim [( x-I ) { fx +2) = ] e +-,1 ο χ -4 α' ξονα χ ' χ στο σημειο' 2 = 3 · 4 = 12 γ) Έχουμε: f(2e) = ln(4e - e) = ln(3e) = ln3 +lne = ι + ln3 * ln3 5. Να βρείτε τα παρακάτω όρια: Άρα η γραφική παράσταση της f δεν διέρχεται · χ 2 - 7χ + 6 · J4+h - 2 L L2 = ι = ι από το σημείο Μ. xl� Χ 2 hl .!!: Χ h -9 +8 .

( )

{

=

--

( )

x --+4

χ ->4

I

ΕΥΚΛΕΙΔΗΣ Β ' τ. Ι /69

{

{

χ--+4

χ ->4


Μαθηματικά για την Γ Λυκείου

χL 3 = lim3 �3 >--+ ν χ + 6 - χ χ-3 Όπου f{x) = ­ x+l

. r (t + h) - r(ι) , L4 = lιm h h --+ 0

Λύ ση .

Λύση

Έχουμε: . χ 2 - 7χ + 6 = lιm . ( χ - 1 ){ χ - 6 ) = ι = lιm 2 Χ -9Χ + 8 ( Χ - 1 ) ( Χ - 8) 1 - 6 = -5 . . χ-6 =lιm-χ -8 1 - 8 7 •

I

χ-+ Ι

χ-+ Ι

χ-+ Ι

�4 + h - 2 = ι 2 = hιm --->0 h . ( �4 + h - 2 ){ �4 + h + 2 ) = = lιm h --+O h { �4 + h + 2 ) 4 + h - 4 = lim 1 = = hlim --+O h { �4 + h + 2 ) h --->0 �4 + h + 2 4 ι.

χ -3 ι 3 = lim Χ-+3 �χ + 6 - Χ = . ( χ - 3 ) { �χ + 6 + χ ) = = lιm χ-- +3 ( �χ + 6 - χ ) ( �χ + 6 + χ )

Χ -+

r ( χ) - 2 =5 Χ-2

α) Να β ρείτε τον αριθμό f(2). β) Να β ρείτε το όριο:

(1)

β) Επειδή f(2)=2, έχουμε: ( χ - 4) ( χ + 2) . χ2 - 2χ - 8 . �-�-.:...,= lιm 2 4 4 χ-+ χ ( χ - 4 ) ( χ + 4) ..... χ ( χ - 1 6 )

L = 1ιm

χ+2 = �. =� 32 1 6 χ-+ 4 χ ( χ - 4)

= lim

f { x) = � χ+2

6. Μια συνάρτηση f είναι συνεχής στο χσ=2 και lim2

και επειδή χ-+ lim2 2 = 2 , έχουμε: lim Χ ---> 2 [ f ( x ) - 2 + 2] = 0 + 2 :::::> limf Χ-+2 ( x ) = 2 :::::> f ( 2) = 2

7. Έστω η συνάρτηση:

+6 + χ ) . ( χ -3 ) { "'χ + 6 + χ ) = lιm---'--. ( χ -3 ) ( "'χ= Ιιm χ -+3 - ( χ - 3 )( χ +---:-2 )--'....3. χ +6-χ2 + χ = -� Γχ+6 = lim χ -+3 - ( χ + 2) 5 l + h -3 +1 f 1) 1 + h) f ( ( . . 1 + h + 1 -ι4 = lιm ιm��--"--= 1h-+0 h-+0 h h h-2 +1 2 2h h +2 = lim h --->0 h +-2 = 1 h-+O h ( h + 2) = limh-+0 h = lim ισχύει:

α) Επειδή η f είναι συνεχής στο χο=2, έχουμε: lim χ-->2 f ( x) = f ( 2 ) . Από την ( 1) και επειδή lim χ--> 2 ( χ - 2) = Ο , έχουμε: lim2 f (Χχ ) - 2 (χ - 2) = 5 · Ο => lim[f Χ-+ Χ-+2 ( χ ) - 2] = Ο -2

α) Με τον ορισμό της παραγώγου, να β ρείτε την f'(2). β) Να β ρείτε την εξίσωση της εφαπτομένης ε της γραφικής παράστασης της f στο σημείο της Α(2, f(2)). Λ ύση .

α) Έχουμε: 2(2 + h) 2 · 2 h) 2 + ( (2) f f = lim 2 + h + 2 - 2+2 = f(2) = 1im h -+0 h -+0 h h 4 + 2h -1 . h . ι ι = lιm = lim 4 + h 1ιm-- = - . = h->0 h h-+O h ( 4 + h) h-+0 4 + h 4

β) Η εφαπτομένη ε της γραφικής παράστασης της f στο σημείο Α(2, f(2)) έχει συντελεστή διεύθυνσης τον αριθμό f'(2)= ..!_4 . Έτσι, η εξίσωση της ε είναι της μορφής: y = � χ + β , όπου β ε JR . Και επειδή η ε διέρχεται από το σημείο Α(2, f(2))=A(2, 1 ), έχουμε:

ΕΥΚΛΕΙΔΗΣ Β' τ.l/70


Μαθηματικά για την Γ Λυκείου

Ι + β � β = Ι - -Ι � β = -Ι . Ι = -2 2 4 2 Άρα η εξίσωση της εφαπτομένης ε είναι: y = -4Ι x + -Ι . 2 8. Να βρείτε τις παραγώγους των παρακάτω

συναρτήσεων: 1 . f(x)=xlnx

2.

3. f(χ)=ημ3χ+eημχ 4 . 5 . f(x)=(x3-x2+1)5 6. f { x ) = �1 + .,Γχ , χ > Ο

χ-1 f { χ ) = -_ 2 χ +_ χ_ +_1 2 f(χ)=συν 3χ

Λύση.

Έχουμε: Ι . f' (x)=(x)Ίnx + x(lnx) ' = lnx+x ·_!_χ = lnx+I

f ( x ) = _!_4 ( e x + e-x ) ' = _!_4 ( e x - e -x ) .

f " ( χ)

f " (3 ) = _!_4 ( e x + e -x ) ' = _!_4 ( e x - e -x ) = f ( x) .

=

± ( e x - e-x ) ' = ±( ex + e-x )

=

f( Χ ) .

Έτσι έχουμε: (Ι) �7 · f(x) - f(x) = 7 · f(x) - f(x) + 4e � 8 · f(x)-8f(x)=4e�2(ex+e-x) -2(ex+e-x)=4e � (ex+e-x) - (ex+e-x) = 2e � e-x = e � -χ = Ι � χ = -Ι 10. Έστω η συνάρτηση : f(x) = 3συν2χ+4ημ2χ. Να δείξετε ότι η εξίσωση : f"(x) + 4 f(x) + συν2χ = J2 , είναι αδύνατη.

(χ - Ι ) ' ( χ 2 + χ + Ι ) - (χ - Ι ) ( χ 2 + χ + Ι ) ' Λύση : 2. f, (χ ) = 2 Με χ Ε JR , έχουμε: 2 χ + χ + Ι ( ) f (x) = 3(-ημ2χ) - (2χ) '+ 4(συν2χ) · (2χ) ' = x 2 + x + l - (x - I)(2x + I ) -χ 2 + 2χ + 2 . = -6ημ2χ + 8συν2χ z := ( x z + x + I ) ( xz + χ + Ι γ f" (x)= -6(συν2χ) · (2χ) ' + 8(-ημ2χ) · (2χ) ' = 3. f' (x) = (ημ3χ) '+(eημχ) ' = = -Ι2 συν2χ - Ι6ημ2χ = (συν3χ)(3χ) '+ eημ\ημχ) ' =3συν3χ+ eημχσυνχ. Έτσι, η δοσμένη εξίσωση είναι ισοδύναμη με την εξίσωση: 4. f' (χ)=2(συν3χ) · (συν3χ) '=2(συν3χ) · (-ημ3χ) · -Ι2συν2χ-16ημ2χ + 4(3συν2χ + 4ημ2χ) + συν2χ= .J2 (3χ) '= -6 · συν3χημ3χ = -3ημ6χ � συν2χ = .J2 , αδύνατη, γιατί .J2 > Ι . 2 2 3 3 ' + + ' 5. f (x)=5(x -x I)4 (χ -χ Ι) = 1 1 . Μια συνάρτηση f είναι ορισμένη και = 5(χ3-xz+ Ι )4 . ( 3xz-2χ)=5χ(χ3-xz+ Ι )4( 3χ-2) παραγωγίσιμη στο R και για κάθε χ Ε JR ισχύει: f(x2 - 2χ + 3) = f(2x - 1) (1) Ι Ι Ι · -- = { I + .J�} = 6. f(x ) = Να δείξετε ότι η εφαπτομένη ε της γραφικής 2�Ι + -Γχ 2-Γχ 2�Ι + -Γχ παράστασης της f στο σημείο της με τετμημένη 1 είναι κάθετη στον άξονα y 'y. = ----;=Ι ==:= . 4�χ + χ-Γχ Λύση . Από την (Ι), παραγωγίζοντας, έχουμε για κάθε 9 . Να λυθεί η εξίσωση : χ E lR : f(x2 - 2χ + 3)· (χ2 - 2χ + 3)' = 7 · Γ(χ) - f'(x) = 7 · f' "(x) - f(x)+ 4 e (1) όπου f(2x - Ι) · (2χ - Ι) ' => (2χ - 2)f(x2 - 2χ + 3)= 2f(2x-I) (2) Από τη (2) με χ = Ι βρίσκουμε 0=2 f (Ι), οπότε f(l)=O. Συνεπώς η εφαπτομένη ε είναι είναι . \ ίι σ η . παράλληλη στον άξονα χ ' χ δηλαδή κάθετη στον Μ ε χ Ε � έχουμε: άξονα y ' y. •

,

ΕΥΚΛΕΙΔΗΣ Β ' τ. Ι /7 1


Μαθηματ ικά για την Γ Λυκείου

χ = 2, ίσο με f(2) = --e2

12. Θεωρούμε τις συναρτήσεις: f(x) = (α - 2)Ιηχ και g(x) = χ2 + βχ - 3 Να βρείτε τους αριθμούς α,β ε IR , για τους οποίους οι γραφικές παραστάσεις των συναρτήσεων f και g να έχουν κοινή εφαπτομένη στο σημείο με τετμημένη χο=1. Λ ίJ ση .

Οι συναρτήσεις f και g είναι ορισμένες και παραγωγίσιμες στο (0,-too) με: f' ( χ ) = ( α - 2 ) _!_ και g ' ( χ ) = 2χ + β χ Για να συμβαίνει το ζητούμενο πρέπει και αρκεί (γιατί; ): f (I) = g (I) Ο=β-2 � β=2 � f' ( I ) = g ' ( I ) α - 2 = 2 + β α = 6 Άρα οι ζητούμενες τιμές των α και β είναι: α = 6 και β = 2

{

{

{

13. Η γραφική παράσταση της συνάρτησης: f(χ) = (αχ + β)·eΧ, όπου α,β ε IR , διέρχεται από το σημείο Α(1 , -2e) και η εξίσωση της εφαπτομένης της στο σημείο αυτό είναι: y = - ex - e. α) Να βρείτε τους αριθμούς α και β β) Να βρείτε τα διαστήματα μονοτονίας και τα ακρότατα της f. Λύση

α) Έστω ότι για δυο αριθμούς α,βε JR. οι υποθέσεις ισχύουν. Η συνάρτηση f είναι ορισμένη και παραγωγίσιμη στο JR. με: f' (x) = αeχ + (αχ + β)eχ = (αχ + α + β)eχ Από τις υποθέσεις έχουμε: f(I ) = -2e (α+β) e = -2e α+β = -2 α = Ι => => => f ( I) = --e ( 2α+β)e = --e 2α+β = -Ι β = -

{

{

{

{

1 4 . Έστω η συνάρτηση : f(x) = χ2 + 2χ - λlnx, χ>Ο (λ ε IR ) . Η εφαπτομένη ε της γραφικής παράστασης της f στο σημείο της Α(1, f(1)) είναι παράλληλη στον άξονα χ'χ. α) Να δείξετε ότι λ=4 β) Να βρείτε τα διαστήματα μονοτονίας και τα ακρότατα της f. γ) Να δείξετε ότι για κάθε χ>Ο, ισχύει: χ2 + 2χ � 3 + 4 Ιηχ Λύση

α) Η συνάρτηση f είναι ορισμένη και παραγωγίσιμη στο Α=(Ο, +οο) με: f' ( χ) = 2χ + 2 - λ -1 (1) χ Ο συντελεστής διεύθυνσης της εφαπτομένης ε είναι: f' (l) = 2 + 2 - λ = 4 - λ. Έτσι, έχουμε: ε // χ ' χ � f' (l) = Ο � 4 - λ = Ο � λ = 4. β) Επειδή λ = 4, από την (1) έχουμε στο Α = (0, +οο): 4 χ2 + χ - 2 = Γ( χ ) = 2χ + 2 - - = 2 χ χ > Ο, αν χ > 1 = 2 ( χ - Ι χ + 2 ) = Ο, αν χ = Ι < Ο, αν Ο < χ < Ι

Α ν τ ισ τρόφω ς.

)�

{

Όπως βρίσκουμε εύκολα, με α = Ι και β = -3 οι υποθέσεις ισχύουν. Άρα, οι ζητούμενοι αριθμοί είναι: α = 1 και β = -3 β) Επειδή α = Ι και β = -3, έχουμε: f(x) = (x-3)ex και > Ο, αν χ > 2 f' ( x ) = ( x - 2 ) e x = 0, αν χ = 2 γ) Από το προηγούμενο ερώτη μα < 0, αν χ < 2 συμπεραίνουμε ότι για κάθε χ > Ο, ισχύει: Συμπεραίνουμε όtι η f είναι γνησίως φθίνουσα f(x) � f(l) ::::> χ2 + 2χ - 4Inx � 3 στο διάστημα ( 2], γνησίως αύξουσα στο [2, (αφού λ = 4) ::::> χ2 + 2χ � 3 + 4lnx +οο) και παρουσιάζει ελάχιστο στη θέση

{

-οο ,

ΕΥΚΛ ΕΙΔΗΣ Β' τ. Ι /72


ΕΘΝ Ι ΚΗ

Η Π ΡΩΤΗ ΑΣΦΑΛΙΣΥΙΚΗ

�/ :

·

·

Όταν n ιστορία 2 . 50 0 χρόνων χωράει σ' ένα παράθυρο . . . τότε αλλάζει όλn n εργασια κn φιλοσοφία Είνα ι σnμ αντικό. όταν εργάζεσαι. ονειρεύεσα ι . σχεδιάζεις. το μ άτι να ελευθερώνεται σ· α υτn τnν εικόνα . Απ' τn θέσn του νέο υ μ α ς κτn ρίου. έχου μ ε δικαίω μ α και στον Ι ε ρ ό β ρ άχο τnς Ακρόπολnς. Θεωρούμε ότι τίποτα δεν ε ίνα ι τυχαίο και ότι οι λεπτομ έρ ειες συνθέτουν το σύνολο. Γιατί n ε ργασι ακn φιλοσοφ ία τnς Εθνικnς Ασφαλιστικnς. που δ ια μ ο ρφώνεται και από το περιβ άλλον. είνα ι α υτn που π άνω α π ό έναν α ιώνα δ n μ ιο υ ργεί τnν ά ρ ρ n κτn σχέσn με τους π ελάτες τnς. Ο Ν ΕΟΣ ΠΑΘ Μ ΟΣ ΑΠΟΓΕΙΩΣΗΣ Τ Η Σ ΕΘΝΙΚΗΣ ΑΣΦΑΛΙΠΙ ΚΗΣ ΠΕΓΑΖΕΙ ΣΕ ΕΝΑ ΧΩ ΡΟ ΟΛΕΣ τJΣ ΥΠ Η Ρ ΕΣ Ι ΕΣ ΤΗΣ. Λεωφ. Συγγρού 1 03- 1 05, 1 1 7 45 ΑθΗ ΝΑ.

Τηλ. Κέντρο: 2 1 Ο 90 99 000, www. ethniki -asta ιistiki.gr


Ο Εuκλεiδnς προτεiνε• «Η καρδιά των μαθηματικών είναι τα προβλήματα και οι λύσεις και ο κύριος λόγος ύπαρξης του μαθηματικού είναι να λύνει προβλήματω>. P. R. HALMOS

Επιμέλεια: Α. Κυριακόπουλος, Γ. Στρατής, Γ. Τριάντος, Ν. Αντωνόπουλος Λύσεις Προτεινομένων Ασκήσεων 80. Ν α αποδείξετε ότι για κάθε δ ο θέντα ακέ ρ αιο αριθμό α, η δ ιοφαντική εξίσωση ( α2 -1 )(β 2 -1) = γ2 -1 έχει του λάχιστον δυο διακεκριμένες ακέ ραιες λύσεις ω ς προς β και γ. (Επροτάθη από τον Ακαδημαϊκό κ. Ν ικόλαο Α ρτεμιάδη )

(Επροτ άθη από το συνάδελφο Μπάμπη Στεργίου - Χαλκίδα) Λύ ση (Από το συνάδελφο Σ αμπ ά Θ εόδωρο - Πάτρα) Α

Λύση (από τον ίδιο) Από τη γνωστή ταυτότητα

χ(χ+ 1 ) (χ+2) (χ+3 ) + 1 = (χ2 +3χ+ 1 ) 2 αν θέσουμε χ=α-1 , έχουμε: (α2- 1 ) (α2+2α)=(α2+α+ 1 ) 2 - 1 ( 1 ) Αν τώρα στην ( 1 ) θέσουμε β=α+ 1 και α2+α- 1 =γ, τότε α2 +2α=β2- 1 και η ( 1 ) γράφεται: (α2- 1 ) (β2- 1 )=γ2- 1 απ ' όπου προκύπτει ότι ο ι αριθμοί β=α+ 1 και γ=α2 +α- 1 αποτελούν λύση δοσμένης εξίσωσης. Αν επαναλάβουμε την ίδια διαδικασία με χ=α-2 και κατόπιν β=α- 1 βρίσκουμε και δεύτερη λύση της δοσμένης εξίσωσης, την: β=α- 1 , γ=α2-α- 1 . Σχόλιο : Είναι φανερό ότι αναζητούμε λύσεις

εξαρτώμενες από τον αριθμό α. Λύσεις έστειλαν επ ίσης ο κύριο ς Ρ . Μπόρης - Δ αφνή ς Αθηνά και ο μηχανικός Ανδρή ς Ιωάννη ς -Αθηνά . 86. Δίνεται τρ ίγωνο ΑΒ Γ και η δ ιχοτ ό μος του Β Δ . Αν Μ είναι το μέσο της πλευρ άς ΑΒ και Μ Δ l. Β Δ , να αποδείξετε ότι ΑΒ = 3Β Γ .

Ρ

Από το Μ φέρνουμε παράλληλη στη διχοτόμο ΒΔ και έστω Κ το σημείο τομής της με την ΑΔ. ΒΔ Τότε ΜΚ = (1) 2 Αν η προέκταση του ΜΔ τέμνει την προέκταση της ΒΓ στο σημείο Λ, τότε στο τρίγωνο ΒΛΜ η διχοτόμος ΒΔ της γωνίας ΛΒΜ είναι και ύψος, οπότε το τρίγωνο είναι ισοσκελές, άρα ΒΛ = ΒΜ =

ΑΒ

(2) 2 Τέλος αν φέρουμε κάθετη στη ΜΛ στο σημείο Λ και ονομάσουμε Ρ το σημείο τομής της με την προέκταση της πλευράς ΑΒ, τότε τα τρίγωνα ΜΚΔ και ΑΡ Δ είναι ίσα, οπότε

ΕΥΚΛΕΙΔΗΣ Β ' τ.Ι/74


Ο Ευκλείδης πρ οτείνει ... Ευκλείδη ... και Δ ιόφαντο

<'> ΒΔ ΡΛ = ΜΚ = 2 Από την ομοιότητα των τριγώνων ΒΓ Δ και , ΒΓ ΒΔ , ΛΓΡ , παιρνουμε: - = - = 2 οποτε ΓΑ ΡΑ ΒΓ (3) ΒΓ = 2ΓΛ δηλ. ΓΛ = τ Τελικά,

(2)

(3)

ΑΒ = 2ΒΜ = 2ΒΛ = 2(ΒΓ + ΓΛ) =

(

2 ΒΓ +

;)

Β

= 3ΒΓ

που είναι και το ζητούμενο. λύ ση (Από το συνάδελφο Κυριακόπουλο Θανάση - Αθήνα) 2η

Α

που είναι και το ζητούμενο. Επίσης έστειλαν Λύσεις

ο χημικός Καρβελάς Δημήτρης - Πεύκη Αττικής οι συνάδελφοι Καραχάλιος Βασίλης Τρίπολη, Κούρτης Χρυσόστομος - Λάρισα, Σταματογιάννης Γιάννης - Δροσιά Αττικής, Μάγκος Αθανάσιος - Κοζάνη, ο κύριος Ρ. Μπορής - Δάφνη Αθήνα και ο μαθητής Σακελλάρης Γιώργος, Μουσικό Λύκειο Βόλου. 90. Έστω μια συνάρτηση f : JR � JR η οποία είναι παραγωγίσιμη στο R, χωρίς να είναι σταθερή σε κάποιο διάστημα, με f(α)=Ο, και f(x0)>0 για κάποιο Χο >α. Αν ισχύουν: f"(x) = p(x)f(x), όπου p(x) συνάρτηση συνεχής μη σταθερή στο R ώστε p(x)>O για κάθε χ ε JR και f'(κ) f(m) = f'(m)f(κ) για κάποια κ,m με α<κ<m, να αποδείξετε ότι υπάρχει ξ ε (κ, m) με την ,

ιδιοτητα ν Ρ(ξ)

I Ιn f(m) - ln f(κ) l

> .:..__ _ _ _ _ ___,_

m-κ

(Επροτάθη από τον κ. Ροδόλφο Μπόρη Δάφνη, Αθήνα)

Β Γ Θεωρούμε σημείο Λ στην ΑΒ, ώστε ΒΛ=ΒΓ. Τότε το τρίγωνο ΒΛΓ είναι ισοσκελές και επειδή η ΒΔ είναι διχοτόμος της γωνίας Β, ισχύει: ΛΓ .l ΒΔ , οπότε ΛΓ II ΜΔ. ΑΔ ΑΜ γ I 2 γ Άρα = = = (1) ΑΓ ΑΛ γ - α 2(γ - α) Από το θεώρημα της εσωτερικής διχοτόμου στο τρίγωνο ΑΒΔ προκύπτει ότι: ΑΔ -= γ (2) ΑΓ α + γ λ;τό n; ( l ) (2) παίρνουμε:

.

,

·

� ( ·�ι -

α .

=

. ..

--

,

-

·

α - .,

::::>

α _._ γ = 2 ( γ - α ) ::::> γ = 3α

Λύση (από τον ίδιο) Αρχικά θα αποδείξουμε ότι ο αριθμός α είναι μοναδική λύση της εξίσωσης f(x)=O. Έστω ότι η εξίσωση έχει και δεύτερη ρίζα β και ας είναι β> α. Τότε επειδή η f δεν είναι σταθερή στο [α,β] , υπάρχει χ ι ε (α,β) ώστε f(x 1 ):;t:O. Χωρίς βλάβη της γενικότητας θεωρούμε f(x 1 )>0. Η f ως συνεχής στο [α,β] παρουσιάζει μέγιστο στο διάστημα αυτό. Αν γ το σημείο του (α, β) στο οποίο η f παρουσιάζει μέγιστο, τότε προφανώς f(γ)>0. Τότε: f"(γ) = p(γ)f(γ)>Ο, οπότε από το κριτήριο της δεύτερη παραγώγου η μη σταθερή στο [α, β] συνάρτηση παρουσιάζει και ελάχιστο στο γ, που αποκλείεται. Ά ρα η εξίσωση f(x)=O έχει μοναδική ρίζα τον αριθμό α και δεδομένου ότι f(x0)>0, ισχύει f(x)>O στο (α, +ω), διότι η f διατηρεί σταθερό πρόση μο στο (α, +ω). Με χ>α έχουμε:

Ε\Κ\ΕΗΗΣ Β ' τ . Ι /75


Ο Ευκλείδη ς πρ οτείνει ... Ευκλείδη ... και Δ ιόφαντο

( J ( J ( J( ) [ ] ( ) ( J ( J )

( J

2 f(x) . = f '(x) - f(x) = P (x) f(x) f(x) f(x) 2 => Ρ( χ ) = f(x) + f(x) f(x) f(x) m => [ P (x)dx = f (x) + r f (x) f(x) f(x) 2 = 0 + r f (x) dx κ

f(x) f(x) 2

2

=>

Λύση (από το χημικό Καρβελά Δημήτρη ­

Πεύκη Αττικής)

dx =

f(x)

οπότε υπάρχει ξ Ε (κ, m) ώστε να ισχύει:

(m - κ

Ρ(ξ) =

(x) 2 f' r dx f(x)

Από την ανισότητα ολοκληρώματα έχουμε:

(1)

B.C.S.

για

( �i:iJ' ( (�i:iJ' ) ) ( �i:iJ' ( ) (---J ( dx [ Ι ' dχ � r

r

m-κ

=>

r

dx ψn f(x) J: )

· ldx

'

τα

' �

2 (x) 2 dx � �--------�1 n f(m) - l n f(κ) r f' f(x)

m-κ

JP© � ! In f(m) - ln f(κ)l

m-κ Η ισότητα ισχύει όταν έχουμε «ίσον» στην ανισότητα B.C.S. δηλ. όταν υπάρχει λ Ε R, ώστε για κάθε χ>α να είναι

f (x) = λ <=> f(x) = ce λx ' f(x)

c

Ε IR '

Αν χ είναι το ψηφίο των δεκάδων και το y το ψηφίο των μονάδων του αριθμού, τότε ισχύει 1 0x+y=x3 +y2 ( 1 ) με Χ Ε { 1 ,2, . . . ,9} και y E {0, 1 , . . . ,9} Είναι: ( 1 ) <=>χ( Ι Ο-χ2 ) = y(y- 1 ) όπου το δεύτερο μέλος είναι άρτιος ως γινόμενο δυο διαδοχικών ακεραίων. Άρα και το πρώτο μέλος πρέπει να είναι άρτιος, οπότε εύκολα βρίσκουμε ότι ο αριθμός χ είναι άρτιος. Εξάλλου y(y- 1 )�0, οπότε: 1 0- χ22: Ο => χ Ε { 1 ,2,3 } και δεδομένου ότι είναι άρτιος, συνεπάγεται χ = 2. Με χ = 2 η ( 1 ) γράφεται 20+y = 8+y2<::> y=4 ή y= -3, οπότε y=4. Επομένως ο ζητούμενος αριθμός είναι ο 24. Λύσεις έστειλαν επίσης οι συνάδελφοι Κούρτης Χρυσόστομος Λάρισα, Ηλιόπουλος Γιάννης - Καλαμάτα, Μάγκος Αθανάσιος - Κοζάνη, και ο κύριος Ρ. Μπόρης Δάφνη - Αττικής.

Απ ' όπου, εξαιτίας της ( Ι ) έχουμε: 2 Ρ(ξ) � ( In f(m) - ln f(κ) ) ( m - κ)2 =>

(Επροτάθη από τους συναδέλφους Τάκη Δρούτσα και Νίκο Πανουσάκη - Αθήνα)

που

αποκλείεται διότι τότε με αντικατάσταση στην f" (x)=p (x)f(x) οδηγούμαστε στο συμπέρασμα ότι η p(x) είναι σταθερή . Επομένως η ισότητα δεν ισχύει ποτέ, πράγμα που σημαίνει ότι JP© > I In f(m) - ln f(κ)l m-κ

9 3 . Ν α βρείτε διψήφιο ακέραιο αριθμό, ο

οποίος να είναι ίσος με το άθροισμα του κύβου του ψηφίου των δεκάδων του και του τετραγώνου των ψηφίων των μονάδων του.

94. Αν για τους πραγματικούς αριθμούς

α,β,γ ισχύουν αβγ1:0 και β 2 + γ 2 α2 γ 2 + α 2 β 2 .:...____:_ .__ _ _ _ _

2βγ

+

_

2αγ

+

α2

+

β2

_

2αβ

γ2

=

1 (1)

να αποδείξετε ότι δυο από τα κλάσματα είναι ίσα με + 1 και το άλλο ίσο με -1 . (Επροτάθη από το συνάδελφο Νίκο Αντωνόπουλο - Ίλιον, Αθήνα) Λύση (Από το συνάδελφο Σαμπά Θεόδωρο - Πάτρα)

Είναι: ( 1 ) <=> α(β 2 +γ2-α2 )+β(γ2+α2-β 2 ) +γ(α2+β2-γ2 ) =2αβγ 2 3 2 2 3 2 2 3 2 <::> αβ +αγ -α +βγ +βα -β +γα +γβ -γ _2αβγ=Ο

ΕΥΚΛΕΙΔΗΣ Β ' τ. Ι /76


Ο ΕυκλΕ ίδη ς πρ οτείνει ... ΕυκλΕ ίδη ... και Δ ιόφαντο

2 2 2 2 2 <=> α ( β-α) -β (β-α) +γ (α + β -2 α β) +γ

- 2 - 2 2<=> -( β α) ( β αz )+γ ( β α) +γz ( α+ β γ)=Ο

2 2 2 <=>-( β-α) ( β+ α) +γ( β-α) + γ (α+ β-γ)=Ο <:::> ( β-α) 2 (γ-β-α) + γ2 (α+ β-γ)= Ο <=> (α+ β-γ) [γz-( β-α)z ] =Ο <=> (α+ β-γ )(γ-β+ α )(γ+ β-α) =Ο

<:::> = α+ β ή β=α+ ή α= β+ γ γ γ Αν γ= α+ β τότε :

+ ( α + β ) 2 α2 2β 2 + 2 αβ β 2 ____. .:. ____:... -Ι 2 αβ ( α + β ) - 2β 2 + 2 αβ - '

_

_

_ _

( α + β ) 2 + α2 β 2 2 α2 + 2 αβ -'----'------Ι - - 2 α2 + 2 αβ - ' 2α ( α + β ) και -2 αβ α2 + β2 - γ2 - = α2 + β 2 ( α + β ) 2 = -= -Ι . 2 αβ 2 αβ 2 αβ Στο ίδιο συμπέρασμα καταλήγου με αν υπο θ έσου με ότι ισχύ ει β=α+γ ή α= β+γ

Λύ ση (από τον ίδιο)

Κατασκευ άζου με τρ ίγωνο Α 'Β Τ ' συμμετρικό του ΑΒΓ ως προς κέντρο συ μμετρίας το μέσον Τ του ΗΜ. Τα τρίγωνα Α'Β 'Γ, ΑΒΓ είναι ΙΣΑ (εκ κατασκευή ς , με πλευρές ίσες κ αι παρ άλληλες μία προς μία κ αι τα ο μόλογ α στοιχεία τους ίσα ένα προς ένα)

Λύ σ εις

έστειλαν επίσης οι συνάδελφοι Σταματογιάννης Γιάννης - Δροσιά Αττικής, Μάρκος Αθανάσιος - Κοζάνη και ο κύριος Ρ. Μπορής - Δάφνη Αθήνα.

45 (Τεύχ ους 54 ) με Δίνεται τυχαίο τρίγωνο ΑΒΓ ορθόκεντρο Η, τυχαίο σημείο Μ του περιγεγραμμένου κύκλου του και (s) η ευθεία Simson του Μ ως προς το τρίγωνο ΑΒΓ. Να αποδείξετε ότι η (s) είναι κοινή ευθεία Simson τριών ση μείων Α ', Β ',Γ ' ως προς τα τρίγωνα ΗΒΓ, ΗΓΑ, ΗΑΒ αντίστοιχα, όπου τα σημεία Α ', Β ',Γ ' είναι κορυφές τριγώνου ίσου προς το τρίγωνο ΑΒΓ και του οποίου το ορθόκεντρο είναι το σημείο Μ. (Επροτάθη

Τρι�-το

από τον συνάδελφο

Γ. Θωμαlδηc;. Γ. nετράκη<;. Κ. Τουλούμη<;. Μ. Σταφυλiδου

ΓΛΩΣΣΑ, Ι ΠΟ Ρ Ι Α

και

ΕΥΚΛ Ε Ι Δ Ε Ι Α ΓΕΩ Μ Η Ρ Ι Α

Μία δοκιμή διαθεματικής προσέγγισης στη Δευτεροβάθμια Εκπαίδευση

Το βιβλίο καλύmει τηv άμεση αvάγκη τωv Εκπαι·

δευτικώv για εvημέρωση σχετ ι κά με τη δια θεματι κ ή διδασκαλία. Παρουσιάζονται το υλικό και τα αποτε­ λέσματα μιας διετούς πειραματικής εφαρμογής δια­

θεματικών προσεγγίσεων στους τομείς της Γλώσσας,

τηc:; Ιστορίας και τωv Μαθηματικών, οε πραγματικέ<; σχολικές τάξεις. Έvα χρήσιμο εργαλείο για κάθε Ε κπαιδευτικό που καλείται vα εφαρμόσει τα vέα

διαθεματικά αvαλυτικά προγράμματα του ΥΠΕΠΘ.

Γιώργο

στο τεύχος 54) Ε\Κ\Ε ΙΔΗΣ Β ' τ. l /77

Εκδόσεις Πανεπιστημ ίου Μακεδονίας τ 23 ιο 891.743. 741 . & 73 1 F 23Ιο 891.730 Ε uompress@uom.gr, W uom.gr/uompress


Ο Ευκλείδης πρ οτείνει ... Ευκλείδη ... και Δ ιόφαντο

11

11

11

Επειδή είναι Β'Μ = ΒΗ , Γ'Μ = ΓΗ , Α'Μ = ΑΗ το σημείο Μ είναι ορθόκεντρο του τριγώνου Α ' Β 'Γ. Η ευθεία Simson (s) του σημείου Μ ως προς το τρίγωνο ΑΒΓ διέρχεται από τις ορθές προβολές Μ 1 , Μ 2 , Μ 3 του Μ πάνω στις πλευρές ΒΓ, ΑΓ, ΑΒ αντίστοιχα και από το σημείο Τ. (Βασική ιδιότητα της ευθείας Simson) Ονομάζουμε Ρ 1 , Ρ 2 , Ρ 3 τα σημεία τομής των πλευρών Α 'Β ' , Α 'Γ, Β 'Γ του τριγώνου Α 'Β 'Γ με τα ύψη ΓΖ, ΒΕ, ΑΔ του τριγώνου ΑΒΓ αντίστοιχα. Παρατηρούμε , ότι το σημείο Η ανήκει στον περιγεγραμμένο κύκλο C 'του τριγώνου Α ' Β ' Γ και ότι τα σημεία Ρι , Ρ 2 , Ρ3 είναι οι ορθές προβολές του Η πάνω στις πλευρές Α 'Β ' , Α 'Γ , Β 'Γτου Α ' Β 'Γ. Άρα ,τα σημεία αυτά ανήκουν στην ευθεία SIMSON (s ') του Η ως προς το τρίγωνο Α 'Β 'Γ, που διέρχεται και αυτή από το μέσον Τ του τμήματος ΗΜ. Θα αποδείξουμε ότι s = s ' . Επειδή το Τ είναι κ ο ιν ό μέ σ ον των Μ Ρ , Μ Ρ , είναι Μ ι Μ 3 II Ρ ι Ρ 3 , οπότε

ι 3 '

3 ι

II s . Επειδή οι Τ, ταυτίζονται. s

s, s ' έχουν κοινό το σημείο

Τα σημεία Μ , Ρ 1 , Ρ είναι οι ορθές προβολές 1 2 του σημείου Α ' πάνω στις πλευρές ΒΓ, ΗΓ, ΗΒ του τριγώνου ΗΒΓ και επειδή είναι συνευθειακά, το σημείο Α ' ανήκει στον περιγεγραμμένο κύκλο του τριγώνου ΗΒΓ. Όμοια, Τα σημεία Μ ,Ρ ι ,Ρ3 είναι οι ορθές

2

προβολές του σημείου Β ' πάνω στις πλευρές ΑΓ, ΗΓ, ΗΑ του τριγώνου ΗΑΓ και επειδή είναι συνευθειακά, το σημείο Β ' ανήκει στον περιγεγραμμένο κύκλο του τριγώνου ΗΑΓ.

οποιωνδήποτε θετικών αριθμών α και β, υπάρχει αριθμός γ ε (-1 , 1) ώστε να ισχύει: α f(γ)f'(-γ)=βf'(γ) f(-γ) (Προτείνεται από τον Ακαδημαϊκό κ. Νικόλαο Αρτεμιάδη) 1 00. Έστω f, g: [α,β] � JR δυο συνεχείς συναρτήσεις ώστε για κάθε χ ε [α,β] να ισχύει f(x);:::O και g(x);:::O . Αν υπάρχει Α ε JR με Α;:::Ο ώστε f(x) ::; A +

,

,

χ ε [α,β] να αποδείξετε οτι: f(x) ::; Ae

r g(t)d(t)

(Προτείνεται από το συνάδελφο Κυριακόπουλο Θανάση - Αθήνα) 1 0 1 . Δίνεται ισοσκελές τρίγωνο ΑΒΓ(ΑΒ=ΑΓ). Αν υπάρχουν σημεία Δ,Ε στις πλευρές ΑΓ ,ΑΒ του τριγώνου ώστε να ισχύει ΓΔ=ΔΕ=ΕΑ=ΒΓ, να υπολογίσετε τη γωνία Α του τριγώνου. (Προτείνεται από το συνάδελφο Γκουντουβά Σωτήρη - Αργυρούπολη) 1 02 . Δίνονται δυο σταθερά σημεία Α,Β και μεταβλητό σημείο Ν. Αν Ν 1 είναι το συμμετρικό του Ν ως προς το Α, Ν2 στην προέκταση της Ν1Β ώστε να ισχύει ΒΝι=2ΒΝz και Μι το μέσο της ΒΝ, να αποδείξετε ότι το κέντρο βάρους του τριγώνου Ν ι N z Μι διατηρείται σταθερό, ανεξάρτητα από τη θέση του σημείου Ν. (Προτείνεται από τον χημικό Καρβελά Δημήτρη - Πεύκη Αττικής) Οι επιστολές των φίλων της στήλης που έφτασαν στα γραφεία της Ε . Μ . Ε . μετά την 1 51612006 δ εν έχουν λη φθεί υ πό ψιν στο τε ύ χος αυτό.

Ελληνική Μαθηματική Εtαι.Qεία

Ι ΙΑΝΕΛΛΗΝΙΟ ΣΥΝΕΔΡΙΟ ΜΑΘΗΜΑτΙΚΗΣ ΙΙΑΙΔΕΙΑΣ Με διε θνε ίς συμ μ ετοχ έ ς

Τα σημεία Μ , Ρ , Ρ είναι οι ορθές προβολές 3 2 3 του σημείου Γ πάνω στις πλευρές ΑΒ, ΗΒ, ΗΑ του τριγώνου ΗΑΒ και επειδή είναι συνευθειακά, το σημείο Γ ανήκει στον περιγεγραμμένο κύκλο του τριγώνου ΗΑΒ . Β. ΑΣΚΗΣΕΙΣ ΓΙΑ ΛΥΣΗ

f r(t)g(t)dt για κάθε

Πάτ ρ α 24 - 25 - 26 Νοεμβρ ίου 2006 ΤΑ JIAAtiii JIIA T Ι it A D % ΠΟΑΙΤΙ% /ΙΙ Ο Ζ. % Τ Ο 'L Y F X Po•o lt O 'L JIAO , Π ΡΟ8ΙιΤΑ% 8 Ι % 'L T II .

99. Έστω f μια συνάρτηση η οποία είναι συνεχής στο [ 0,1] και παραγωγίσιμη στο (0,1 ) Αν f(1)=0 και f(x)>O για κάθε x e (-1 ,1), να αποδείξετε ότι δοθέντων δυο

lt PIT Ιltll 'L it . Ψ II .

Τ ΙΙ • 8 Π Ι Χ 8 Ι ΡΙΙΙΙΑΑΤΟΑΟ F Ι Α ltAI

ΕΥΚΛΕΙΔΗΣ Β ' τ. Ι/78

Τ 11• A Ι 'L ti ii T Ι lt ll


Τα Μαθηματικά μας δ:tαο:κε:ΟΟζουν Τα μαθηματικά αν και είναι επιστήμη που απαιτεί αυστηρή διατύπωση, έχουν τη μαγεία να αποσπούν το ενδιαφέρον όλων των ανθρώπων. Επινοήσεις σε προβλήματα ή ασκήσεις με κατάλληλο τρόπο διατυπωμένα εξάπτουν το πνεύμα, διεγείρουν τη φαντασία και κεντρίζουν την περιέργεια. Πρώτοι οι Αρχαίοι Έλληνες όπως ο Δ ιόφαντος, ο Ζήνωνας κ. ά. μας δίδαξαν αυτά τα μαθηματικά. Στη στήλη αυτή θα παρουσιάζουμε θέματα τα οποία δεν απαιτούν ιδιαίτερες μαθηματικές γνώσεις αλλά μας διασκεδάζουν με την εκφώνησή τους ή τη λύση τους και είναι μια ευχάριστη και συναρπαστική ασχολία •

Επιμέλεια: Παναγιώτη ς Χρ ιστόπουλος

Στο καταπληκτικό του μυθιστόρημα ο DAN αριθμούς αναφέρει τους BROWN Ι , Ι ,2,3 ,5,8, 1 3 ,2 Ι που είναι 8 πρώτοι όροι μιας ακολουθίας. Προσθέστε τους 2 τελευταίους να βρείτε τον επόμενο, συνεχίστε. Τώρα διαιρέστε καθένα από αυτούς με τον προηγούμενο.

Το χαρτζιλίκι

Ο πατέρας του Πέτρου μαθητή της Β ' Λυκείου για να διαπιστώσει αν ο υιός του είναι καλός στα μαθηματικά του είπε: <<Κάθε μέρα θα σου δίνω χαρτζιλίκι 1 00 Ευρώ και εσύ θα μου επιστρέφεις την l η μέρα 1 λεπτό, τη 2η μέρα 2 λεπτά, την 3η μέρα 4 λεπτά και κάθε μέρα διπλάσια λεπτά από την προηγούμενη. Α υτό

θα γίνει από την πρώτη μέχρι την τελευταία μέρα του μήνα Οκτώβρη. »

Τι λέτε πρέπει να

δεχθεί ο Πέτρος ή όχι;

Τα χρήματα4

Η διαπίστωση είναι ότι όλα τα αποτελέσματα προσεγγίζουν τον αριθμό Ι ,6 Ι 803 . . . . . . που λέγεται Φ «χρυσή τομή». Αν τον aντιστρέψετε απλά θα χάσετε το ακέραιο μέρος του. Μήπως ο αριθμός αυτός κρύβει το Άγιο Δισκοπότηρο και δεν το κατάλαβε ο DAN BROWN;

Ξέρετε ότι τα χρήματα κυκλοφορούν σε Ι €, σε 2 €, σε 5€ στις υποδιαιρέσεις δεκά'fου ( 1 0,20,50 λεπτά) και εκατοστού ( 1 ,2,5 λεπτά) κάθώς και στα πολλαπλάσια δεκάδων ( Ι Ο,20,50 €), εκατοντάδων ( Ι ΟΟ,200,500 €), σε νομίσματα άλλων χωρών έχουμε και πολλαπλάσια μεγαλύτερα. Να βρείτε τα λιγότερα τεμάχια χρημάτων, χωρίς να λείπει κανένα είδος, που δίνουν Ι 000 €. Αν κυκλοφορούσε το € σε 1 , 3 , 5 θα είχαμε τα ίδια τεμάχια; Ο

Αριθμομνήμον

Ζήτησε έναν 3 ψήφιο, πολλαπλασίασε τον με

ΕΥΚΛΕΙΔΗΣ Β ' τ.Ι/79


Τ α Μαθηματικά μας Δ ιασκεδάζουν

έναν άλλο 3ψήφιο αριθμό που θα σου δώσουν, αλλά και με έναν 3ψήφιο αριθμό που θα σου δώσει ο φίλος σου και πές το άθροισμα των δύο γινομένων, όλα αυτά από μνήμης ! ! ! -Πώς; Εύκολο. Έστω ότι σας έδωσαν τον 764 χ 265 και 764χ 734 φίλος το άθροισμα των γινομένων είναι 763 236. (το μυστικό σσσσς ! ! ! Ο φίλος δίνει 734 διότι με το 265 θα έχουμε 999 και το αποτέλεσμα; 764 - l και τα αντίστοιχα 236 για να έχουμε 999. Ο Κ\)βος και τ ο Τ�:τ ράγωνο Ποιος είναι ο 2ψήφιος που είναι και τετράγωνο και κύβος; Υπάρχει και 3ψήφιος; Τα κελι ιi

Έχουμε ι 00 κελιά αριθμημένα από το ι έως το 1 00 και 1 00 δεσμοφύλακες με τους αριθμούς από το ι έως το 1 00. Ο κάθε δεσμοφύλακας περνάει από το πρώτο κελί και από τα κελιά που έχουν αριθμό πολ/σιο του αριθμού του. Στην αρχή όλα τα κελιά είναι κλειστά .Ο κάθε δεσμοφύλακας από το κάθε κελί που περνάει, αν το βρει κλειστό το ανοίγει, ενώ αν το βρει ανοικτό το κλείνει. Όταν περάσουν όλοι οι δεσμοφύλακες πόσα κελιά θα είναι ανοικτά; (Το έστειλε ο συνάδελφος Σωτήρης Γκουντουβάς). Ο fΗ ΙΟtΗ ω τ ικός

Σε ένα σχολείο διορίστηκε πρόσφατα νέος μαθηματικός ο οποίος το χ 2 έτος θα είναι χ χρόνων, τι ηλικία έχει σήμερα; Η Γη

Αν υποθέσουμε ότι η ΔΕΗ έχει•ένα καλώδιο γύρω από τη Γη, στον Ισημερινό, πάνω σε πυλώνες ύψους 5 μέτρων και αποφασίζει να σκάψει και να το τοποθετήσει μέσα στη Γη σε βάθος 5 μέτρα. Πόσα μέτρα καλώδιο θα εξοικονομήσει;

Ολα Ε ίναι 992

Γράψτε ένα 3ψήφιο π.χ. 267 αλλάξτε τη σειρά των ψηφίων, 762, τους αφαιρούμε 762-267=495 , στο αποτέλεσμα αλλάξτε τη σειρά, 594, τους προσθέτουμε 495+594= 089, αλλάζουμε πάλι τη σειρά στο αποτέλεσμα, 980 1 =99 2 Δοκιμάστε με όποιο 3ψήφιο θέλετε. •

\ Ι άντεψ ε

Ζητήστε από το φίλο σας να γράψει ένα 3ψήφιο αριθμό και να επαναλάβει τα ψηφία του ώστε να γίνει 6ψήφιος. Ύστερα να διαιρέσει τον 6ψήφιο διαδοχικά με τους 7, ι ι , ι 3 και θα έχει τον αριθμό του. ΊΌ παπιη\τσ ι

Ζητήστε από το φίλο σας να γράψει σε ένα χαρτί το νούμερο του παπουτσιού του (π.χ.44) να του προσθέσει 5 μονάδες στη συνέχεια να πολλαπλασιάσει με το 50 από το αποτέλεσμα να αφαιρέσει το έτος γέννησης του και ύστερα να προσθέσει τον ι 76 ι , τώρα σας ενημερώνει μόνο για το αποτέλεσμα στο οποίο τα δύο πρώτα ψηφία είναι το ζητούμενο νούμερο. Ο πατiφα..;

Ο Γιώργος δεν είχε ούτε ένα Ευρώ και του έδωσε ο πατέρας του 200€. Η Ελένη επίσης δεν είχε ούτε ένα Ευρώ και της έδωσε ο πατέρας της ι 50€. Συμφώνησε ο Γιώργος με την Ελένη να αγοράσουν με τα χρήματά τους ένα ποδήλατο, πλήρωσαν ι 98€ και τους περίσσεψε από ι € πώς έγινε αυτό; Ο φοιτητίtς Όταν η κόρη πέρασε σε Πανεπιστήμιο άλλης πόλης ο πατέρας της κατέθεσε σε μια τράπεζα ένα ποσό και της έδωσε το βιβλιάριο. Η κόρη κάθε μήνα έπαιρνε για τα έξοδά της ι 000 Ευρώ. Τον πρώτο μήνα ο πατέρας βλέποντας το ποσό που έμεινε το αύξησε κατά το ι /3 του. Αυτό έγινε τον επόμενο και το μεθεπόμενο μήνα τότε ο πατέρας διαπίστωσε ότι το ποσό ήταν το ίδιο με το αρχικό. Ποιο ποσό είχε καταθέσει αρχικά; Η Ηλικία

Στον τύπο Α = (2 p 1 ) 4 + p � + χ αντικαταστήστε τους p 1 , p 2 με πρώτους αριθμούς μεγαλύτερους του 5 και το χ με την ηλικία σας. από το αποτέλεσμα μπορούμε να βρούμε την ηλικία σας, εσείς; ( Το έστειλε ο χημικός Δημήτριος Καρβελάς) ΕΥΚΛΕΙΔΗΣ Β ' τ. Ι /80


,

Ί

α .,

\

,

ια

από τις εκδόσεις << Εν Δυνάμει>>

Για

την

r· Λυκείου

Φυσική 1Κστεύ8υνσηc; (ταλαντώσεις - Κί.ιματα) - Γ. Θ. Ντοί.ιβαλης • Φυσική Κcιτεύ8υνσηc; (Μηχανική του στερεοί.ι σώματος) - Γ. θ. Ντοί.ιβαλης • Φυσική Κατεύβυνσηc; (Κροί.ισεις - Φαινόμενο Doppler) - Γ. θ. Ντοί.ιβαλης •

Ασκήσeιc; Βιολοyίας Γενικής Παιδείας - Κ. Ρ. Παπαζήσης Μα8πματικά Κστεύ8υνσης (Παράγωγοι) - Γ. & Π. Λουκόποuλος • Μcιβnμcπ:ικά Κcπεύβυνσης (Ολοκληρώματα) - Γ. & Π. Λουκόποuλος • · εκφρασπ - · εκecση - Α. Καλλή - Γ. Σοuλτάνης • Ανάπτuξπ εφαρμογών σe προyραμμσnστικό περιβάλλον - Κ. Ν. Ιορδανόποuλος Αρχές Οικονομικής θεωρίας - Κ. Γαροφαλάκης •

Φυσική - Κ. Ρ. Παπαζήσης

"Άλycβρα

...

·:a w::;

- Γ. & Π. Λοuκόπουλος

:z:

Εv Δuvάμειι Ε Κ Δ Ο Σ

ΧΡΥΣΙΠΠΟΥ 1

& ΟΥΛΟΦ ΠΑΛΜ Ε

-

Ε Ι Σ

ΖΩΓΡΑΦΟΥ

Τηλ. : 2 1 Ο 7 4 88 030,

fax: 2 1 Ο 74 83 0 3 1


·•

Πανεπιστη μίου (Ελευθερίου Βενιζέλου) 34 - 1 06 79 ΑΘΗΝΑ Τηλ. 3 6 1 65 3 2 - 3 6 1 7 784 - Fax: 364 1 025

ΕΛΛΗΝΙΚΉ ΜΑΘΗΜΑΤΙΚΉ ΕΤΑΙΡΕΙΑ www.hms.gr - e-mail : info@hms.gr

Παίγνια και Λήψη Αποφάσεων

Σελίδες 3 1 8

ISBN: 960-73 4 1 -27-9.

Σχήμα

Ν Ε ΕΣ Ε ΚΔΟΣ Ε Ι Σ ΕΛΛΗΝΙΚΗ ΜΑΘΗΜΑτtΚΗ ΕΤΑΙΡΕΙΑ

I

1 9Χ23 Λ. Τ. 2 8 €

Το βιβλίο πραγματεύεται με ενιαίο τρόπο το αντικείμενο της Θεωρίας Παιγνίων και της κλασικής Θεωρίας Βελτιστοποίησης υπό το πρίσμα των εφαρμογών των θεωριών και των τεχνικών τους στα οικονομικά, τις πολιτικές επιστήμες και τη διοίκηση επιχειρήσεων. Αποφεύγοντας τη σχολαστικότητα, αλλά χωρίς να υστερεί σε μαθη ματική αυστηρότητα, το βιβλίο απευθύνεται κυρίως σε προπτυχιακούς αλλά και μεταπτυχιακούς φοιτητές των οικονομικών, θετικών και τεχνολογικών σχολών, καθώς και σε όσους ενδιαφέρονται εν γένει για τη Θεωρία Παιγνίων και τη Θεωρία Αποφάσεων. Ο μεγάλος αριθμός των επεξεργασμένων παραδειγμάτων και των προς επίλυση ασκήσεων που καλύπτουν ένα ευρύτατο φάσμα εφαρμογών, εισάγουν βαθμιαία τον αναγνώστη στη μεθοδολογία και τη λογική της θεωρίας, και τον διδάσκουν πώς να αξιολογεί και να εφαρμόζει κατάλληλα τις σχετικές τεχνικές

Χ. Δ� Αλιπρόντn

και

,..

5. Κ. Chakrabartl

ΗΡΩΝΟΣ ΑΛΕΞΑΝΔΡΕΩΣ

Η Ελληνική Μαθηματική Εταιρεία με μεγάλη ικανοποίηση και υπερηφάνεια παραδίδει το πολύ ση μαντικό αυτό έργο όχι μόνο στους Μαθηματικούς αλλά και σε όλους αυτούς που θέλουν να μελετήσουν σε βάθος το Ηρωνικό έργο. Ο Αλεξανδρινός «μηχανικός» Ηρών εκτός από τις περίφημες πραγματείες του που αναφέρονται σε θέματα κατασκευής και λειτουργίας διαφόρων μηχανών, μας έδωσε και αξιόλογα μαθηματικά κείμενα. ·

&,!iJMJi@jjCJ'\If.Λ6f?·..· ,....-----,__-· 4&!

ΗΡΩΝΟΣ ΑΛΕΞΑΝΔΡΕΩΣ

ΗΡΩΝΟΣ ΑΛΕΞΑΝΔΡΕΩΣ

ΟΝΟΜΑΤΑ ΓΕΩΜΕΤΡΙΚΩΝ

ΜΕ ΤΡ!ΚΑ

Σχήμα 1 7Χ24 Σελίδες 328

ΟΡΩΝ ΓΕΩΜΕΤΡΙΚΆ

Σχήμα 1 7Χ24 Σελίδες 5 1 6 ISBN: 960-734 1 - 1 6-3

- ΔΙΟΠΤΡΑ

ISBN: 960-734 1 -22-8 Λ.Τ. 20 €

ΗΡΩΝΟΣ ΑΛΕΕΑΝΔΡΕΩΣ ΣΤΕΡΕΟιΊ4ΕrΡΙΚΑ

Σχήμα 1 7Χ24 Σελίδες 4 1 4 ISBN: 960-7341 -26-0 Λ.Τ. 20 €

Λ.Τ. 2 0 €

ΑΣΤΡΟΜΒΟΣ

ΑΣΤΡΟΛΑΒΟΣ

Σχήμα 1 7Χ24 Σελίδες 1 20 ISSN: 1 1 06-2878 Λ.Τ. 10 €

Στο περιοδικό ΑΣΤΡΟΛΑΒΟΣ δημοσιεύονται επιστημονικές εργασίες πάνω σε σύγχρονα θέματα που εμπίπτουν στο περιεχόμενο και στους στόχους του όπως: Διδακτική και Νέες Τεχνολογίες, Εξ Αποστάσεως Εκπαίδευση , Τεχνητή Νοημοσύνη, Προγραμματισμός, Τεχνολογίες Διαδικτύου, Ηλεκτρονικό Εμπόριο, Λειτουργικά Συστήματα, Αρχιτεκτονική Υπολογιστών, Αριθμητική Ανάλυση, Υπολογιστικά Μαθη ματικά και Εφαρμογές, Αλγόριθμοι, Θεωρία Πληροφοριών, Υπολογιστική Γεωμετρία κ.λ.π . . Ο κοινός τόπος των άρθρων που θ α δημοσιεύονται είναι: • Η συμβολή των Μαθη ματικών στη θεμελίωση και εξέλιξη της Επιστή μης της Πληροφορικής. • Η παιδαγωγική και διδακτική αξιοποίηση των Νέων Τεχνολογιών στην εν γένει εκπαιδευτική διαδικασία.

Συνάδελφοι Επισκεφθείτε την ιστοσελίδα της Ε.Μ.Ε. (www.hms.gr ) όπου υπάρχουν όλα τα τεύχη του Ευκλέιδη Γ και της Μ αθηματικής Επιθεώρησης με τα περιεχόμενά τους καθ(ος και όλες οι εκδόσεις της Ελληνικς Μ αθηματικής Εταιρείας (Ε .Μ.Ε.).


Issuu converts static files into: digital portfolios, online yearbooks, online catalogs, digital photo albums and more. Sign up and create your flipbook.